You are on page 1of 284

SOLVED PAPERS

Physics • Chemistry • Biology • Mathematics • History & Civics • Geography


English Language-I • English Language-II • Hindi • Commercial Studies
Commercial Applications • Economics • Economic Applications
Computer Applications • Physical Education
ICSE EXAMINATION QUESTION PAPER – 2012 (SOLVED)
Time : 1½ hours 80 Marks
Answers to this paper must be written on the paper provided separately.
You will not be allowed to write during the first 15 minutes.
This time is to be spent in reading the Question Paper.
The time given at the head of this paper is the time allowed for writing the answers.
Section I is compulsory. Attempt any four questions from Section II.
The intended marks for questions or parts of questions are given in brackets [ ].

SECTION – I (40 Marks)


(Attempt all questions from this section)
Question 1.
(a) (i) Define 1 kgf.
(ii) How is it related to the S.I. unit of force ? [2]
(b) (i) What are non-contact forces ?
(ii) How does the distance of separation between two bodies affect the magnitude of the
non-contact force between them ? [2]
(c) A boy of mass 30 kg is sitting at a distance of 2 m from the middle of a see-saw. Where
should a boy of mass 40 kg sit so as to balance the sea-saw ? [2]
(d) (i) What is meant by the term ‘moment of force’ ?
(ii) If the moment of force is assigned a negative sign then will the turning tendency of
the force be clockwise or anticlockwise ? [2]
(e) A ball is placed on a compressed spring. When the spring is released, the ball is observed
to fly away.
(i) What form of energy does the compressed spring possess ?
(ii) Why does the ball fly away ? [2]

Compressed spring
Answer.
(a) (i) The force with which a mass of 1 kg is attracted towards the centre of the earth, due
to the acceleration due to gravity is called 1 kgf.
(ii) 1 kgf = 1 kg × acceleration due to gravity
= 1 kg × 9.8 ms–2 = 9.8 N
(b) (i) The forces which do act on bodies without being physically touched are called the
non-contact forces.
(ii) The magnitude of non-contact forces acting between the two bodies is inversely
proportional to the square of distance between the mass centres of the two bodies.
Ph 1 2012
Ph 2 2012

(c) Let ‘x’ be the distance of 40 kg boy from the middle of see-saw.
 Moments due to 40 kg boy = Moments due to 30 kg boy
40 kg × x = 30 kg × 2 m
30 kg  2 m
 x= 40 kg = 1.5 m

(d) (i) The turning effect of a force about a fixed point or a fixed axis is called moment of
force.
(ii) Negative sign of the moment of force implies that turning tendency of the force is in
anticlockwise direction.
(e) (i) The compressed spring possesses potential energy.
(ii) The potential energy of the spring on releasing changes to kinetic energy. It is the
kinetic energy which makes the ball to fly away.
Question 2.
(a) (i) State the energy conversion taking place in a solar cell.
(ii) Give one disadvantage of using a solar cell. [2]
(b) A body of mass 0.2 kg falls from a height of 10 m to a height of 6 m above the ground.
Find the loss in potential energy taking place in the body. [g = 10 ms–2] [2]
(c) (i) Define the term refractive index of a medium in terms of velocity of light.
(ii) A ray of light moves from a rare medium to a dense medium as shown in the diagram
below. Write down the number of the ray which represents the partially reflected ray.
[2]
Ray 1

Ray 2

Rare medium
Dense medium

Ray 3

(d) You are provided with a printed piece of paper. Using this paper how will you differentiate
between a convex lens and a concave lens ? [2]
(e) A ray of light incident at an angle of incidence ‘i’ passes through an equilateral glass prism
such that the refracted ray inside the prism is parallel to its base and emerges from the
prism at an angle of emergence ‘e’.
(i) How is the angle of emergence ‘e’ related to the angle of incidence ‘i’ ?
(ii) What can you say about the value of the angle of deviation in such a situation ? [2]
Answer.
(a) (i) In a solar cell, the light energy directly changes to electric energy.
Ph 3 2012

(ii) Solar cell does not produce electric energy during night or in darkness.
(b) Loss in potential energy = mass × g × loss of height
= 0.2 kg × 10 ms–2 × 4 m = 8 J
(c) (i) Refractive index of a medium is the ratio of velocity of light in vacuum or air to the
velocity of light in a given medium.
(ii) Ray 2, represents partially reflected ray.
(d) Hold each of the lens 5 cm above the printed paper and look for the image. In case of
convex lens the print appears enlarged. However, in case of concave lens, the print appears
diminished.
(e) (i) Angle of incidence is equal to the angle of emergence.
(ii) The angle of deviation is minimum in this particular case.
Question 3.
(a) (i) What is meant by ‘Dispersion of light’ ?
(ii) In the atmosphere which colour of light gets scattered the least ? [2]
(b) Which characteristic of sound will change if there is a change in
(i) its amplitude (ii) its waveform. [2]
(c) (i) Name one factor which affects the frequency of sound emitted due to vibrations in
an air column.
(ii) Name the unit used for measuring the sound level. [2]
(d) An electrical appliance is rated at 1000 kVA, 220V. If the appliance is operated for 2 hours,
calculate the energy consumed by the appliance in :
(i) kWh (ii) joule
(e) Calculate the equivalent resistance between P and Q from the following diagram : [2]

Answer.
(a) (i) The phenomenon due to which white light splits into component colours on passing
through a prism is called dispersion of light.
(ii) Red colour scatters least in the atmosphere.
(b) (i) With the change in amplitude, the loudness of sound changes.
(ii) With the change in waveform the quality of sound changes.
(c) (i) The length of vibrating air column affects its frequency. More the length of vibrating
air column, lesser is its frequency.
(ii) Decibel (dB) is the unit used for measuring sound level.
Ph 4 2012

(d) (i) Energy consumed in kWh = 1000 kVA × 2h = 2000 kWh.


(ii) Energy consumed in Joules = 1000 × 1000 VA × 2 × 3600 s
= 7,200,000,000 J
(e) Resistance of two 10 resistance in series = 10 + 10 = 20
 Equivalent resistance of 20 in parallel with 5 resistance

1 1 1 1 4 5 1
R p = 20 + 5 = 20 = 20 = 4  Rp = 4

 Resistance between P and Q in series = 3 + 4 + 2 = 9


Question 4.
(a) (i) What is an a.c. generator or Dynamo used for ? [2]
(ii) Name the principle on which it works.
(b) Differentiate between heat capacity and specific heat capacity. [2]
(c) A hot solid of mass 60 g at 100ºC is placed in 150 g of water at 20ºC. The final steady
temperature recorded is 25ºC. Calculate the specific heat capacity of the solid. [Specific
heat capacity of water = 4200 J kg–1ºC–1] [2]
(d) (i) What is the value of the speed of gamma radiations in air or vacuum ?
(ii) Name a material which exhibits fluorescence when cathode rays fall on it. [2]
(e) Give any two important sources of background radiation. [2]
Answer.
(a) (i) Dynamo is used for conversation of mechanical energy into electric energy.
(ii) Principle of A.C. Generator : When a conductor is rotated in a magnetic field, the
magnetic flux linked with it changes and e.m.f. is induced in the coil.
(b) Heat capacity is the amount of heat required to raise the temperature of a given mass of
substance through 1 K or (1ºC).
Specific heat capacity is the amount of heat required to raise the temperature of 1 kg (or
1g) of a substance through 1K (1ºC).
(c) msolid × Csolid × fall = mwater × Cwater× R

60 150
kg × Csolid × 75ºC = kg × 4200 Jkg–1ºC–1 × 5ºC
1000 1000

150  4200  5  1000


 Csolid = J kg–1ºC–1 = 700 J kg–1ºC–1
1000  60  75
(d) (i) The speed of gamma rays () is 3 × 108 ms–1.
(ii) Zinc sulphide, Barium platinocyanide show fluorescence when cathode rays fall on
it.
(e) (i) The radioactive emissions given out by the earth.
(ii) Sources are K-40, C-14 and Radium contained inside our body.
Ph 5 2012

SECTION – II (40 Marks)


(Attempt any four questions from this Section)
Question 5.
(a) (i) Which of the following remains constant in uniform circular motion : Speed or Velocity
or both ?
(ii) Name the force required for uniform circular motion. State its direction ? [3]
(b) (i) State the class of levers and the relative positions of load (L), effort (E) and fulcrum
(F) in each of the following cases.
1. a bottle opener
2. Sugar tongs.
(ii) Why is less effort needed to lift a load over an inclined plane as compared to lifting
the load directly ? [3]
(c) (i) A moving body weighing 400 N possesses 500 J of kinetic energy.
Calculate the velocity with which the body is moving. (g = 10 ms–2)
(ii) Under what condition will a set of gears produce –
1. a gain in speed
2. a gain in torque. [4]
Answer.
(a) (i) Speed remains constant during uniform circular motion.
(ii) Centripetal force. It is always directed towards the centre of the circular path.
(b) (i) 1. In case of bottle opener the load (L) acts in the middle of metal crown, the
fulcrum (F) at the edge of metal crown and effort (E) at the end of the handle.
It is a lever of second order.
2. Sugar tongs is a lever of third order. The sugar cube at the ends of tongs acts as
load (L), the effort (E) acts in the middle and fulcrum (F) acts at its end.

1 1
(ii) Here, mechanical advantage = sin θ  sin  = Mechanical advantage

If a body rises 1 m vertically, when it moves along the inclined plane by 100 m.
Then sin  = 1/100. Higher the magnitude of the gradient, more difficult, more difficult
is the slop to climb and vice-versa. Thus keeping gradient as low as possible which
helps in using lesser effort for lift a load over inclined plane as compared to lifting the
load directly.
(c) (i) Force acting on body (F) = mg
400 N = m × 100 ms–2
 Mass of body (m) = 400 N  10 ms–2 = 40 kg.

1
Kinetic energy of body (KE) = mv2
2
Ph 6 2012

1
500 J = × 40 kg v2
2

500 2 –2
v2 = m s
20

 v= –1
25 = 5ms
(ii) 1. When the radius of driven wheel is less than the driving wheel.
2. When the radius of driven wheel is more than the driving wheel.
Question 6.
(a) (i) What is meant by the term ‘critical angle’ ?
(ii) How is it related to the refractive index of the medium ?
(iii) Does the depth of a tank of water appear to change or remain the same when viewed
normally from above ? [3]
(b) A ray of light PQ is incident normally on the hypotenuse of a right angled prism ABC as
shown in the diagram given alongside :

A P

B C

(i) Copy the diagram and complete the path of the ray PQ till it emerges from the prism.
(ii) What is the value of the angle of deviation of the ray ?
(iii) Name an instrument where this action of the prism is used. [3]
(c) A converging lens is used to obtain an image of an object placed in front of it.
The inverted image is formed between F2 and 2F2 of the lens.
(i) Where is the object placed ?
(ii) Draw a ray diagram to illustrate the formation of the image obtained. [4]
Answer.
(a) (i) The angle of incidence in a denser medium for which angle of refraction in rarer
medium is 90º is called critical angle.
1
(ii) Refractive index () = sin C

(iii) The depth of tank remains same when viewed normally from above.
Ph 7 2012

(b) (i) P

A
Q
U

45°
R 45°
T
45°45°
B
S C
(ii) The ray deviates through 180º.
(iii) Binoculars use this action of right angled prism.
(c) (i) The object is anywhere between 2F1 and infinity.
(ii)
A

B O F2 B1 2F

2F1 F1
A1
Converging lens
Question 7.
(a) (i) What is meant by Resonance ?
(ii) State two ways in which Resonance differs from Forced vibrations. [3]
(b) (i) A man standing between two cliffs produces a sound and hears two successive
echoes at intervals of 3 s and 4 s respectively. Calculate the distance between the two
cliffs. The speed of sound in the air is 330 ms–1.
(ii) Why will an echo not be hears when the distance between the source of sound and
the reflecting surface is 10 m ? [3]
(c) The diagram alongside shows the displacement-time graph for a vibrating body.

(i) Name the type of vibrations produced by the vibrating body.


(ii) Give one example of a body producing such vibrations.
(iii) Why is the amplitude of the wave gradually decreasing ?
(iv) What will happen to the vibrations of the body after some time ?
Ph 8 2012

Answer.
(a) (i) The phenomenon due to which the natural frequency of a given body corresponds to
the frequency of sound impressed on it, such that it rapidly starts vibrating is called
resonance.
(ii) 1. The resonance takes place only when the natural frequency of a given body is
equal to the frequency of sound impressed on it, whereas during forced vibration
a body is forced to vibrate with the frequency of sound impressed on it.
2. Loud sound is produced during resonance, but not in case of forced vibrations.

vt 300 ms 1  7 s
(b) (i) Distance between two cliffs = = = 1155 m.
2 2
(ii) The persistence of sound on ear drum is 1/10 of a second. The echo can be heard if
the minimum distance of the source of sound from the vibrating body is 17 m. As the
distance is only 10 m, therefore, no echo is produced.
(c) (i) Transverse vibrations are produced which are gradually damped.
(ii) A stretched string of a guitar.
(iii) As the energy of wave is dissipated its amplitude decreases.
(iv) The body will stop vibrating.
Question 8.
(a) (i) A cell is sending current in an external circuit. How does the terminal voltage compare
with the e.m.f. of the cell ?
(ii) What is the purpose of using a fuse in an electrical circuit ?
(iii) What are the characteristic properties of fuse wire ? [3]
(b) (i) Write an expression for the electrical energy spent in the flow of current through an
electrical appliance in terms of I, R and t.
(ii) At what voltage is the alternating current supplied to our houses ?
(iii) How should the electric lamps in a building be connected ? [3]
(c) Three resistors are connected to a 6 V battery as shown in the figure given alongside :

Calculate :
(i) the equivalent resistance of the circuit.
(ii) total current in the circuit.
(iii) potential difference across the 7.2  resistor.
Ph 9 2012

Answer.
(a) (i) Terminal voltage is slightly less than the e.m.f. of the cell.
(ii) The fuse wire melts and stops the flow of electric current in a given circuit, in case
the circuit is overloaded or short circuited.
(iii) 1. Fuse wire should have low melting point around 200ºC.
2. Fuse wire should have high electrical resistance.
(b) (i) Electrical energy (E) = I2.R.t.
(ii) Alternating current is supplied at 220V for domestic consumption.
(iii) All lamps should be connected in parallel.
(c) (i) Resistance of 8 and 12 resistors in parallel.

1 1 1 3 2 5
R p = 8 + 12 = 24 = 24

24
 Rp = = 4.8
5
Equivalent resistance of circuit = Rp + 7.2 = 4.8 + 7.2 = 12
V 6V
(ii) Current in circuit I = = = 0.5 A.
R 12
(iii) Potential difference across 7.2 resistor
V = I.R = 0.5 A × 7.2 = 3.60V
Question 9.
(a) (i) Write an expression for the heat energy liberated by a hot body.
(ii) Some heat is provided to a body to raise its temperature by 25ºC.
What will be the corresponding rise in temperature of the body as shown on the
kelvin scale ?
(iii) What happens to the average kinetic energy of the molecules as ice melts at 0ºC ?
[3]
(b) A piece of ice at 0ºC is heated at a constant rate and its temperature recorded at regular
intervals till steam is formed at 100ºC. Draw a temperature – time graph to represent the
change in phase. Label the different parts of your graph. [3]
(c) 40 g of ice at 0ºC is used to bring down the temperature of a certain mass of water at 60ºC
to 10ºC. Find the mass of water used.
[Specific heat capacity of water = 4200 J kg–1 ºC–1]
[Specific latent heat of fusion of ice = 336 × 10 3 J kg–1] [4]
Answer.
(a) (i) Heat liberated by a body = mass × sp. heat capacity × fall in temperature
= m × C × fall
(ii) The rise in temperature on kelvin scale will be 25 K.
Ph 10 2012

(iii) The average kinetic energy of the molecules remains same.


(b)

D E
100

Temp (ºC)
B C
0°C

A
Time (s)

(c) Heat absorbed by ice to form water at 0ºC = 40 g × 336 Jg–1 = 13440 J.
Heat absorbed by water at 0º to attain temperature of 10ºC = mCR
= 40 g × 4.2 Jg–1 ºC–1 × 10ºC = 1680 J.
 Total heat absorbed = (13440 + 1680) J = 15120 J
Heat given out by water at 60ºC = mCF
= m × 4.2 Jg–1 ºC–1 × 50ºC = 210 m J g–1
Now, Heat given out = Heat absorbed
 210 m Jg–1 = 15120 J
15120
 m= g = 72 g.
210
Question 10.
(a) The diagram alongside shows a current carrying loop or a circular coil passing through a
sheet of cardboard at the points M and N. The sheet of cardboard is sprinkled uniformly
with iron filings.

(i)Copy the diagram and draw an arrow on the circular coil to show the direction of
current flowing through it.
(ii) Draw the pattern of arrangement of the iron filings when current is passed through
the loop. [3]
(b) (i) Draw a simplified labelled diagram of a hot cathode ray tube.
(ii) Name a common device where a hot cathode ray tube is used. [3]
(c) A certain nucleus X has a mass number 14 and atomic number 6. The nucleus X changes
to 7Y14 after the loss of a particle.
(i) Name the particle emitted.
Ph 11 2012

(ii) Represent this change in the form of an equation.


(iii) A radioactive substance is oxidized. What change would you expect to take place in
the nature of its radioactivity ? Give a reason for you answer. [4]
Answer.
(a) (i) and (ii)

(b) (i)

(ii) Hot cathode ray tube is used in monitors in computers.


(c) (i) The particle emitted is beta particle.
(ii) 6Y14 +  7Y
14

(iii) No change will take place in its rate of activity. It is because oxidation is a chemical
change which takes at electron level. It has nothing to do with nucleus of the atom.
ICSE EXAMINATION QUESTION PAPER – 2013 (SOLVED)

(Time : 1 1/2 hours) (M.Marks 80)


Answers to this Paper must be written on the paper provided separately.
You will not be allowed to write during the first 15 minutes.
This time is to be spent in reading the Question Paper,
The time given at the head of this Paper is the time allowed for writing the answers.
Section I is compulsory. Attempt any four questions from Section II.
The intended marks for questions or parts of questions are given in brackets [ ].
Section I (40 Marks)
Attempt all questions from this Section.
Question 1
(a) Give any two effects of a force on a non-rigid body.
Fixed
(b) One end of a spring is kept fixed while the other end F end

is stretched by a force as shown in the diagram.


(i) Copy the diagram and mark on it the direction of the restoring force.
(ii) Name one instrument which works on the above principle. (2)
(c) (i) Where is the centre of gravity of a uniform ring situated ?
(ii) ‘The position of the centre of gravity of a body remains unchanged even when the body
is deformed.’ State whether the statement is true or false. (2)
–1
(d) A force is applied on a body of mass 20 kg moving with a velocity of 40 ms . The body
attains a velocity of 50 ms–1 in 2 seconds. Calculate the work done by the body. (2)
(e) A type of single pulley is very often used as a machine even though it does not give any
gain in mechanical advantage.
(i) Name the type of pulley used.
(ii) For what purpose is such a pulley used ? (2)
Answer.
(a) (i) A force can bring about change in the state of rest or uniform motion of a body.
(ii) A force can change the shape of dimensions of a body.
(b) (i) F Restoring force (ii) Spring balance.
F Fixed
end

(c) (i) The centre of gravity of a uniform ring is its centre. (ii) False
  u (50  40) ms 1
(d) Acceleration of body (a) =  = 5 ms–2
t 2s
Distance covered by body while accelerating,
1 1
S = ut + at2 = 40 × 2 + × 5 × 2 × 2 = 80 + 10 = 90 m
2 2
Force possessed by body, F = m.a = 20 (kg) × 5 ms–2 = 100 N
Ph 12 2013
Ph 13 2013

 Work done by the body, W = F × S = 100 N × 90 m = 9000 J


(e) (i) Single fixed pulley. (ii) It helps in changing the direction of applying efforts.
Question 2.
(a) (i) In what way does an ‘Ideal machine’ differ from a ‘Practical machine’ ?
(ii) Can a simple machine act as a force multiplier and a speed multiplier at the same time?(2)
(b) A girl of mass 35 kg climbs up from the first floor of a building at a height 4 m above
the ground to the third floor at a height 12 m above the ground. What will be the increase
in her gravitational potential energy ? (g = 10 ms–2).
(c) Which class of lever found in the human body is being used by a boy -
(i) when he holds a load on the palm of his hand.
(ii) when he raises the weight of his body on his toes ?
(d) A ray of light is moving from a rarer medium
to a denser medium and strikes a plane mirror
placed at 90o to the direction of the ray as
shown in the diagram.

(i) Copy the diagram and mark arrows to show the path of the ray of light after it is reflected
from the mirror.
(ii) Name the principle you have used to mark the arrows to show the direction of the ray.(2)
(e) (i) The refractive index of glass with respect to air is 1.5. What is the value of the
refractive index of air with respect to glass ?
(ii) A ray of light is incident as a normal ray on the surface of separation of two different
mediums. What is the value of the angle of incidence in this case ? (2)
Answer
(a) (i) An ideal machine is the one whose parts are frictionless as well as weightless, such
that its mechanical advantage is equal to its velocity ratio.
A practical machine is the one whose parts are neither frictionless nor weightless. Furthermore,
its mechanical advantage is always less than its velocity ratio.
(ii) No, it can be either speed multiplier or force multiplier.
(b) Mass of girl (m) = 35 kg
Height gained by girl (h) = (12 – 4) = 8 m
 Increase in gravitational potential
energy = mgh = 35 (kg) × 10 ms–2 × 8 m Rarer medium
= 2800 J
Denser medium
(c) (i) Lever of third order.
(ii) Lever of second order
90°
(d) (i)
(ii) The principle is the law of reversibility of light Mirror
Ph 14 2013

g 1 1 2
(e) (i) a  a
   0.67 (Appox.)
 g 1 .5 3
(ii) Angle of incidence is zero.
Question 3
(a) A bucket kept under a running tap is getting filled with water. A person sitting at a distance
is able to get an idea when the bucket is about to be filled.
(i) What change takes place in the sound to give this idea ?
(ii) What causes the change in the sound ? (2)
(b) A sound made on the surface of a lake takes 3 s to reach a boatman.
How much time will it take to reach a diver inside the water at the same depth ?
[Velocity of sound in air = 330 ms–1 ; Velocity of sound in water = 1450 ms–1] (2)
(c) Calculate the equivalent resistance between the points A and B for the following combination
of resistors : (2)

(d) You have been provided with a solenoid AB.

A B

(i) What is the polarity at end A ?


(ii) Give one advantage of an electromagnet over a permanent magnet. (2)
(e) (i) Name the device used to protect the electric circuits from overloading and short
circuits.
(ii) On what effect of electricity does the above device work ? (2)
Answer
(a) (i) The sharp pitched sound slowly changes to low pitched sound as the bucket gets filled.
The sound almost dies when the bucket is completely filled.
(ii) As the length of vibrating air column decreases due to the water, the frequency of the
sound changes.
(b) Distance covered by the sound to reach boatman = 330 ms–1 × 3 s = 990 m
 Distance of diver from the source of sound = 990 m
Ph 15 2013

990 m 990
 Time taken by the sound to reach diver = 1450 ms –1  145 s = 0.68 s (Appox.).

(c) Resistance of three 4  resistors in series = 4 × 3 = 12 


Resistance of three 2  resistors in series = 2 × 3 = 6 
 Equivalent resistance of 12  , 6  and 4  in parallel.
1 1 1 1 1 2  3 6 1
       Rp = 2 
R p 12 6 4 12 12 2

 Equivalent resistance of 5  , 2  and 6  in series.


Rs = (5 + 2 + 6)  = 13 
(d) (i) Polarity at the end A is NORTH.
(ii) An electromagnet’s strength can be increased by increasing the flow of current in the coil,
which is not possible in case of a permanent magnet.
(e) (i) Electric fuse.
(ii) It works on the heating effect of electric current.
Question 4
(a) Define the term ‘Heat capacity’ and state its S.I. unit. (2)
(b) What is meant by Global warming ? (2)
(c) How much heat energy is released when 5 g of water at 20 C changes to ice at 0oC?
o

[Specific heat capacity of water = 4.2 Jg–1 oC–1 ; Specific latent heat of fusion of ice =
336 g–1] (2)
(d) Which of the radioactive radiations -
(i) can cause severe genetical disorders. (ii) are deflected by an electric field ?
(e) A radioactive nucleus undergoes a series of decays according to the sequence
 
X
 X1 
 X2 
 X3.
If the mass number and atomic number of X3 are 172 and 69 respectively, what is the
mass number and atomic number of X ?
Answer
(a) Heat capacity : The amount of heat energy required to raise the temperature of a given
mass of a substance through 1 K (or 1oC) is called its heat capacity.
S.I. unit of heat capacity is JK–1.
(b) The rise in average temperature of the atmosphere around the earth, due to the trapping
of radiant solar heat due to carbon dioxide and chloroflurocarbons is called global warming.
(c) Heat energy released in cooling water to 0oC = mc  f  5  4.2  20  420J
Heat energy released in freezing water = mL = 5 × 336 = 1680 J
 Total heat energy released = (1680 + 420) J = 2100 J
(d) (i) Gamma radiations (ii) Alpha and beta radiations
172 a 176 176 a 180 180  180
(e) 69 X 3  71 X2 ; 71 X 2  73 X1 73 X 1  72 X
Thus, mass number of X is 180 and atomic number 72 ;
Ph 16 2013

SECTION II (40 Marks)


Attempt any four questions from this Section.
Question 5.
(a) (i) With reference to their direction of action, how does a centripetal force differ from
a centrifugal force ?
(ii) State the Principle of conservation of energy.
(iii) Name the form of energy which a body may possess even when it is not in motion. (3)
(b) A coolie is pushing a box weighing 1500 N up an inclined plane 7.5 m long on to a
platform, 2.5 m above the ground.
(i) Calculate the mechanical advantage of the inclined plane.
(ii) Calculate the effort applied by the coolie.
(iii) In actual practice, the coolie needs to apply more effort than what is calculated. Give one
reason why you think the coolie needs to apply more effort. (3)
(c) A block and tackle system of pulley’s a velocity ratio 4.
(i) Draw a labelled diagram of the system indicating clearly the points of application and
directions of a load and effort.
(ii) What is the value of the mechanical advantage of the given pulley system if it is an ideal
pulley system ? (4)
Answer.
(a) (i) With reference to action, the centripetal force is directed towards the centre of circular
path and the centrifugal force is directed away from the centre of circular path, such that
they are acting opposite to one another.
(ii) Law of conservation of energy : Energy in a system cannot be created, nor it can be
destroyed and the sum total of energy remains constant, no matter it can change its form.
(iii) Potential energy.
(b) (i) For a perfect machnine ;
Mechanical advantage = Velocity ratio
Distance through which effort acts 7.5 m
= Distance through which load acts  2.5 m  3

Load 1500 N Block and tackle system


(ii) Mechanical advantage =  3 = with velocity ratio 4
Effort Effort
1500 N
Effort = = 500 N. E
3
(iii) Inclined plane is not a perfect machine. It offers E E E
certain amount of resistance to the load. Thus, in actual
practice coolie has to apply more effort. E
(c) (i) Load = 4T
E= T
 V.R. = 4
Block and tackle system of pulley having V.R. = 4
Load
(ii) Mechanical advantage will be equal to the number
L
of pulleys engaged in a given pulley system.
Ph 17 2013

Question 6
(a) Name the radiations :
(i) that are used for photography at night. (ii) used for detection of fracture in bones.
o o
(ii) whose wavelength range is from 100 A to 4000 A (or 10 nm to 400 nm). (3)
(b) (i) Can the absolute refractive index of a medium be less than one ?
(ii) A coin placed at the bottom of a beaker appears to be raised by 4.0 cm.
If the refractive index of water is 4/3, find the depth of the water in the beaker. (3)
(c) An object AB is placed between 2F1 and F1 on the principal axis of a convex lens as
shown in the diagram.

Copy the diagram and using three rays starting from point A, obtain the image of the
object formed by the lens. (4)
Answer
(a) (i) infrared radiation. (ii) X-rays. (iii) Ultraviolet radiation.
(b) (i) No, absolute refractive index of a medium is always greater than 1, as speed of light
in any medium is always less than that in vacuum.
(ii) Let the real depth of water = x
Apparent depth of water = (x – 4) cm.
Real depth 4 x
Now, refractive index of water = Apparent depth  3  x  4 or (4x – 16) = 3x

 Real depth = x = 16 cm
(c)

Question 7.
(a) (i) What is the principle on which SONAR is based.
(ii) An observer stands at a certain distance away from a cliff and produces a loud sound.
He hears the echo of the sound after 1.8 s. Calculate the distance between the cliff and
the observer if the velocity of sound in air is 340 ms–1.
Ph 18 2013

(b) A vibrating tuning fork is placed over the mouth of a burette filled with water. The tap
of the burette is opened and the water level gradually starts falling. It is found that the
sound from the tuning fork becomes very loud for a particular length of the water column.
(i) Name the phenomenon taking place when this happens.
(ii) Why does the sound become very loud for this length of the water column ? (3)
(c) (i) What is meant by the terms (1) amplitude (2) frequency of a wave ?
(ii) Explain why stringed musical instruments, like the guitar, are provided with a hollow box.
(4)
Answer
(a) (i) Echo depth sounding
Ultrasonic waves have the same speed as of audible sound but are not absorbed in the
medium. So transmitter sends these waves receiver receives the waves back after striking
vt
the rigid obstacle so time taken is recorded. And we can calculate distance d = .
2
Velocity of sound  time 340ms –1  1.8s
(ii) Distance between cliff and source of sound = 
2 2
= 306 m
(b) (i) The phenomenon is called ‘resonance of sound’.
(ii) A some particular length of air column the natural frequency of air column corresponds
the frequency of tuning fork. At this moment the sound waves reinforce to produce loud
sound.
(c) (i) The maximum displacement of vibrating particle about its mean position is called its
amplitude.The number of waves which pass through a point in a medium in one second
is called frequency.
(ii) The air trapped in the hollow box starts vibrating with forced vibrations, thereby producing
lounder sound.
Question 8.
(a) (i) It is observed that the temperature of the surrounding starts falling when the ice in a
frozen lake starts melting. Give a reason for the observation.
(ii) How is the heat capacity of the body related to its specific heat capacity ? (3)
(b) (i) Why does a bottle of soft drink cool faster when surrounded by ice cubes than by ice
cold water, both at 0o C ?
(ii) A certain amount of heat Q will warm 1 g of material X by 3oC and 1 g of material Y
by 4oC. Which material has a higher specific heat capacity.
(c) A calorimeter of mass 50 g and specific heat capacity 0.42 J g–1 oC–1 contains some mass
of water at 20oC. A metal piece of mass 20 g at 100 oC is dropped into the calorimeter.
After stirring, the final temperature of the mixture is found to be 22oC. Find the mass of
water used in the calorimeter.
[specific heat capacity of the metal piece = 0.3 Jg–1 oC–1 ; specific heat capacity of water
= 4.2 Jg–1 oC–1] (4)
Answer
(a) (i) Every kilogram of ice at 0oC on melting to form water at 0oC needs 336 × 103 J of
heat energy as its specific latent heat is 336 × 103J. This heat energy is supplied by the
surrounding of the lake, which in turn results in the fall in temperature.
Ph 19 2013

Heat capacity of the body


(ii) Specific heat capacity of a body = Mass of the body
(b) (i) Every gram of ice surrounding the soft drink extracts out 336 J of heat energy from
it and the temperature of surrounding the soft drink remains at 0oC. However, in case of
cold water, it will extract out only 4.2 J of heat energy per gram. Furthermore, the
temperature of surrounding water starts rising. Thus, soft-drink bottle cools better in case
of ice.
(ii) Material X has higher specific heat capacity compared to material Y.
It is because for the mass amount of heat its temperature rises less than Y.
(c) Data
Substance Mass SHC Initial temperature Final temperature = 22 oC
Calorimeter 50 g 0.42 Jg–1 oC–1 20oC  R = (22 – 20) = 2oC
Cold water ? (x) 0.42 Jg–1 oC–1 20oC  R = (22 – 20) = 2oC
Hot metal piece 20 g 0.3 Jg–1 oC–1 100 oC  F = (100 – 22) = 78oC
Heat absorbed by cold water = mc  R = x × 4.2 × 2 = 8.4x
Heat absorbed by calorimeter = mc  R = 50 × 0.42 × 2 = 42
Total heat absorbed = 8.4x + 42.
Heat given out by metal mc  F = 20 × 0.3 × 78 = 468.
By the principle of calorimetry, Total heat absorbed = total heat lost
426
8.4x + 42 = 468  8.4x = 468 – 42 = 426  x = = 50.7 g
8.4
Question 9.
(a) (i) State Ohm’s law.
(ii) A metal wire of resistance 6  is stretched so that its length is increased to twice its
original length. Calculate its new resistance. (3)
(b) (i) An electrical gadget can give an electric shock to its user under certain circumstances.
Mention any two of these circumstances.
(ii) What preventive measure provided in a gadget can protect a person from an electric
shock ?
(c) The figure shows a circuit

When the circuit is switched on, the ammeter reads 0.5 A.


Ph 20 2013

(i) Calculate the value of the unknown resistor R.


(ii) Calculate the charge passing through the 3  resistor in 120s.
(iii) Calculate the power dissipated in the 3  resistor.. (4)
Answer
(a) (i) Ohm’s Law : It states, all physical conditions of a conductor remaining same, the
current flowing through it is directly proportional to the potential difference at its ends.
(ii) Let the original length be (l) and area of cross-section (a), such that its resistance is 6  .
1 Kl
Applying, R = K  6= ...(i)
a a
a
When the length 2l, its area of cross-section becomes . If R is the new resistance of
2
conductor then :
2l Kl
R = K 4 ....(ii)
a/2 a
R
Dividing (ii) by (i) =4  R = 24 
6
(b) (i) (1) The electrical gadget may be short circuited i.e., its live or neutral wire is touching
its metallic body directly.
(2) The hands of the user may be wet, such that water dripping from his hands makes contact
with the live wire.
(ii) The body of the electric gadget is connected to the earth terminal by means of earth wire.
In case of short circuit a huge surge of current flows through the earth terminal. This in
turn melts fuse in the live wire and hence the flow or current stops in the gadget.
V 6
(c) (i) I =  0.5 =
R R 3
 0.5 R + 1.5 = 6  0.5 R = 4.5  R = 9 
(ii) Charge Q = I × t = 0.5 × 120 = 60 Coulombs
(iii) Power dissipated in 3  resistor = I2R = 0.5 × 0.5 × 3 = 0.75 W
Question 10.
(a) Name the three main parts of a Cathode Ray Tube.
(b) (i) What is meant by Radioactivity ? (3)
(ii) What is meant by nuclear waste ? (3)
(iii) Suggest one effective way for the safe disposal of nuclear waste.
(c) (i) Draw a simple labelled diagram of d.c. electric motor.
(ii) What is the function of the split rings in a d.c. motor ?
(iii) State one advantage of a.c. over d.c. (4)
Answer
(a) (i) (1) electron gun (2) Deflecting plates (3) Fluorescent screen
(b) (i) The phenomenon due to the nucleus of certain elements decays on its own, giving out
Ph 21 2013

harmful radiations, such as a alpha particles, beta particles and gamma radiations is called
radioactivity.
(ii) The residual material left in the nuclear reactors after generating heat energy is called
nuclear waste. The nuclear waste is radioactive and very harmful to the environment.
(iii) The nuclear waste should be stored in stainless steel containers, lined from within with
thick sheets of lead, so that no radioactive rays come out of it. The containers should be
stored in safe and well guarded place, so that they do not fall in the hands of criminal
elements.
(c) (i)

(ii) The split rings alter the direction of current in the coil after every half rotation. This in
turn helps the coil to move in the same direction, i.e., clockwise or anticlockwise direction.
(iii) The alternating current can be easily stepped up or down and can be transmitted over long
distance cable wires. This is not possible in case of direct current.
ICSE EXAMINATION QUESTION PAPER – 2013 (SOLVED)

(Time : 1 1/2 hours) (M.Marks 80)


Answers to this Paper must be written on the paper provided separately.
You will not be allowed to write during the first 15 minutes.
This time is to be spent in reading the Question Paper,
The time given at the head of this Paper is the time allowed for writing the answers.
Section I is compulsory. Attempt any four questions from Section II.
The intended marks for questions or parts of questions are given in brackets [ ].
Section I (40 Marks)
F Fixed
Attempt all questions from this Section. end
Question 1
(a) Give any two effects of a force on a non-rigid body.
(b) One end of a spring is kept fixed while the other end
is stretched by a force as shown in the diagram.
(i) Copy the diagram and mark on it the direction of the restoring force.
(ii) Name one instrument which works on the above principle. (2)
(c) (i) Where is the centre of gravity of a uniform ring situated ?
(ii) ‘The position of the centre of gravity of a body remains unchanged even when the body
is deformed.’ State whether the statement is true or false. (2)
–1
(d) A force is applied on a body of mass 20 kg moving with a velocity of 40 ms . The body
attains a velocity of 50 ms–1 in 2 seconds. Calculate the work done by the body. (2)
(e) A type of single pulley is very often used as a machine even though it does not give any
gain in mechanical advantage.
(i) Name the type of pulley used.
(ii) For what purpose is such a pulley used ? (2)
Answer.
F Restoring force
(a) (i) FA force can bring about change in Fixed
the state of rest or uniform motion of a body.
end
(ii) A force can change the shape of dimensions of a body.
(b) (i) (ii) Spring balance.

(c) (i) The centre of gravity of a uniform ring is its centre. (ii) False
  u (50  40) ms 1
(d) Acceleration of body (a) =  = 5 ms–2
t 2s
Distance covered by body while accelerating,

Ph 1 1
S =12ut + at2 = 40 × 2 + × 5 × 2 × 2 = 80 + 10 = 90 m 2013
2 2
Force possessed by body, F = m.a = 20 (kg) × 5 ms–2 = 100 N
Ph 13 2013

 Work done by the body, W = F × S = 100 N × 90 m = 9000 J


(e) (i) Single fixed pulley. (ii) It helps in changing the direction of applying efforts.
Question 2.
(a) (i) In what way does an ‘Ideal machine’ differ from a ‘Practical machine’ ?
(ii) Can a simple machine act as a force multiplier and a speed multiplier at the same time?(2)
(b) A girl of mass 35 kg climbs up from the first floor of a building at a height 4 m above
the ground to the third floor at a height 12 m above the ground. What will be the increase
in her gravitational potential energy ? (g = 10 ms–2).
(c) Which class of lever found in the human body is being used by a boy -
(i) when he holds a load on the palm of his hand.
(ii) when he raises the weight of his body on his toes ?
(d) A ray of light is moving from a rarer medium
to a denser medium and strikes a plane mirror
placed at 90o to the direction of the ray as
shown in the diagram.

(i) Copy the diagram and mark arrows to show the path of the ray of light after it is reflected
from the mirror.
(ii) Name the principle you have used to mark the arrows to show the direction of the ray.(2)
(e) (i) The refractive index of glass with respect to air is 1.5. What is the value of the
refractive index of air with respect to glass ?
(ii) A ray of light is incident as a normal ray on the surface of separation of two different
mediums. What is the value of the angle of incidence in this case ? (2)
Answer
(a) (i) An ideal machine is the one whose parts are frictionless as well as weightless, such
that its mechanical advantage is equal to its velocity ratio.
A practical machine is the one whose parts are neither frictionless nor weightless. Furthermore,
its mechanical advantage is always less than its velocity ratio.
(ii) No, it can be either speed multiplier or force multiplier.
(b) Mass of girl (m) = 35 kg
Height gained by girl (h) = (12 – 4) = 8 m
 Increase in gravitational potential
energy = mgh = 35 (kg) × 10 ms–2 × 8 m Rarer medium
= 2800 J
Denser medium
(c) (i) Lever of third order.
(ii) Lever of second order
90°
(d) (i)
(ii) The principle is the law of reversibility of light Mirror
Ph 14 2013

g 1 1 2
(e) (i) a  a
   0.67 (Appox.)
 g 1 .5 3
(ii) Angle of incidence is zero.
Question 3
(a) A bucket kept under a running tap is getting filled with water. A person sitting at a distance
is able to get an idea when the bucket is about to be filled.
(i) What change takes place in the sound to give this idea ?
(ii) What causes the change in the sound ? (2)
(b) A sound made on the surface of a lake takes 3 s to reach a boatman.
How much time will it take to reach a diver inside the water at the same depth ?
[Velocity of sound in air = 330 ms–1 ; Velocity of sound in water = 1450 ms–1] (2)
(c) Calculate the equivalent resistance between the points A and B for the following combination
of resistors : (2)

(d) You have been provided with a solenoid AB.

A B

(i) What is the polarity at end A ?


(ii) Give one advantage of an electromagnet over a permanent magnet. (2)
(e) (i) Name the device used to protect the electric circuits from overloading and short
circuits.
(ii) On what effect of electricity does the above device work ? (2)
Answer
(a) (i) The sharp pitched sound slowly changes to low pitched sound as the bucket gets filled.
The sound almost dies when the bucket is completely filled.
(ii) As the length of vibrating air column decreases due to the water, the frequency of the
sound changes.
(b) Distance covered by the sound to reach boatman = 330 ms–1 × 3 s = 990 m
 Distance of diver from the source of sound = 990 m
Ph 15 2013

990 m 990
 Time taken by the sound to reach diver = 1450 ms –1  145 s = 0.68 s (Appox.).

(c) Resistance of three 4  resistors in series = 4 × 3 = 12 


Resistance of three 2  resistors in series = 2 × 3 = 6 
 Equivalent resistance of 12  , 6  and 4  in parallel.
1 1 1 1 1 2  3 6 1
       Rp = 2 
R p 12 6 4 12 12 2

 Equivalent resistance of 5  , 2  and 6  in series.


Rs = (5 + 2 + 6)  = 13 
(d) (i) Polarity at the end A is NORTH.
(ii) An electromagnet’s strength can be increased by increasing the flow of current in the coil,
which is not possible in case of a permanent magnet.
(e) (i) Electric fuse.
(ii) It works on the heating effect of electric current.
Question 4
(a) Define the term ‘Heat capacity’ and state its S.I. unit. (2)
(b) What is meant by Global warming ? (2)
(c) How much heat energy is released when 5 g of water at 20 C changes to ice at 0oC?
o

[Specific heat capacity of water = 4.2 Jg–1 oC–1 ; Specific latent heat of fusion of ice =
336 g–1] (2)
(d) Which of the radioactive radiations -
(i) can cause severe genetical disorders. (ii) are deflected by an electric field ?
(e) A radioactive nucleus undergoes a series of decays according to the sequence
 
X
 X1 
 X2 
 X3.
If the mass number and atomic number of X3 are 172 and 69 respectively, what is the
mass number and atomic number of X ?
Answer
(a) Heat capacity : The amount of heat energy required to raise the temperature of a given
mass of a substance through 1 K (or 1oC) is called its heat capacity.
S.I. unit of heat capacity is JK–1.
(b) The rise in average temperature of the atmosphere around the earth, due to the trapping
of radiant solar heat due to carbon dioxide and chloroflurocarbons is called global warming.
(c) Heat energy released in cooling water to 0oC = mc  f  5  4.2  20  420J
Heat energy released in freezing water = mL = 5 × 336 = 1680 J
 Total heat energy released = (1680 + 420) J = 2100 J
(d) (i) Gamma radiations (ii) Alpha and beta radiations
172 a 176 176 a 180 180  180
(e) 69 X 3  71 X2 ; 71 X 2  73 X1 73 X 1  72 X
Thus, mass number of X is 180 and atomic number 72 ;
Ph 16 2013

SECTION II (40 Marks)


Attempt any four questions from this Section.
Question 5.
(a) (i) With reference to their direction of action, how does a centripetal force differ from
a centrifugal force ?
(ii) State the Principle of conservation of energy.
(iii) Name the form of energy which a body may possess even when it is not in motion. (3)
(b) A coolie is pushing a box weighing 1500 N up an inclined plane 7.5 m long on to a
platform, 2.5 m above the ground.
(i) Calculate the mechanical advantage of the inclined plane.
(ii) Calculate the effort applied by the coolie.
(iii) In actual practice, the coolie needs to apply more effort than what is calculated. Give one
reason why you think the coolie needs to apply more effort. (3)
(c) A block and tackle system of pulley’s a velocity ratio 4.
(i) Draw a labelled diagram of the system indicating clearly the points of application and
directions of a load and effort.
(ii) What is the value of the mechanical advantage of the given pulley system if it is an ideal
pulley system ? (4)
Answer.
(a) (i) With reference to action, the centripetal force is directed towards the centre of circular
path and the centrifugal force is directed away from the centre of circular path, such that
they are acting opposite to one another.
(ii) Law of conservation of energy : Energy in a system cannot be created, nor it can be
destroyed and the sum total of energy remains constant, no matter it can change its form.
(iii) Potential energy.
(b) (i) For a perfect machnine ;
Mechanical advantage = Velocity ratio
Distance through which effort acts 7.5 m
= Distance through which load acts  2.5 m  3

Load 1500 N Block and tackle system


(ii) Mechanical advantage =  3 = with velocity ratio 4
Effort Effort
1500 N
Effort = = 500 N. E
3
(iii) Inclined plane is not a perfect machine. It offers E E E
certain amount of resistance to the load. Thus, in actual
practice coolie has to apply more effort. E
(c) (i) Load = 4T
E= T
 V.R. = 4
Block and tackle system of pulley having V.R. = 4
Load
(ii) Mechanical advantage will be equal to the number
L
of pulleys engaged in a given pulley system.
Ph 17 2013

Question 6
(a) Name the radiations :
(i) that are used for photography at night. (ii) used for detection of fracture in bones.
o o
(ii) whose wavelength range is from 100 A to 4000 A (or 10 nm to 400 nm). (3)
(b) (i) Can the absolute refractive index of a medium be less than one ?
(ii) A coin placed at the bottom of a beaker appears to be raised by 4.0 cm.
If the refractive index of water is 4/3, find the depth of the water in the beaker. (3)
(c) An object AB is placed between 2F1 and F1 on the principal axis of a convex lens as
shown in the diagram.

Copy the diagram and using three rays starting from point A, obtain the image of the
object formed by the lens. (4)
Answer
(a) (i) infrared radiation. (ii) X-rays. (iii) Ultraviolet radiation.
(b) (i) No, absolute refractive index of a medium is always greater than 1, as speed of light
in any medium is always less than that in vacuum.
(ii) Let the real depth of water = x
Apparent depth of water = (x – 4) cm.
Real depth 4 x
Now, refractive index of water = Apparent depth  3  x  4 or (4x – 16) = 3x

 Real depth = x = 16 cm
(c)

Question 7.
(a) (i) What is the principle on which SONAR is based.
(ii) An observer stands at a certain distance away from a cliff and produces a loud sound.
He hears the echo of the sound after 1.8 s. Calculate the distance between the cliff and
the observer if the velocity of sound in air is 340 ms–1.
Ph 18 2013

(b) A vibrating tuning fork is placed over the mouth of a burette filled with water. The tap
of the burette is opened and the water level gradually starts falling. It is found that the
sound from the tuning fork becomes very loud for a particular length of the water column.
(i) Name the phenomenon taking place when this happens.
(ii) Why does the sound become very loud for this length of the water column ? (3)
(c) (i) What is meant by the terms (1) amplitude (2) frequency of a wave ?
(ii) Explain why stringed musical instruments, like the guitar, are provided with a hollow box.
(4)
Answer
(a) (i) Echo depth sounding
Ultrasonic waves have the same speed as of audible sound but are not absorbed in the
medium. So transmitter sends these waves receiver receives the waves back after striking
vt
the rigid obstacle so time taken is recorded. And we can calculate distance d = .
2
Velocity of sound  time 340ms –1  1.8s
(ii) Distance between cliff and source of sound = 
2 2
= 306 m
(b) (i) The phenomenon is called ‘resonance of sound’.
(ii) A some particular length of air column the natural frequency of air column corresponds
the frequency of tuning fork. At this moment the sound waves reinforce to produce loud
sound.
(c) (i) The maximum displacement of vibrating particle about its mean position is called its
amplitude.The number of waves which pass through a point in a medium in one second
is called frequency.
(ii) The air trapped in the hollow box starts vibrating with forced vibrations, thereby producing
lounder sound.
Question 8.
(a) (i) It is observed that the temperature of the surrounding starts falling when the ice in a
frozen lake starts melting. Give a reason for the observation.
(ii) How is the heat capacity of the body related to its specific heat capacity ? (3)
(b) (i) Why does a bottle of soft drink cool faster when surrounded by ice cubes than by ice
cold water, both at 0o C ?
(ii) A certain amount of heat Q will warm 1 g of material X by 3oC and 1 g of material Y
by 4oC. Which material has a higher specific heat capacity.
(c) A calorimeter of mass 50 g and specific heat capacity 0.42 J g–1 oC–1 contains some mass
of water at 20oC. A metal piece of mass 20 g at 100 oC is dropped into the calorimeter.
After stirring, the final temperature of the mixture is found to be 22oC. Find the mass of
water used in the calorimeter.
[specific heat capacity of the metal piece = 0.3 Jg–1 oC–1 ; specific heat capacity of water
= 4.2 Jg–1 oC–1] (4)
Answer
(a) (i) Every kilogram of ice at 0oC on melting to form water at 0oC needs 336 × 103 J of
heat energy as its specific latent heat is 336 × 103J. This heat energy is supplied by the
surrounding of the lake, which in turn results in the fall in temperature.
Ph 19 2013

Heat capacity of the body


(ii) Specific heat capacity of a body = Mass of the body
(b) (i) Every gram of ice surrounding the soft drink extracts out 336 J of heat energy from
it and the temperature of surrounding the soft drink remains at 0oC. However, in case of
cold water, it will extract out only 4.2 J of heat energy per gram. Furthermore, the
temperature of surrounding water starts rising. Thus, soft-drink bottle cools better in case
of ice.
(ii) Material X has higher specific heat capacity compared to material Y.
It is because for the mass amount of heat its temperature rises less than Y.
(c) Data
Substance Mass SHC Initial temperature Final temperature = 22 oC
Calorimeter 50 g 0.42 Jg–1 oC–1 20oC  R = (22 – 20) = 2oC
Cold water ? (x) 0.42 Jg–1 oC–1 20oC  R = (22 – 20) = 2oC
Hot metal piece 20 g 0.3 Jg–1 oC–1 100 oC  F = (100 – 22) = 78oC
Heat absorbed by cold water = mc  R = x × 4.2 × 2 = 8.4x
Heat absorbed by calorimeter = mc  R = 50 × 0.42 × 2 = 42
Total heat absorbed = 8.4x + 42.
Heat given out by metal mc  F = 20 × 0.3 × 78 = 468.
By the principle of calorimetry, Total heat absorbed = total heat lost
426
8.4x + 42 = 468  8.4x = 468 – 42 = 426  x = = 50.7 g
8.4
Question 9.
(a) (i) State Ohm’s law.
(ii) A metal wire of resistance 6  is stretched so that its length is increased to twice its
original length. Calculate its new resistance. (3)
(b) (i) An electrical gadget can give an electric shock to its user under certain circumstances.
Mention any two of these circumstances.
(ii) What preventive measure provided in a gadget can protect a person from an electric
shock ?
(c) The figure shows a circuit

When the circuit is switched on, the ammeter reads 0.5 A.


Ph 20 2013

(i) Calculate the value of the unknown resistor R.


(ii) Calculate the charge passing through the 3  resistor in 120s.
(iii) Calculate the power dissipated in the 3  resistor.. (4)
Answer
(a) (i) Ohm’s Law : It states, all physical conditions of a conductor remaining same, the
current flowing through it is directly proportional to the potential difference at its ends.
(ii) Let the original length be (l) and area of cross-section (a), such that its resistance is 6  .
1 Kl
Applying, R = K  6= ...(i)
a a
a
When the length 2l, its area of cross-section becomes . If R is the new resistance of
2
conductor then :
2l Kl
R = K 4 ....(ii)
a/2 a
R
Dividing (ii) by (i) =4  R = 24 
6
(b) (i) (1) The electrical gadget may be short circuited i.e., its live or neutral wire is touching
its metallic body directly.
(2) The hands of the user may be wet, such that water dripping from his hands makes contact
with the live wire.
(ii) The body of the electric gadget is connected to the earth terminal by means of earth wire.
In case of short circuit a huge surge of current flows through the earth terminal. This in
turn melts fuse in the live wire and hence the flow or current stops in the gadget.
V 6
(c) (i) I =  0.5 =
R R 3
 0.5 R + 1.5 = 6  0.5 R = 4.5  R = 9 
(ii) Charge Q = I × t = 0.5 × 120 = 60 Coulombs
(iii) Power dissipated in 3  resistor = I2R = 0.5 × 0.5 × 3 = 0.75 W
Question 10.
(a) Name the three main parts of a Cathode Ray Tube.
(b) (i) What is meant by Radioactivity ? (3)
(ii) What is meant by nuclear waste ? (3)
(iii) Suggest one effective way for the safe disposal of nuclear waste.
(c) (i) Draw a simple labelled diagram of d.c. electric motor.
(ii) What is the function of the split rings in a d.c. motor ?
(iii) State one advantage of a.c. over d.c. (4)
Answer
(a) (i) (1) electron gun (2) Deflecting plates (3) Fluorescent screen
(b) (i) The phenomenon due to the nucleus of certain elements decays on its own, giving out
Ph 21 2013

harmful radiations, such as a alpha particles, beta particles and gamma radiations is called
radioactivity.
(ii) The residual material left in the nuclear reactors after generating heat energy is called
nuclear waste. The nuclear waste is radioactive and very harmful to the environment.
(iii) The nuclear waste should be stored in stainless steel containers, lined from within with
thick sheets of lead, so that no radioactive rays come out of it. The containers should be
stored in safe and well guarded place, so that they do not fall in the hands of criminal
elements.
(c) (i)

(ii) The split rings alter the direction of current in the coil after every half rotation. This in
turn helps the coil to move in the same direction, i.e., clockwise or anticlockwise direction.
(iii) The alternating current can be easily stepped up or down and can be transmitted over long
distance cable wires. This is not possible in case of direct current.
ICSE EXAMINATION QUESTION PAPER – 2014 (SOLVED)
Time : 2 hours 80 Marks
Answers to this paper must be written on the paper provided separately.
You will not be allowed to write during the first 15 minutes.
This time is to be spent in reading the Question Paper.
The time given at the head of this paper is the time allowed for writing the answers.
Section I is compulsory. Attempt any four questions from Section II.
The intended marks for questions or parts of questions are given in brackets [ ].

SECTION – I (40 Marks)


(Attempt all questions from this section)
Question 1.
(a) A force is applied on (i) a non-rigid body and (ii) a rigid body. How does the effect of the
force differ in the above two cases? [2]
(b) A metallic ball is hanging by a string from a fixed support. Draw a neat labelled diagram
showing the forces acting on the ball and the string. [2]
(c) (i) What is the weight of a body placed at the centre of the earth?
(ii) What is the principle of an ideal machine? [2]
(d) Is it possible to have an accelerated motion with a constant speed? Explain. [2]
(e) (i) When does a force do work?
(ii) What is the work done by the moon when it revolves around the earth? [2]
Answer.
(a) (i) When a force applied on a non-rigid body, it changes the interspacing between its
constituent particles and therefore causes a change in its dimensions and can also
produce motion in it.
(ii) When a force is applied on a rigid body, it does not change the interspacing between
its constituent particles and therefore it does not change the dimensions of the object
but causes only the motion in it.
Support
(b)
T
T  Tension in the string
mg  Weight of the ball

mg
(c) (i) The weight of a body placed at the centre of the earth is zero because value of g = 0
at the center of the earth.
(ii) An ideal machine is that in which there is no dissipation of energy in any manner.
 Principle of an ideal machine = work done by a machine = work done on the machine /
work output = work input
Ph 22 2014
Ph 23 2014

(d) Yes. The velocity of particle in circular motion is variable or the circular motion is accelerated
even though the speed of particle is uniform.
(e) (i) Work is said to be done only when the force applied on a body makes the body moves
i.e., there is displacement of the body.
(ii) The work done by the moon when it revolves around the earth is zero.
Question 2.
(a) Calculate the change in the Kinetic energy of a moving body if its velocity is reduced to
1/3rd of the initial velocity. [2]
(b) State the energy changes in the following devices while in use :
(i) A loud speaker.
(ii) A glowing electric bulb. [2]
(c) (i) What is nuclear energy?
(ii) Name the process used for producing electricity using nuclear energy. [2]
(d) State one important advantage and disadvantage each of using nuclear energy for producing
electricity. [2]
(e) (i) The conversion of part of the energy into an undesirable form is called ............. .
(ii) For a given height h, ............. the length 1 of the inclined plane, lesser will be the
effort required. [2]
Answer.

1
(a) Initial K.E = mv2
2 1 1
1
Change in velocity = of initial velocity
3
2
1 1 v
 Final K.E. = K.E2 = mv2 = × m  
2 2 3

1  v2 
= m  9 
2  

1 1 2
=  mv 
9 2 
1
= th of initial K.E.
9
(b) (i) Electrical energy converted to sound energy.
(ii) Electrical energy converted to heat energy and to light energy.
(c) (i) In nuclear fission and fusion there is a loss in mass due to loss in mass energy
released. The energy so released is called the nuclear energy.
Ph 24 2014

(ii) Nuclear fission.


(d) Advantage : A very small amount of nuclear fuel can produce a tremendous amount of
energy.
Disadvantage : Very harmful nuclear radiations are produced in the process which are
highly energetic and penetrating.
(e) (i) Dissipation of energy. (ii) More.
Question 3.
(a) Draw the diagram given below and clearly show the path taken by the emergent ray. [2]

45°

(b) (i) What is consumed using different electrical appliances, for which electricity bills are
paid?
(ii) Name a common device that uses electromagnets. [2]
(c) (i) A ray of light passes from water to air. How does the speed of light change?
(ii) Which colour of light travels fastest in any medium except air? [2]
(d) Name the factors affecting the critical angle for the pair of media. [2]
(e) (i) Name a prism required for obtaining a spectrum of Ultraviolet light.
(ii) Name the radiations which can be detected by a thermopile. [2]
Answer.
(a)

45° Q

45°

 Correct path of the refracted ray shown


 Correct path of emergent ray shown
(b) (i) Electrical energy. (ii) Electrical bell (Door Bell) / microphone / Electric motor
(c) (i) Speed of light increases.
(ii) Red colour of light travels fastest in any medium except air.
Ph 25 2014

(d) Factors affecting the critical angle for the pair of media are :
(i) Wavelength of light
(ii) Effect of temperature of the medium
(iii) Refractive index of the pair of media.
(e) (i) Quartz prism.
(ii) Infrared radiations can be detected by thermopile.
Question 4.
(a) Why is the colour red used as a sign of danger? [2]
(b) (i) What are mechanical waves?
(ii) Name one property of waves that do no change when the wave passes from one
medium to another. [2]
(c) Find the equivalent resistance between points A and B. [2]

3 4

3 5
A B
6
3

(d) 50 g of metal piece at 27 ºC requires 2400 J of heat energy so as to attain a temperature of


327 ºC. Calculate the specific heat capacity of the metal. [2]
(e) An electron emitter must have ............. work function and ............. melting point. [2]
Answer.
(a) Since the wavelength of red light is maximum in the visible light, the light of red colour is
scattered least by the air molecules of the atmosphere and therefore it can reach to a
longer distance and can be seen easily from farthest distance.
(b) (i) The waves which require a medium for their propagation are called elastic or
mechanical waves. These waves travel in the medium though the vibrations of the medium
particles about their mean positions.
(ii) Frequency.

1 1 1 1 3
(c) R = + + = =1
1 3 3 3 3
R1 = 1
R2 = 5

1 1 1 3 2 5
= + = =
R3 4 6 12 12
Ph 26 2014

12
R3 = = 2.4
5

Total R = R1 + R2 + R3 = 1 + 5 + 2.4 = 8.4.


(d) Given : m = 50g; t = 327ºC – 27ºC = 300ºC, Q = 2400 J.
Now, Q = mC t
2400 = 50 × C × 300

2400
C= = 0.16 J/g ºC
50  300
(e) An electron emitter must have low work function and high melting point.

SECTION – II (40 Marks)


(Attempt any four questions from this Section)
Question 5.
(a) (i) A man having a box on his head, climbs up a slope and another man having an
identical box walks the same distance on a levelled road. Who does more work against
the force of gravity and why?
(ii) Two forces each of 5N act vertically upwards and downwards respectively on the
two ends of a uniform metre rule which is placed at its mid-point as shown in the
diagram. Determine the magnitude of the resultant moment of these forces about the
midpoint. [4]

5N–

50 cm

5N

(b) (i) A body is thrown vertically upwards. Its velocity keeps on decreasing. What happens
to its kinetic energy as its velocity becomes zero?
(ii) Draw a diagram to show how a single pulley can be used so as to have its ideal M.A.
= 2. [3]
(c) Derive a relationship between mechanical advantages, velocity ratio and efficiency of a
machine. [3]
Answer.
(a) (i) Man climbs up a slope does more work against the force of graving whereas the
work done by the man walking on a levelled road is zero because displacement is
normal to the force of gravity.
Ph 27 2014

(ii) F1 = F2 = 5N, d1 = d2 = 50 cm = 0.5 m


Torque  = F × d
1 = F1 × d1 = = 5N × 0.5 m
= 2.5 N m (Anti-clockwise)
2 = F2 × d2 = 5N × 0.5 m
= 2.5 N m (Anti-clockwise)
The resultant moment of force = 1 + 2
= 2.5 + 2.5 = 5N m in Anti clockwise direction.
(b) (i) When a body thrown vertically upward its velocity continuously decreases, kinetic energy
also decrease and potential energy increases due to increase in height.
 K.E. completely changes to P.E.
(ii) Single Movable Pulley. Rigid Support

T E

T=E

Axle

Frame
Single movable Pulley
L

(c) Let a machine overcome a load L by the application of an effort E. In time t, let the displacement
of effort be dE and the displacement of load be dL.
Work input = Effort × displacement of effort
= E × dE
Work output = Load × displacement of load
= L × dL

Work output
Efficiency =
Work input

L  dL L dL
= = ×
E  dE E dE

L 1
= ×
E dE / dL

L
But = M.A.
E
Ph 28 2014

dE
and d = V.R.
L

M.A.
 Efficiency  = or M.A. = V.R. × 
V.R.
Thus, the mechanical advantage of a machine is equal to the product of its efficiency and
velocity ratio.
Question 6.
(a) (i) Light passes through a rectangular glass slab and through a triangular glass prism. In
what way does the direction of the two emergent beams differ and why?
(ii) Ranbir claims to have obtained an image twice the size of the object with a concave
lens. Is he correct? Give a reason for your answer. [4]
(b) A lens forms an erect, magnified and virtual image of an object.
(i) Name the lens.
(ii) Draw a labelled ray diagram to show the image formation. [3]
(c) (i) Define the power of a lens.
(ii) The lens mentioned in 6(b) above is of focal length 25 cm. Calculate the power of the
lens. [3]
Answer.
(a) (i) In a rectangular glass slab, the emergent ray is parallel to the incident ray but they are
not along the same line whereas in a prism the emergent ray is not parallel to incident
ray. Because in a glass block the two surfaces at which refraction occurs is parallel
to each other.
(ii) No. By the concave lens only diminished images are obtained.
(b) (i) Convex lens.
(ii) Object drawn b/w the optical center and the principle focus.
B’

B
2F2
2F1 A’ F1 A O F2

Convex lens

(c) (i) The power of a lens is a measure of deviation produced by it in the path of rays
refracted through it.
Ph 29 2014

1
(ii) Power of lens (in D) = Focal length (in metre)

100
= Focal length (in centimetre)

100
= = 4D.
25
Question 7.
(a) The adjacent diagram shows three different modes of vibrations P, Q and R of the same
string.
(i) Which vibrations will produce a louder sound and why?
(ii) The sound of which string will have maximum shrillness?
(iii) State the ratio of wavelengths of P and R. [4]

(b) A type of electromagnetic wave has wavelength 50 A .


(i) Name the wave.
(ii) What is the speed of the wave in vacuum?
(iii) State one use of this type of wave. [3]
(c) (i) State one important property of waves used for echo depth sounding.
(ii) A radar sends a signal to an aircraft at a distance of 30 km away and receives it back
after 2 × 10–4 second. What is the speed of the signal? [4]
Answer.
(a) (i) R will produce louder sound because its amplitude is more than P and Q.
(ii) The sound of P string will have maximum shrillness.
(iii) The ratio of wavelength of P and R is 1 : 3.
(b) (i) X-Rays. (ii) 3 × 108 m/s.
(iii) X-Rays are used for detection of fracture in bones.
Ph 30 2014

(c) (i) The ultrasonic waves travel undeviated through long distances and they can be confined
to a narrow beam.
(ii) Distance of aircraft = 30 km = 30 × 1000 m = 30,000 m
Total distance = 2 × 30,000 = 60,000 m
Time taken = 2 × 10–4 second
distance travelled (2d )
Hence, Speed of sound V = time taken (t )

60,000
= = 30,000 × 104 m/s
2  10  4
= 3 × 108 m/s.
Question 8.
(a) Two resistors of 4 and 6 are connected in parallel to a cell to draw 0.5 A current from
the cell.
(i) Draw a labelled circuit diagram showing the above arrangement.
(ii) Calculate the current in each resistor. [3]
(b) (i) What is an Ohmic resistor?
(ii) Two copper wires are of the same length, but one is thicker than the other.
(1) Which wire will have more resistance?
(2) Which wire will have more specific resistance? [3]
(c) (i) Two sets A and B, of the three bulbs each, are glowing in two separate rooms. When
one of the bulbs in set A is fused, the other two bulbs, alos cease to glow. But in set
B, when one bulb fuses, the other two bulbs continue to glow. Explain why this
phenomenon occurs.
(ii) Why do we prefer arrangement of Set B for house circuiting? [3]
Answer.
(a) (i)

4

6
0.5A
r = 0.6 

1.5 V

(ii) Let current through 4 resistance is i then current through 6 resistance is (0.5 – i)
 i × 4 = (0.5 – i) × 6
4i = 3 – 6i
Ph 31 2014

4i + 6i = 3
10i = 3
i = 0.3 A
 Current through 4 resistance = 0.3 A
and current through 6 resistance = 0.5 – 0.3 = 0.2 A
(b) (i) Ohmic Resistors : The conductors which obey Ohm’s law are called the Ohmic resistors
or linear resistances. For such resistors, a graph plotted for the potential difference V
against current I is a straight line.
(ii) (1) Thin wire will have more resistance because the resistance is inversely proportional
to area of cross-section.
(2) Specific resistance of both wire is same because specific resistance depends upon
the nature of the medium which is the same (copper) in both cases.
(c) (i) The bulbs of set A are connected in series. Therefore when one bulb fuse the current stop
flowing. Whereas the bulbs of set B are connected in parallel. When one bulb fuse then the
current flows through the other bulb.
(ii) Set B prefer parallel combination because in it potential difference remains same.
Question 9.
(a) Heat energy is supplied at a constant rate to 100g of ice at 0 ºC. The ice is converted into
water at 0 ºC in 2 minutes. How much time will be required to raise the temperature of
water from 0 ºC to 20 ºC? [Given : sp. heat capacity of water = 4.2 J g–1 ºC–1, sp. latent
heat of ice = 336 J g–1]. [4]
(b) Specific heat capacity of substance A is 3.8 J g–1 K–1 whereas the Specific heat capacity
of substance B is 0.4 J g–1 K–1.
(i) Which of the two is a good conductor of heat?
(ii) How is one led to the above conclusion?
(iii) If substances A and B are liquids then which one would be more useful in car radiators?
[3]
(c) (i) State any two measures to minimize the impact of global warming.
(ii) What is the Greenhouse effect? [3]
Answer.
(a) m = 100g, t = 2 minutes = 2 × 60 sec.
Heat energy taken by ice at 0ºC to convert to water at 0ºC.
Q = mL = 100 × 336 = 33600 J
Q 33600
 P= = = 280 J/S
t 2  60
The heat energy required to convert water at 0ºC to 20ºC.
Q = mC t
= 100 × 4.2 × 20 = 8400 J
Q=P×t
Ph 32 2014

8400 = 280 × t
8400
t=
280
t = 30 sec. = 0.5 min.
(b) (i) Substance B is a good conductor of heat.
(ii) Because specific capacity of B is less then that of A. [lower the specific heat capacity,
better the conducting power]
(iii) Substance A is more useful in car radiator.
(c) (i) Two measures to minimize the impact of global warming are :
(1) Technological measures
(2) Economic measures
(ii) The green house effect is the phenomenon in which infrared radiations of long wavelength
given out from the surface of the earth are absorbed by its atmospheric gases to keep the
environment warm at the earth’s surface and the lower atmosphere.
Question 10.
(a) (i) Name two factors on which the magnitude of an induced e.m.f. in the secondary coil
depends.
(ii) In the following diagram an arrow shows the motion of the coil towards the bar
magnet.
(1) State in which direction the current flows, A to B or B to A?
(2) Name the law used to come to the conclusion.

Magnet
S N

A B

(b) A nucleus 11Na24 emits a beta particle to change into Magnesium (Mg)
(i) Write the symbolic equation for the process.
(ii) What are numbers 24 and 11 called?
24 24
(iii) What is the general name of 12 Mg
with respect to 11 Na
? [3]
(c) In a cathode ray tube state :
(i) The purpose of covering cathode by thorium and carbon.
(ii) The purpose of the fluorescent screen.
(iii) How is it possible to increase the rate of emission of electrons. [3]
Answer.
(a) (i) The magnitude of induced e.m.f. in the secondary coil depends on the following two
factors :
Ph 33 2014

(1) The change in the magnetic flux, and


(2) The time in which the magnetic flux changes.
(ii) (1) A to B (2) Lenz’s law.
(b) (i) 11NA 24 24 0
12 Mg + –1e
(ii) The number 24 is atomic mass number (number of protons and neutrons) and 11 is
atomic number.
(iii) They are isobars.
(c) (i) The purpose of covering cathode by thorium and carbon is to decrease the work
function or to emit electrons at lower temperature.
(ii) To convert electrical signal into a visual signal.
(iii) By increasing the temperature of cathode or By increasing the filament current, the
rate of emission of electrons from the cathode will increase.
ICSE EXAMINATION QUESTION PAPER – 2015 (SOLVED)
Time : 2 hours 80 Marks
General Instructions:
1. Answers to this paper must be written on the paper provided separately.
2. You will not be allowed to write during the first 15 minutes. This time is to be spent in reading
the question paper.
3. The time given at the head of paper is the time allotted for writing the answers.
4. Attempt all questions from Section I and any four questions from Section II.
5. The intended marks of questions or parts of questions are given in brackets [ ].

SECTION - I (40 Marks)


Attempt all question from this Section.

Question 1
(a) When a body is placed on a table top, it exerts a force equal to its weight downwards on
the table top but does not move or fall. [2]

Table top

Force due to weight of the body

(i) Name the force exerted by the table top.


(ii) What is the direction of the force?
Ans.
(i) Normal reaction force is exerted by the table top.
(ii) The force is in the vertically upward direction.
(b) [2]
(i) Name one factor that affects the lateral displacement of light as it passes through a
rectangular glass slab.
(ii) On reversing the direction of the current in a wire, the magnetic field produced by it
gets ____________.
Ph 34 2015
Ph 35 2015

Ans.
(i) The thickness of the glass block, angle of incidence and refractive index of glass (any one) are
the factors which affect the lateral displacement of light as it passes through a rectangular glass
slab.
(ii) On reversing the direction of the current in a wire, the magnetic field produced by it gets
reversed.
(c) [2]
(i) On what factor does the position of the centre of gravity of a body depend?
(ii) What is the SI unit of the moment of force?
Ans.
(i) The position of the centre of gravity of a body depends on its shape / on the distribution of
mass.
(ii) The SI unit of moment of force is newton . metre (Nm).
(d) Name the factors affecting the turning effect of a body. [2]
Ans. The following are the factors which affect the turning effect of a body:
(1) Magnitude of the force applied
(2) The perpendicular distance between the point of action of force and the turning point on the
rigid body.
(e) [2]
(i) Define equilibrium.
(ii) In a beam balance when the beam is balanced in a horizontal position, it is in ______
equilibrium.
Ans.
(i) When several forces acting on a body produce no change in its state of rest or motion, the body
is said to be in equilibrium.
(ii) In a beam balance, when the beam is balanced in a horizontal position, it is in static equilibrium.

Question 2
(a) How is work done by a force measured when the force: [2]
(i) is in the direction of displacement?
(ii) is at an angle to the direction of displacement?
Ans.
(i) When force is in the direction of displacement  = 0o, then cos0 = 1.
Hence, the work done by a force measured in the direction of displacement is
W=F×S
Ph 36 2015

The work done is maximum and positive.


(ii) When the displacement is in the direction making an angle, with the direction of force,
Work done = Component of force in the direction of displacement × displacement W = F × S
W = FS cos 
(b) State the energy changes in the following while in use: [2]
(i) Burning of a candle. (ii) A steam engine.
Ans.
(i) Burning of a candle: Conversion of chemical energy to light energy and heat energy.
When a candle burns wax vapour first changes to light energy and then to heat energy.
(ii) A steam engine: Conversion of chemical energy heat energy to mechanical energy.
In a steam engine, chemical energy of coal first changes to heat energy. The heat energy
changes to kinetic energy of steam, and the kinetic energy of the steam then changes to
mechanical energy of the steam engine.
(c) [2]
(i) A scissor is a _________ multiplier. (ii) 1 kWh = ________ J.
Ans.
(i) A scissor is a speed multiplier because the effort applied is less than the load.
(ii) 1 kWh = 1 kilowatt × 1 hour
= 1000 J s–1 × 3600 s = 3.6 × 106 J
(d) Explain the motion of a planet around the Sun in a circular path. [2]
Ans. A planet moves around the Sun in a nearly circular path for which the gravitational force of
attraction on the planet by the Sun provides the centripetal force.
(e) Rajan exerts a force of 150 N in pulling a cart at a constant speed of 10 m/s. Calculate
the power exerted. [2]
Ans. Given that
Force = 150 N
Velocity = 10 m s–1
 Power = F × v = 150 N × 10 m s–1 = 1500 W

Question 3
(a) [2]
(i) Give the expression for mechanical advantage of an inclined plane in terms of the length
of an inclined plane.
(ii) Name a common device where a gear train is used.
Ph 37 2015

Ans.

l
(i) M.A. =
h
(ii) A clock is a common device where a gear train is used. In a clock, the gear system is used to
obtain gain in torque.
(b) The speed of light in glass is 2 × 105 km/s. What is the refractive index of glass? [2]
Ans. Given :
Speed of light in glass = 2 × 105 km/s = 2 × 108 m/s
Refractive index of glass is

Speed of light in air 3  10 5 km/s


μ glass 
Speed of light in glass = 2  10 5 km/s = 1.5

(c) [2]
(i) Draw a graph between displacement and the time for a body executing free vibrations.
(ii) Where can a body execute free vibrations?
Ans.
(i) The displacement–time graph for a body executing free vibrations is given below:

+a
Displacement

O
t

–a

(ii) The free vibrations of a body actually occur only in vacuum because the presence of a medium
offers some resistance due to which the amplitude of vibration does not remain constant and
decreases continuously.
Thus, we define free vibrations as the periodic vibrations of a body of constant amplitude in the
absence of any external force on it.
(d) [2]
(i) What happens to the resistivity of semi-conductor with the increase of temperature?
(ii) For a fuse, higher the current rating ________ is the fuse wire.
Ph 38 2015

Ans.
(i) The resistivity of a semiconductor decreases with increase in temperature.
(ii) For a fuse, higher the current rating, thicker is the fuse wire.
(e) [2]
(i) Name the high energetic invisible electromagnetic waves which help in the study of the
structure of crystals.
(ii) State an additional use of the waves mentioned in part (i).
Ans.
(i) X-rays are used in the study of structure of crystals.
(ii) X-rays are used in the detection of fracture in bones and teeth.
Question 4
(a) Rishi is surprised when he sees water boiling at 115.C in a container. Give reasons as to
why water can boil at the above temperature. [2]
Ans. The water boils at the higher temperature because of the reasons given below:
(1) The water used by Rishi might be impure. The boiling of a liquid increases with the addition of
impurities.
(2) Rishi might have used a container which creates a pressure within. The boiling point of a liquid
increases with an increase in pressure.
(b) [2]
(i) Why does a current carrying, freely suspended solenoid rest along a particular direction?
(ii) State the direction in which it rests.
Ans.
(i) A current-carrying freely suspended solenoid acts as a bar magnet, and thus, due to the Earth’s
magnetic field, it rests along a particular direction.
(ii) It rests in the North–South direction.
(c) Find the equivalent resistance between points A and B. [2]

Ans. Let RP be the equivalent resistance of the resistors 12 Ω , 6 Ω and 4 Ω connected in parallel.
Ph 39 2015

Hence, we have

1 1 1 1 1 2  3 1
   
R P 12 6 4 = 12 2

R P  2Ω
Therefore, the equivalent resistance of the circuit is 2
2Ω  R P  5Ω  2Ω  2Ω  5Ω  9Ω
Thus, the equivalent resistance between points A and B is 9 Ω
(d) Give two similarities between an AC generator and a DC motor [2]
Ans. Two similarities between an AC generator and a DC motor are
a. Both in an AC generator and DC motor A coil rotates in a magnetic field between the pole pieces
of a powerful electromagnet.
b. The external circuit is connected to two carbon brushes.
(e) [2]
(i) Why is a cathode ray tube evacuated to a low pressure?
(ii) What happens if the negative potential is changed on a grid?
Ans.
(i) The cathode ray tube is evacuated to a low pressure to avoid collisions of electrons with air
molecules.
(ii) If the negative potential on the grid is changed, then the number of electrons reaching the anode
and striking the screen changes which ultimately changes the brightness of the pattern of the
screen.
SECTION - II (40 Marks)
Attempt any four questions from this Section.

Question 5
(a) Draw a simplified diagram of a lemon crusher, indicting direction of load and effort. [2]
Ans. The diagram below shows a lemon crusher indicating the direction of effort (E) and load (L).

L
Ph 40 2015

(b) [4]
(i) Name the physical quantity measured in terms of horse power.
(ii) A nut is opened by a wrench of length 20 cm. If the least force required is 2N, find the
moment of force needed to loosen the nut.
(iii) Explain briefly why the work done by a fielder when he takes a catch in a cricket match
is negative.
Ans.
(i) Power is measured in horse power. 1 HP = 746 W
(ii) According to the principle of moments, we have Moment of load about the fulcrum = Moment
of effort about the fulcrum Load × Load arm = Effort × Effort arm
Given that the effort arm = 20 cm = 0.2 m, the minimum force E = 2 N
Therefore, the moment of load or the moment of force = 0.2 × 2 = 0.4 Nm
The moment of force needed to loosen the nut = 0.4 Nm
(iii) The work done by a fielder when he takes a catch in a cricket match is negative because the
force applied by the fielder is in the direction opposite to the displacement of the ball. The angle
o
between the force applied and the displacement of the ball is 180 We know that work done =
F.s cos 
Therefore, work done = – F.s
(c) A block and tackle system has V.R. = 5. [4]
(i) Draw a neat labelled diagram of a system indicating the direction of its load and effort.
(ii) Rohan exerts a pull of 150 kgf. What is the maximum load he can raise with this pulley
system if its efficiency = 75%?
Ans.
(i) A block and tackle system whose velocity ratio is 5 is as shown below:
(ii) Given that
Velocity ratio, VR = 5
Effort = 150 kgf
Efficiency = 75%
Mechanical Advantage

Mechanical Advantage
We know that efficiency, η 
Velocity Ratio

75 MA

100 5
L=l+x L
Ph 41 2015

75
MA = 5 ×
100
 Mechanical advantage = 3.75
Load(L)
But, Mechanical advantage = Effort(E)

 Load (L) = Mechanical advantage × Effort

= 3.75 × 150 Kgf = 562.5 Kgf

Therefore, Rohan can raise a maximum load of 562.5 kgf with this pulley system.

Question 6
(a) [3]
(i) Where should an object be placed so that a real and inverted image of the same size as
the object is obtained using a convex lens?
(ii) Draw a ray diagram to show the formation of the image as specified in the part a(i).
Ans.
(i) When an object is placed at 2F1 of a convex lens, a real and inverted image of the same size as
that of the object is formed at 2F2.
(ii) The ray diagram for the same is as shown below :

M
A Convex lens
Object

B O F2 2F2

2F1 F1 C2 B
C1
Image
A
N

(b) [3]
(i) Why does the Sun appear red at sunrise?
(ii) Name the subjective property of light related to its wavelength.
Ans. (i) The Sun appears red at sunrise because of the scattering of light by the atmospheric
Ph 42 2015

particles. During sunrise, the light from the Sun has to travel a longer distance through the
atmosphere to reach the observer. During this, most of the shorter wavelengths present in it are
scattered away from our line of sight by the molecules of air and other fine particles in the
atmosphere. So, the light reaching us directly from the rising Sun consists mainly of longer
wavelength red colour due to which the Sun appears red.
(ii) The subjective property of light related to its wavelength is colour.
(c) Jatin puts a pencil into a glass container having water and is surprised o see the pencil in
a different state. [4]
(i) What change is observed in the appearance of the pencil?
(ii) Name the phenomenon responsible for the change.
(iii) Draw a ray diagram showing how the eye sees the pencil.
Ans.
(i) The immersed part of the pencil appears to be bent and shortened.
(ii) The phenomenon responsible for the above observation is the refraction of light.
(iii) The ray diagram for the same is as shown below :

erlQuestion 7
(a) [2]
(i) State the safe limit of sound level in terms of decibel for human hearing.
(ii) Name the characteristic of sound in relation to its waveform.
Ans.
(i) The safe limit of sound level is upto 120 db.
(ii) The characteristic of sound in relation to its waveform is quality or timbre.
(b) A person standing between two vertical cliffs and 480 m from the nearest cliff shouts. He
hears the first echo after 3s and the second echo 2s later. Calculate : [3]
(i) The speed of sound.
(ii) The distance of the other cliff from the person.
Ph 43 2015

Ans.
(i) Let d1 be the distance of the nearest cliff and d2 be the distance of the farther cliff.
The time for the first echo is t1 = 3 s
The first echo will be heard from the nearest cliff.
The total distance travelled by sound before reaching the person is 2d1.
We know that

2d 2d1
Speed of sound = v = 
t t1

2  480
v=  320m/s
3
Hence, the speed of sound is 320 m/s.
(ii) The second echo is heard 2 s after the first one.
Hence, t2 = 3 + 2 = 5 s
Again the sound travels a total distance 2d2 before reaching the person.
So, we get

2d 2
v=
t2

vt 2 320  5
 d2    800m
2 2
Hence, the distance between the other cliff and the person is 800 m.
(c) In the diagram below, A, B, C, D are four pendulums suspended from the same elastic
string PQ. The length of A and C are equal to each other while the length of pendulum
B is smaller than that of D. Pendulum A is set in to a mode of vibrations. [5]
Ph 44 2015

(i) Name the type of vibrations taking place in pendulums B and D?


(ii) What is the state of pendulum C?
(iii) State the reason for the type of vibrations in pendulum B and C.
Ans.
(i) The vibrations which occur in pendulums B and D are called forced vibrations.
(ii) Pendulum C is in the state of resonance with pendulum A as it is of the same length.
(iii) The pendulums vibrate because the forced vibration from A is transferred due to string PQ.
Pendulum B is of a different length as compared to pendulum A. Hence, it will continuously
vibrate with a frequency which is different from that of pendulum A. Its amplitude will also be
very small.
Pendulum C is of the same length as compared to pendulum A. Hence, it will vibrate in phase
with pendulum A. Its amplitude will be equal to that of pendulum A as it will attain resonance.

Question 8
(a) [3]
(i) Name the device used to increase the voltage at a generating station.
(ii) At what frequency is AC supplied to residential houses?
(iii) Name the wire in a household electrical circuit to which the switch is connected.
Ans.
(i) The device used to increase voltage at the generating station is the step-up transformer.
(ii) The residential houses are supplied with AC of frequency 50 Hz.
(iii) The switch is connected to the live (or phase) wire in a household electric circuit.
(b) The relationship between the potential difference and the current in a conductor is
stated in the form of a law. [3]
(i) Name the law.
(ii) What does the slope of V-I graph for a conductor represent?
(iii) Name the material used for making the connecting wire.
Ans.
(i) The relationship between the potential difference and the current in a conductor is given by
Ohm’s law.
(ii) The slope of the V–I graph gives the resistance of the conductor.

V
Slope R = R
I
(iii) The material used for making connecting wires is copper.
Ph 45 2015

(c) A cell of Emf 2 V and internal resistance 1.2 Ù is connected with an ammeter of resistance
0.8 Ù and two resistors of 4.5 Ù and 9 Ù as shown in the diagram below: [4]

(i) What would be the reading on the Ammeter?


(ii) What is the potential difference across the terminals of the cell? [4]
Ans.
Given that
ε  2V, r  1.2Ω. RA  0.8Ω. R1  4.5Ω. R2  9Ω
(i) We know that for the circuit
ε  IR total
Now, the total resistance of the circuit is
R total = r + RA + RP

1 1 1 3
  
R P 4.5 9 9
 RP = 3 Ω
 R total = 1.2 + 0.8 + 3 = 5 Ω
Hence, the current through the ammeter is
ε 2
I= R = = 0.4A
total 5

EV
(ii) I =
r
Ph 46 2015

2V
 0.4 =
1 .2
 0.48 = 2 – V
V = 2 – 4.8 = 1.52 V
 Potential difference Vcell = 1.52 V
Question 9
(a) [2]
(i) Name a gas caused by the Greenhouse effect.
(ii) Which property of water makes it an effective coolant?
Ans.
(i) A gas caused by the greenhouse effect is carbon dioxide and chloroflurocarbons.
(ii) The high specific heat capacity (4200 J kg–1 k–1) of water makes it an effective coolant.
(b) [4]
(i) Water in lakes and ponds do not freeze at once in cold countries. Give a reason is
support of your answer.
(ii) What is the principle of Calorimetry?
(iii) Name the law on which this principle is based.
(iv) State the effect of an increase of impurities on the melting point of ice.
Ans.
(i) The specific latent heat of fusion of ice is sufficiently high (336 J g-1), and so to freeze water,
a large quantity of heat has to be withdrawn. Hence, it freezes slowly and thus keeps the
surroundings moderate.
(ii) Principle of calorimetry: If no heat energy is exchanged with the surroundings, i.e. if the system
is fully insulated, then the heat energy lost by the hot body is equal to the heat energy gained by
the cold body.
(iii) The principle of calorimetry is based on the law of conservation of energy.
(iv) Increasing the impurities causes the melting point of ice to decrease.
(c) A refrigerator converts 100 g of water at 20°C to ice at –10°C in 35 minutes.
Calculate the average rate of heat extraction in terms of watts.
Given: Specific heat capacity of ice = 2.1 J g–1.C–1
Specific heat capacity of water = 4.2 J g–1.C–1
Specific latent heat of fusion of ice = 336 J g–1 [4]
Ans. Given that
Mass of water converted to ice = m = 100 g
Ph 47 2015

Temperature of water tw = 20oC

Temperature of ice ti = –10oC


Total time t = 35 min = 2100 s
Specific heat capacity of ice = 2.1 J g–1oC–1

Specific heat capacity of water = 4.2 J g–1oC–1

Specific latent heat of fusion of ice = 336 J g–1

Amount of heat released when 100 g water cools from 20oC to 0oC is

Q1 = mc Δ T
= 100 × 4.2 × 20
= 8400 J
Amount of heat released when 100 g water converts to ice at 0oC is
Q2 = mL
= 100 × 336
= 33600 J
Amount of heat released when 100 g ice cools from 0oC to –10oC is

Q3 = mc Δ T
= 100 × 2.1 × 10
= 33600 J
Hence, the total heat released is
Q = Q 1 + Q2 + Q3
Q = 8400 + 33600 + 2100
Q = 44100 J
Therefore, the average rate of heat extraction is

Q 44100
P=  = 21 W
t 2100

Question 10
(a) [2]
(i) What is thermionic emission?
(ii) Name the unit in which the work function of a metal is expressed.
Ph 48 2015

Ans.
(i) The emission of electrons from a metal surface when heat energy is imparted to it is called
thermionic emission.
(ii) Work function of a metal is expressed in terms of electron volt (eV) where 1eV = 1.6 × 10–19J.
(b) [5]
(i) Complete the diagram as given below by drawing the deflection of radioactive radiations
in an electric filed.
(ii) State any two precautions to be taken while handling radioactive substances.
Ans.
(i) Deflection of radioactive radiations ,  and  in an electric field is as shown below:

Lead box
Radioactive substance

(ii) The two safety precautions to be taken while handling radioactive substances are (any two):
i. Radioactive substances should be kept in thick lead containers with a very narrow opening so as
to restrict the radiations coming out from other directions.
ii. Radioactive materials should be handled with long lead tongs.
Ph 49 2015

iii. People working with radioactive substances should put on special lead lined aprons and lead
gloves.
(c) An atomic nucleus A is composed of 84 protons and 128 neutrons. [3]
(i) The nucleus A emits an alpha particle and is transformed into nucleus B. What is the
composition of nucleus B?
(ii) The nucleus B emits a beta particle and is transformed into nucleus C. What is the
composition of nucleus C?
(iii) Does the composition of nucleus C change if it emits gamma radiations?
212 208 4
Ans. (i) 84
A 

82
B  2
He [Z = P + N = 84 + 128 = 212]
(-particle)
Composition of NUCLEUS (B) P = 82
N = 126
208 208
(ii) 82
B 

83
C  o
1( - particle)

One NEUTRON is converted into PROTON with emission of high speed electron (-
particle)
 Composition of NUCLEUS (C) P = 83
N = 125
(iii) MASS NUMBER OF A = P + N = 212
ON EMISSION of  -radiation there is no change in composition of C.
ICSE QUESTION PAPER-2016 (SOLVED)
PHYSICS
SCIENCE Paper – 3
(One hour and a half)
Answers to this Paper must be written on the paper provided separately.
You will not be allowed to write during the first 15 minutes.
This time is to be spent in reading the Question Paper,
The time given at the head of this Paper is the time alloted for writing the answers.
Attempt all questions from Section I and any four questions from Section II.
The intended marks of questions or parts of questions are given in brackets [ ].

SECTION I (40 Marks)


Attempt all questions from this Section
Question 1
(a)
(i) Give an example of a non contact force which is always of attractive nature.
(ii) How does the magnitude of this non contact force on the two bodies depend on the distance of
separation between them? [2]
(b) A boy weighing 40 kgf climbs up a stair of 30 steps each 20 cm high in 4 minutes and a girl
weighing 30 kgf does the same in 3 minutes. Compare:
(i) The work done by them.
(ii) The power developed by them. [2]
(c) With reference to the terms Mechanical Advantage, Velocity Ratio and efficiency of a machine,
name and define the term that will not change for machine of a given design. [2]
(d) Calculate the mass of ice required to lower the temperature of 300 g of water 40oC to water at
0oC. [2]
o
(Specific latent heat of ice = 336 J/g, Specific heat capacity of water = 4.2J/g C)
(e) What do you understand by the following statements:
(i) The heat capacity of the body is 60JK-1.
(ii) The specific heat capacity of lead is 130 Jkg-1K-1. [2]
Answer: (a)
(i) The example of non-contact force is gravitational force. It is always attractive in nature.
(ii) The magnitude of non-contact forces acting on two bodies depends on the distance of separation
between them. The magnitude of force decreases with an increase in separation and increases as
the separation decreases. It varies inversely as the square of the distance of separation.
(b) Given:
Force of gravity of the boy, Fb = 40 kgf
Time taken by him, tb = 4 minutes = 4 × 60 s = 240 s
Force of gravity of the girl, Fg = 30 kgf
Time taken by her, tg = 3 minutes = 3 × 60 s = 180 s

Ph 50 2016
Ph 51 2016

Distance covered by both in 30 steps is


D = 30 × 20 = 600 cm
While climbing, both have to do work against the force of gravity.
(i) Work done by the boy in climbing the stairs:
Wb = F × D = 40 kgf × 600 cm
Wb = 24000 J
Work done by the girl in climbing the stairs:
Wg = F × D = 30 kgf × 600 cm

Wg = 18000 J

Wb 24000J
 =
Wg 18000J

Wb 4
 =
Wg 3

Work done
(ii) Power developed 
Time taken
For the boy :

24000 J
Power Developed = = 100W
240 s
For the girl :
18000 J
Power Developed = = 100W
180 s
Thus, power developed by them is 1 : 1
Or
(i) Work done by the boy = 40 × 10 × 6 = 2400 J
Work done by the girl = 30 × 10 × 6 = 9800 J
 Ratio = 2400 : 1800 = 4 : 3
2400
(ii) Power of boy = = 10 watt
4  60
2400
Power of girl = = 10 watt
3  60
 Ratio = 1 : 1
(c) Velocity ratio.
It is the ratio of the velocity of effort to the velocity of load.
Ph 52 2016

VE
V.R. = V .
L
Velocity ratio does not change.
(d) Let m be the mass of the ice to be added.
Heat energy required to melt to lower the temperature is = mL = m × 336
Heat energy imparted by the water in fall of its temperature from 40°C to 0°C = mass of the water
× specific heat capacity × fall in temperature.
= 300 × 4.2 × 40°C
If there is no loss of heat,
m × 336 J/g = 300 g × 4.2 J/g°C × 40°C
300  4.2  40
 m= 336
 m = 150g
(e)
(i) Heat capacity is the amount of heat required to raise the temperature of a body by 1°C or 1 K.
Thus, 60 JK–1 of energy is required to raise the temperature of the given body by 1 K.
(ii) Specific heat capacity is the amount of heat energy required to raise the temperature of unit mass
of a substance through 1°C or 1K. Thus, 130 JKg–1 K–1 of heat energy required to raise the
temperature of unit mass of lead through 1 K.
Question 2
(a) State two factors upon which the heat absorbed by a body depends. [2]
(b) A boy uses blue colour of light to find the refractive index of glass. He then repeats the experiment
using red colour of light. Will the refractive index be the same or different in the two cases? Give
a reason to support your answer. [2]
A
(c) Copy the diagram given below and complete the path of the light ray
till it emerges out of the prism. The critical angle of glass is 42°. In 60°
your diagram mark the angles wherever necessary. [2]
P
(d) State the dependence of angle of deviation:
Q
(i) On the refractive index of the material of the prism. 60° 60°
(ii) On the wavelength of light [2] B C
(e) The ratio of amplitude of two waves is 3:4. What is the ratio of their:
(i) loudness?
(ii) Frequencies? [2]
Answer :
(a) Heat absorbed by a body depends on the mass of the body, change in temperature of the body and
the specific heat capacity of the body.
(b) The refractive index will be different in both cases.
Refractive index of glass is different for different colours. The speed of blue light is less than the
speed of red light. So, the wavelength of blue light is less than that of red light. Thus, red light
would deviate less than blue light because of difference in wavelength.
Ph 53 2016
(c) The diagram is as shown :

(d)
(i) Angle of deviation is directly proportional to the refractive index of the material of prism. For a
given angle of incidence, the prism with higher refractive index produces a greater deviation than
the prism which has a lower refractive index. Thus, the angle of deviation increases with an
increase in the refractive index of the medium.
(ii) Angle of deviation is inversely propotional to the wavelength of the light used. The angle of
deviation decreases with an increase in the wavelength of light. Thus, a prism deviates violet light
the most and red light the least.
(e)
(i) Let a1 and a2 be the amplitudes and I1 and I2 be the intensities of the two waves.

I1 a12 32
 I  2  2
2 a2 4

I1 9
 I  16  I1 : I1 = 9 : 16
2
(ii) Frequency is the number of waves formed per second. It only depends on time period. Thus, the
ratio of their frequencies is 1:1.
Question 3
(a) State two ways by which the frequency of transverse vibrations of a stretch string can be increased.
[2]
(b) What is meant by noise pollution? Name one source of sound causing noise pollution. [2]
(c) The V-I graph for a series combination and for a parallel combination of two resistors is shown in
the figure below. Which of the two A or B. represents the parallel combination? Give reasons for
Ph 54 2016
(d) A music system draws a current of 400 mA when connected to a 12 V battery. [2]
(i) What is the resistance of the music system?
(ii) The music system if left playing for several hours and finally the battery voltage drops to 320 mA
and the music system stops playing when the current.
(e) Calculate the quantity of heat produced in a 20  resistor carrying 2.5 A current in 5 minutes.
[2]
Answer :
(a) The frequency of transverse vibration is given by

1 T
f=
2l r 2d

1 T
or f =
2l m
where l = length of the vibrating string
T = tension in the string
m = mass per unit length of the string
Therefore, the frequency of transverse vibration of a stretched string can be increased by
(i) decreasing the length of the string
(ii) decreasing the mass per unit length of the string
(iii) increasing the tension T in the string
(b) The disturbance produced in the environment because of undesirable loud and harsh sound of
level more than 120 dB from the various sources such as a loudspeaker and moving vehicles is
called noise pollution.
Ex : Honking of vehicles in traffic jams.
(c) For the same change in I, change in V is less for the straight line A than for the straight line B (i.e.
the straight line A is less steep than B). The straight line A represents small resistance, while the
straight line B represents more resistance. The equivalent resistance is less in a parallel combination
than in a series combination. So, line A represents a parallel combination.
(d)
(i) Given : I = 400 mA = 400 × 10–3 A
V = 12 V
V = IR
V 12V
 R= I =
400×10-3A
 R = 30 
(ii) Current drops to I = 320 mA = 320 × 10–3 A
The music stops playing at
V = IR
= 320 × 10–3 × 30
 V = 9.6 V
Ph 55 2016
(e) Given : R = 20  , I = 2.5 A
t = 5 minutes = 5 × 60 = 300 s
Quantity of heat produced is given as
H = I2Rt
= (2.5)2 × 20 × 300
 H = 37500 J
Question 4
(a) State the characteristics required of a good thermion emitter. [2]
A 222
(b) An element ZS decays to 85R after emitting 2  particles and 1  particle. Find the atomic
number and atomic mass of the element S. [2]
(c) A radioactive substance is oxidized. Will there be any change in the nature of its radioactivity?
Give a reason for your answer. [2]
(d) State the characteristics required in a material to be used as an effective fuse wire. [2]
(e) Which coil of a step up transformer is made thicker and why? [2]
Answer :
(a) Two characteristic which a thermionic emitter should possess are
1. The work function of the substance should be low such that the electrons are emitted from it even
when it is not heated to a very high temperature.
2. The melting point of the substance should be quite high so that it may not melt when it is heated
to the temperature required for thermionic emission.
(b) The decay will follow the following sequence

Z
SA α
 z–2 P
A– 4

α
 z–4 Q
A– 8 β
  z–3 R
A– 8

Therefore, we have
Z – 3 = 85 or Z = 85 + 3 = 88
And A – 8 = 222 or A = 222 + 8 = 230
Z = 88, A = 230
(c) No, radioactivity is not affected by any physical process.
(d) The material should have high resistivity and low melting point.
(e) Primary coil, as it has lesser number of turns.
SECTION II (40 Marks)
(Attempt any four questions from this Section)
Question 5
(a) A stone of mass ‘m’ is rotated in a circular path with a uniform speed by tying a strong string with
the help of your hand. Answer the following questions : (3)
(i) Is the stone moving with a uniform or variable speed ?
(ii) Is the stone moving with a uniform acceleration ? In which direction does the acceleration act?
(iii) What kind of force acts on the hand and state its direction ?
(b) From the diagram given below. answer the question that follow : (3)
Ph 56 2016

T B
T T
A C

(i) What kind of pulleys are A and B ?


(ii) State the purpose of pulley B.
(iii) What effort has to be applied at C just raise the load L = 20 kgf ?
(Neglect the weight of pulley A and friction)
(c)
(i) An effort is applied on the bigger wheel of a gear having 32 teeth. It is used to turn a wheel of 8
teeth. Where it is used.
(ii) A pulley system has three pulleys. A load of 120 N is overcome by applying an effort of 50 N.
Calculate the Mechanical Advantage and Efficiency of this system. (4)
Answer :
(a)
(i) The stone is moving with uniform speed as given in the question.
(ii) Although the stone is rotating with uniform speed, its direction keeps on acceleration. The direction
of acceleration is towards the centre of the circular path.
(iii) Centrifugal force acting radially outwards.
(b)
(i) The pulley A is a single movable pulley and B is a single fixed pulley.
(ii) So as to apply the effort in a convinient direction that is vertically downwards.
(iii) Distance moved by load dL = x / 2.
Distance moved by the effort dE = x
Load = 20 kgf, Effort = ?
Now, Load × dL = Effort × dE
20 × x / 2 = Effort × x
or Effort = 10 kgf
(c)
(i) The number of teeth in the driving wheel is nB = 32 and that in the driven wheel is nA = 8. Hence,
this system is used to obtain gain in speed.
n B 32
Gain in speed = n = 8 =4
A
(ii) Load = 120 N; E = 50 N; n = 3
Mechanical advantage of pulley system is
Ph 57 2016

L 120
M.A. =  = 2.4
E 50
The efficiency of the system is
M.A 2.4 2.4
 = = = =0.8=80%
V.A n 3
Question 6.
(a)
(i) What is the principle of method of mixtures ? (3)
(ii) What is the other name given to it ?
(iii) Name the law on which the principle is based
(b) Some ice is heated at a constant rate, and its temperature is recorded after every few seconds, till
steam is formed at 100°C. Draw a temperature time graph to represent the change. Label the two
phase changes in your graph. (3)
(c) A copper vessel of mass 100 g contains 150 g of water at 50°C. How much ice is needed to cool
it to 5°C ? (4)
Given : Specific heat capacity of copper = 0.4 Jg–1 °C–1
Specific heat capacity of water = 4.2 Jg–1 °C–1
Specific latent heat of fusion ice = 336 Jg–1
Answer :
(a)
(i) The principle of method of mixture says that the heat lost by a hot body is equal to the heat gained
by a cold body provided no heat is last to the surroundings.
(ii) The other name given to the principle of mixture is the principle of calorimetry.
(iii) The principle of mixture is based on the law of conservation of energy.
(b) The figure for phase change is shown below :

Vaporization
100°C

temp

0°C Melting time

Correct graph (Axes and Shape)


Melting marked
Vaporization marked
(c) Heat energy lost by the vessel and water contained in it in cooling the water from 50°C to 5°C is
used in heating ice to melt it and then to raise its temperature from 0°C to 5°C.
Now, heat energy lost by the copper vessel is
Ph 58 2016
QC = mCCC  t = 100 × 0.4 × (50 – 5) = 100 × 0.4 × 45
QC = 1800 J
Similarly, heat energy lost by water is = 150 × 4.2 × 45
QW = mWCW  t = 150 × 4.2 × (50 – 5)
QW = 28350 J
Hence, the total heat energy lost is
QL = 1800 + 28350 = 30150 J
Let m g of ice be used to cool water. So, heat gained by ice is
QI = mLice + mcW  t
QI = 336 m + m × 4.2 × 5 = 336m + 21m = 357 J
Therefore, from the principle of calorimetry, the mass of ice is
QL = QI
 357m = 30150
30150
 m= 357
= 84.45 g

Question 7.
(a) (3)
(i) Write a relationship between angle of incidence and angle of refractions for a given pair of media.
(ii) When a ray of light enters from one medium to another having different optical densities it bends.
Why does this pehnomenon occur ?
(iii) Write one conditions where it does not bend when entering a medium of different optical density.
(b) A lens produces a virtual image between the object and the lens. (3)
(i) Name the lens.
(ii) Draw a ray diagram to show the formation of this image.
(c) What do you understand by the term ‘Scattering of light’ ? Which colour of white light is scattered
the least and why ? (4)
Answer : (a)
(i) The ratio of the sine of the angle of incidence to the sine of the angle of refraction is constant for
sin i
a given pair of media called the refractive index = =μ
sin r
(ii) When a ray of light enters from one medium to another with different optical densities, it bends
because there is a change in the speed of light in the two medium.
(iii) A ray of light passing from one medium to another does not bend when incident normally at the
surface of the second medium.
(b)
(i) The image formed by the lens is virtual and between the object and the lens. Hence, the lens used
is a concave lens.
(ii) The following image is of formation of the above image :
Ph 59 2016

B
B1

2F2 A F2 A1 O

(c) Scattering is the process of absorption and then re-emission of light energy without the change in
the wavelength.
The red colour of the white light is scattered the least because scattering of light depends inversely
upon the four power of wavelength. As red colour has the maximum wavelength in the visible
region, therefore, it scattered the least.
Question 8.
(a) (3)
(i) Name the waves used for echo depth sounding.
(ii) Give one reason for their use for the above purpose.
(iii) Why are the waves mentioned by you not audible to us ?
(b) (3)
(i) What is an echo
(ii) State two conditions for an echo to take place.
(c)
(i) Name the phenomenon involved in tuning a radio set to a particular station.
(ii) Define the phenomenon named by you in part (i) above.
(iii) What do you understand by loudness of sound ?
(iv) In which units is the loudness of sound measured ?
Answer :
(a)
(i) The waves used for echo depth sounding are ultrasonic waves.
(ii) Ultrasonic waves are used for echo depth ranging because they can travel undeviated through a
long distance.
(iii) Ultrasonic waves have frequency larger than 20000 Hz. Hence, these waves are not audible to us
as the audible range for the human ear is 20 Hz to 20000 Hz.
(b)
(i) The sound heard after reflection from a distant obstacle after the original sound has ceased is
called an echo.
(ii) The conditions for an echo to take place are
Ph 60 2016
a. The minimum distance between the source of sound and the reflector in air must be at least
present 16 to 18 m.
b. The size of the reflector must be large enough as compared to the wavelength of sound wave.
(c)
(i) The phenomenon involved in tuning a radio set to a particular station is called resonance.
(ii) Resonance : When the frequency of an externally applied periodic force on a body is equal to its
natural frequency, the body readily begins to vibrate with an increased amplitude. This phenomenon
is known as resonance.
(iii) Loudness is the property by virtue of which a loud sound can be distinguished from a faint one,
both having the same pitch and quality.
(iv) The loudness of sound is measured in phon / db / bel.
Question 9.
(a) (3)
(i) Which particles are responsible for current in conductors ?
(ii) To which wire of a cable in a power circuit should the metal case of a geyser be connected ?
(b) (3)
(i) Name the transformer used in the power transmitting station of a power plant.
(ii) What type of current is transmitted from the power station ? 12 V
(iii) At what voltage is this current available to our household ?
2
(c) A battery of emf 12 V and internal resistance 2  is
connected with two resistors A and B of resistance 4  and
6  respectively joined in series. (4)
Find : A B
(i) Current in the circuit
4 6
(ii) The terminal voltage of the cell.
(iii) The potential difference across 6  Resistor.
(iv) Electrical energy spent per minute in 4  resistor.
Answer :
(a)
(i) Electrons are responsible for current in conductors.
(ii) The metal case of a geyser should be connected to the Earth wire.
(iii) The fuse should always be connected to the live wire.
(b)
(i) A step-up transformer is used in the power transmitting station of a power plant.
(ii) An alternating current is transmitted from the power station.
(iii) The current is available to our household at a voltage of 220 V.
(c) E = 12 V ; r1 = 2 ; RA = 4 ; RB = 6 
(i) The current in the circuit is
E E
I= R =
total R1 + R A + R B
Ph 61 2016

12
 I = 2  4  6  1A

(ii) The terminal voltage of the cell is


T.V. = IR
T.V. = 1 × (6 + 4)
T.V. = 10 V
(iii) The potential difference across the 6  resistor is
VB = IRB
 VB = 1 × 6 = 6 V
(iv) The electrical energy spent per minute (= 60s) is
W = I2Rt
W = 12 × 4 × 60 = 240 J
Question 10.
(a) Arrange  ,  , and  rays in ascending order with respect to their (3)
(i) Penetrating power.
(ii) Ionising power.
(iii) Biological effect.
(b) (3)
(i) In a cathode ray tube what is the function of anode ?
(ii) State the energy conversion taking place in a cathode ray tube.
(iii) Write one use of cathode ray tube.
(c) (3)
(i) Represent the change in the nucleus of a radioactive element when a  particle is emitted.
(ii) What is the name given to elements with same mass number and different atomic number.
(iii) Under which conditions does the nucleus of an atom tend to radioactive ?
Answer :
(a)
(i) Penetrating power :     
(ii) Ionising power :  <  < 
(iii) Biological effect =  <  < 
(b)
(i) In a cathode ray tube, the anode accelerates the electrons and also focuses them in a fine energetic
beam.
(ii) In a cathode ray tube, electrical energy is converted into heat and than light energy.
(iii) A cathode ray tube is used to investigate the wave form of an unknown alternating potential by
applying it on the Y-plates and a known periodic time-based potential on the X-plates or picture
tube of T.V.
Ph 62 2016
(c)
(i) In an unstable nucleus, the neutron is changed into a proton by emitting a beta particle. This is
represented as
1 1 0
0 n  1p  –1 e
(neutron) (proton) ( -particle)

(ii) Elements with the same mass number but different atomic numbers are called isobars.
(iii) The nucleus of an atom tends to be radioactive when
— nucleus has more mass
— nucleus has excess energy
— neutrons proton ratio becomes more than 1.3 to 1.5.
ICSE EXAMINATION QUESTION PAPER – 2017 (SOLVED)
SCIENCE
Paper–I (PHYSICS)
(One hour and a half hours)
Answers to this paper must be written on the paper provided separately.
You will NOT be allowed to write during the first 15 minutes.
This time is to be spent in reading the question paper.
The time given at the head of this paper is the time allowed for writing the answers.
Section I is compulsory. Attempt any four questions from Section II.
The intended marks for questions or parts of questions are given in brackets [ ].

SECTION – I (40 Marks)


(Attempt all questions from this Section)
Question 1.
(a) A brass ball is hanging from a stiff cotton thread. Draw a neat labelled diagram showing the
forces acting on the brass ball and the cotton thread. (2)
(b) The distance between two bodies is doubled. How is the magnitude of gravitational force
between them affected? (2)
(c) Why is a jack screw provided with a long arm? (2)
(d) If the power of a motor be 100 kW, at what speed can it raise a load of 50,000 N? (2)
(e) Which class of level will always have MA > 1 and why? (2)
Answer 1.
(a) Support
Tension
(T)

Weight, (W)

(b) The magnitude of gravitational force between two bodies is inversely proportional to the square
of the distance between them.
1
G.F. 
r2
 As the distance is doubled, the force will become one fourth of the initial force.
(c) Increase in torque arm, increases the moment of force, so lesser effort is required for turming
(d) P = 100kW
 P = 100 × 1000W, F = 50000N
 Power = Force × Velocity
Ph 63 2017
Ph 64 2017

 100000 = 50000 × V
 V = 2 ms–1
(e) Class II lever.

Effort arm
Mechanical advantage = Load arm

In a class II lever, the effort arm is always longer than the load arm, hence its MA > 1.
Question 2.
(a) Define heat capacity and state its SI unit. (2)
(b) Why is the base of a cooking pan generally made thick? (2)
(c) A solid of mass 50 g at 150°C is placed in 100 g of water at 11°C, when the final temperature
recorded is 20°C. Find the specific heat capacity of the solid.
(Specific heat capacity of water = 4.2 J/g°C)
(d) How is the refractive index of a material related to :
(i) real and apparent depth?
(ii) velocity of light in vacuum or air and the velocity of light in a given medium? (2)
(e) State the conditions required for total internal reflection of light to take place. (2)
Answer 2.
(a) Heat capacity of a body is the heat energy required to raise the temperature of the body by 1°C.
Its S.I. unit is Jk–1.
(b) The base is thick hence the mass of the base is more.
This increases the heat capacity which enables the pan to hold a large amount of heat energy at a
low temperature. The food is cooked properly and remains warm for a long time.
(c) Heat lost by solid = Heat gained by water
(MCt)Solid = (MCt)Water
 50 × C × (150 – 20) = 100 × 4.2 × (20 – 11)

100  4.2  9
C=
50  130
 C = 0.582 Jg–1°C–1

Real depth
(d) (i) Refractive Index = Apparent depth

Velocity of light in air or vacuum


(ii) Refractive Index = Velocity of light in given medium

(e) (i) The ray of light must travel from denser medium to rarer medium.
(ii) The angle of incidence should be greater than the critical angle.
Ph 65 2017

Question 3.
(a) Draw a ray diagram to show the refraction of a monochromatic ray through a prism when it
suffers minimum deviation. (2)
(b) The human ear can detect continuous sounds in the frequency range from 20 Hz to 20,000 Hz.
Assuming that the speed of sound in air is 330 ms–1 for all frequencies, calculate the wavelengths
corresponding to the given extreme frequencies of the audible range. (2)
(c) An enemy plane is at a distance of 300 km from a radar. In how much time will the radar be able
to detect the plane? Take velocity of radiowaves as 3 × 108 ms–1. (2)
(d) How is the frequency of a stretched string related to :
(i) its length?
(ii) its tension? (2)
(e) Define specific resistance and state its SI unit. (2)
Answer 3.

(a) A
A
D
i e

B C

i  e
(b) V = 330 ms–1, f = 20 Hz
 V=f
 330 = 20 × 

330
=
20
highest = 16.5 m
V = 330 Hz, f = 20,000 Hz
 V=f

330
 =
20000
 lowest = 0.0165 m = 1.65 × 10–2 m
 Range of wavelength from 20 Hz to 20000 Hz is 16.5 m to 1.65 × 10–2 m.
(c) D = 300 km = 300000 m
V = 3 × 108 ms–1
Ph 66 2017

2D
 S=
T

2  300000
 3 × 108 =
T

2  300000
 T= = 2 × 10–3 s
3  108

1
(d) (i) Frequency of a stretched string is inversely proportional to its length.  f 

(ii) Frequency of a stretched string is directly proportional to its tension. f  T


(e) Specific resistance of a conductor is the resistance of unit length of the conductor and of unit
area of cross section.
Its SI unit is  m.
Question 4.
(a) An electric bulb of resistance 500  draws a current of 0.4 A. Calculate the power of the bulb
and the potential difference at its end.
(b) State two causes of energy loss in a transformer.
(c) State two characteristics of a good thermion emitter.
(d) State two factors upon which the rate of emission of thermions depends.
(e) When does the nucleus of an atom tend to be radioactive?
Answer 4.
(a) R = 500  I = 0.4 A
P= I2R = (0.4)2 × 500 = 80
 P = 80 W
 P = VI
 80 = V × 0.4

80
 V=
0.4
V = 200 V
(b) (i) Formation of Eddy currents—the changing magnetic field induces current in the soft-iron
core.
(ii) Resistance of primary or secondary coil.
(iii) Hysteresis—magnetisation of the core is reversed.
(c) (i) Low work function.
(ii) High melting point.
Ph 67 2017

(d) (i) Nature of the surface.


(ii) Temperature of the surface.
(e) (i) When the number of neutrons is greater than the number of protons in the nucleus.
(ii) When the atomic number of an element is more than 82.

SECTION – II (40 Marks)


(Attempt any four questions from this Section)
Question 5.
(a) A uniform half-metre rule balances horizontally on a knife edge at the 29 cm mark when a weight
of 20 gf is suspended from one end.
(i) Draw a diagram of the arrangement.
(ii) What is the weight of the half-metre rule? (3)
(b) (i) A boy uses a single fixed pulley to lift a load of 50 kgf to some height. Another boy uses a
single movable pulley to lift the same load to the same height. Compare the efforts applied by
them. Give a reason to support your answer.
(ii) How does uniform circular motion differ from uniform linear motion?
(iii) Name the process used for producing electricity using nuclear energy. (3)
(c) A pulley system with VR = 4 is used to lift a load of 175 kgf through a vertical height of 15 m.
The effort required is 50 kgf in the downward direction (g = 10 N kg–1).
(i) Distance moved by the effort.
(ii) Work done by the effort.
(iii) MA of the pulley system.
(iv) Efficiency of the pulley system. (4)
Answer 5.
(a) (i) 0 25 29 50
W gf

(ii) By principle of moments


Anticlockwise moment = clockwise moment
 f1d1 = f2d2
 W × 4 = 20 × 21
 W × 105 gf
 Weight of the rule = 105 gf
(b) For a single fixed pulley, MA = 1
(i)  Load = Effort  Effort = 50 kgf
For a single movable pulley, MA = 2
Load
 MA =
Effort
Ph 68 2017

50
 =2
Effort
 Effort = 25 kgf
Effort applied using a single fixed pulley 50
 Effort applied using a single movable pulley = 25 = 2 : 1
Effort applied by the boy using a single fixed pulley is twice the effort used by the boy using
a single movable pulley.
(ii) Uniform circular motion has variable velocity and hence variable acceleration. Uniform linear
motion has constant velocity and no acceleration and  is un accelerated motion.
(iii) Nuclear fission.
dE
(c) (i) Velocity ratio = VR = d
L

dE
 4=
15
 dE = 60 m
(ii) Work done by effort = Effort E × Distance dE
E = 50 kgf = 50 × 10 = 500 N
 Work done by effort = 500 N × 60 m = 30000 J
L 175
(iii) MA = = = 3.5
E 50
MA 3.5
(iv) Efficiency = × 100% = × 100% = 87.5%
VR 4
Question 6.
(a) (i) How is the transference of heat energy by radiation prevented in a calorimeter?
(ii) You have a choice of three metals, A, B and C, of specific heat capacities 900 Jkg–1°C–1,
380 Jkg–1°C–1 and 460 Jkg–1°C–1 respectively, to make a calorimeter. Which material will
you select? Justify your answer? (3)
(b) Calculate the mass of ice needed to cool 150 g of water contained in a calorimeter of mass 50 g
at 30°C such that the final temperature is 5°C. (3)
Specific heat capacity of calorimeter 0.4 J/g°C
Specific heat capacity of water 4.2 J/g°C
Latent heat capacity of ice 330 J/g
(c) (i) Name the radiations which are absorbed by greenhouse gases in the earth's atmosphere.
(ii) A radiation X is focused by a particular device on the bulb of a thermometer and mercury in
the thermometer shows a rapid increase. Name the radiation X.
(iii) Name two factors on which the heat energy liberated by a body depends. (4)
Ph 69 2017

Answer 6.
(a) (i) The outer and inner surfaces of the calorimeter are polished to reduce loss of heat due to
radiation.
(ii) Metal B, since metal B has lowest specific heat capacity among the three. So the amount of
heat energy taken by the calorimeter from its contents to acquire the temperature of its
contents is negligible.
(b) Heat gained by ice = mL + mc T
= m(330 + 4.2 × 5) = 351 mJ
Heat given out by water and calorimeter = (150 × 4.2 + 50 × .4) × (32 – 5) = 17550 J
17550
Mass of ice = = 50 g
351
(c) (i) Infrared radiations of loy wavelength.
(ii) Infrared radiations.
(iii) Mass, specific heat capacity.
Question 7.
(a) A lens forms an upright and diminished image of an object when the object is placed at the focal
point of the given lens.
(i) Name the lens.
(ii) Draw a ray diagram to show the image formation. (3)
(b) A ray of light travels from water to air as shown in the diagram given below :

Air
Water
°
48
=
1

(i) Copy the diagram and complete the path of the ray. Given that the critical angle for water is
48°.
(ii) State the condition so that total internal reflection occurs in the above diagram. (3)
(c) The diagram below shows a point source P inside a water container. Four rays A, B, C, D starting
from the source P are shown up to the water surface.

Air

C 48°
B D
A Water

P
Ph 70 2017

(i) Show in the diagram the path of these rays after striking the water surface. The critical angle
for water-air surface is 48°.
(ii) Name the phenomenon which the rays B and D exhibit. (4)
Answer 7.
(a) (i) Concave lens.

(ii)
B
B
O
A F1 A F2

Image is between F and O. It is virtual, erect and diminished.

(b) (i)

Air
Water
48°

(ii) (1) The ray of light must travel from denser medium to rarer medium.
(2) The angle of incidence should be greater than the critical angle.

(c) (i)
Air
B C 48° i r
A D
Water

(ii) Ray B exhibits refraction of light.


Ray D exhibits total internal reflection of light.
Question 8.
(a) Name the factor that determines : (3)
(i) Loudness of the sound heard.
(ii) Quality of the note.
(iii) Pitch of the note.
(b) (i) What are damped vibrations? (3)
(ii) Give one example of damped vibrations.
(iii) Name the phenomenon that causes a loud sound when the stem of a vibrating tuning fork is
kept pressed on the surface of a table.
Ph 71 2017

(c) (i) A wire of length 80 cm has a frequency of 256 Hz. Calculate that length of a similar wire
under similar tension, which will have a frequency of 1024 Hz. (4)
(ii) A certain sound has a frequency of 256 hertz and a wavelength of 1.3 m.
(1) Calculate the speed with which this sound travels.
(2) What difference would be felt by a listener between the above sound and another sound
travelling at the same speed, but of wavelength 2.6 m?
Answer 8.
(a) (i) Amplitude
(ii) Waveform
(iii) Frequency
(b) (i) The periodic vibrations of a body of decreasing amplitude in presence of a resistive force are
called damped vibrations.
(ii) Oscillations of a simple pendulum in air. A tuning fork when struck on a rubber pad.
(iii) Resonance

1
(c) (i) Length of wire  Frequency

f1 l2

f 2 l1
 L1 f1 = L2 f2
 80 cm × 256 Hz = L2 × 1024 Hz  L2 = 20 cm
 Length of the wire = 20 cm.
(ii) (1) V = f  = 256 × 1.3 = 332.8 ms–1
(2) As V = f   332.8 = f × 2.6
 f = 128 Hz.
Since the frequency of this sound is half of the first one the listener would feel the sound to
be less shrill or flat.
Question 9.
(a) (i) Name the colour code of the wire which has connected to the metallic body of an appliance.
(ii) Draw the diagram of a dual control switch when the appliance is switched 'ON'. (3)
(b) (i) Which particles are responsible for current in conductors?
(ii) To which wire of a cable in a power circuit should the metal case of a geyser be connected?
(iii) To which wire should the fuse be connected? (3)
(c) (i) Explain the meaning of the statement 'current rating of a fuse is 5A'.
(ii) In the transmission of power the voltage of power generated at the generating stations is
stepped up from 11 kV to 132 kV before it is transmitted. Why? (4)
Ph 72 2017

Answer 9.
(a) (i) Green or yellow.

(ii) Load

N
a a Supply
S1 S2 b L
b
c c

(b) (i) Electrons


(ii) Earth wire
(iii) Live wire
(c) (i) The live wire or the line wire has a current-carrying capacity of 5 A. Hence, a thin fuse wire
of low current-carrying capacity of 5 A is used. It is used for light and fan circuits.
(ii) The voltage is increased to decrease the current and prevent loss of energy due to heating in
the line wires.
Question 10.
(a) Answer the following questions based on a hot cathode ray tube. (3)
(i) Name the charged particles.
(ii) State the approximate voltage used to heat the filament.
(iii) What will happen to the beam when it passes through the electric field?
(b) State three factors on which the rate of emission of electrons from a metal surface depends.
(3)
(c) (i) What are the free electrons?
(ii) Why do they not leave the metal surface on their own?
(iii) How can they be made to leave the metal surface? (State any two ways) (4)
Answer 10.
(a) (i) Cathode rays / high speed electrons
(ii) 6 V
(iii) The electron beam gets deflected towards the positive plate in a direction opposite to the
direction of the electric field.
(b) (i) The nature of the metal surface
(ii) The temperature of the surface
(iii) The surface area of the metal.
(c) (i) Electrons in the outer orbits are weakly attracted by the nucleus. They are loosely bound and
leave their individual atom and become free to move inside the solid. These electrons are
called free electrons.
(ii) The free electrons do not have sufficient kinetic energy to leave its surface.
(iii) The free electrons can leave the metal surface by
(1) Thermionic emission: when energy is supplied as heat.
(2) Photo-electric emission: when energy is supplied as light.
ICSE EXAMINATION QUESTION PAPER – 2018 (SOLVED)
SCIENCE
Paper–I (PHYSICS)
(One hour and a half hours)
Answers to this paper must be written on the paper provided separately.
You will NOT be allowed to write during the first 15 minutes.
This time is to be spent in reading the question paper.
The time given at the head of this paper is the time allowed for writing the answers.
Section I is compulsory. Attempt any four questions from Section II.
The intended marks for questions or parts of questions are given in brackets [ ].

SECTION – I (40 Marks)


(Attempt all questions from this Section)
Question 1.
(a) (i) State and define the SI unit of power.
(ii) How is the unit horse power related to the SI unit of power? (2)
(b) State the energy changes in the following cases while in use : (2)
(i) An electric iron.
(ii) A ceiling fan.
(c) The diagram below shows a lever in use : (2)
(i) To which class of levers does it belong?
(ii) Without changing the dimensions of the lever, if the load is shifted towards the fulcrum what
happens to the mechanical advantage of the lever?
E

F
L

(d) (i) Why is the ratio of the velocities of light of wavelengths 4000 A and 8000 A in vacuum 1 : 1?
(ii) Which of the above wavelengths has a higher frequency? (2)
(e) (i) Why is the motion of a body moving with a constant speed around a circular path said to be
accelerated?
(ii) Name the unit of physical quantity obtained by the formula 2K/V2 where K: kinetic energy, V:
Linear velocity. (2)
Answer 1.
(a) (i) The SI unit of power is watt (W). The power of an agent is said to be one watt if it does one
joule of work in one second.
(ii) 1 hp = 746 W
Ph 73 2018
Ph 74 2018
(b) (i) Electrical into heat energy
(ii) Electrical into mechanical energy.
(c) (i) Second class lever.
(ii) It increases.
(d) (i) Because all wavelengths of light travel with the same velocity in vacuum.
(ii) 4000 A higher frequency..
(e) (i) Because at every point of motion the direction of speed changes i.e., the body possesses
velocity which changes with time.
(ii) Kilogram (kg).
Question 2.
(a) The power of a lens is -5D. (2)
(i) Find its focal length.
(ii) Name the type of lens.
(b) State the position of the object in front of a converging lens if : (2)
(i) It produces a real and same size image of the object.
(ii) It is used as a magnifying lens.
(c) (i) State the relation between the critical angle and the absolute refractive index of a medium.
(ii) Which colour of light has a higher critical angle? Red light or Green light. (2)
(d) (i) Define scattering. (2)
(ii) The smoke from a fire looks white. Which of the following statements is true?
(1) Molecules of the smoke are bigger than the wavelength of light.
(2) Molecules of the smoke are smaller than the wavelength of light.
(e) The following diagram shows a 60°, 30°, 90° glass prism of critical angle 42°. Copy the diagram
and complete the path of incident ray AB emerging out of the prism marking the angle of
incidence on each surface. (2)

X
60°

B 30°
Y Z
A

Answer 2.
1 1
(a) (i) f = = = –0.2 m = 20 cm
P 5
(ii) Concave
(b) (i) At 2F
(ii) Between the optical center and principle focus.
Ph 75 2018

1
(c) (i)  = sin C

(ii) Red
(d) (i) Scattering is a general physical process where light is forced to deviate from a straight
trajectory by one or more paths due to localized non-uniformities in the medium through
which it passes.
(ii) Molecules of the smoke are bigger than the wavelength of light.
(e) The diagram is as shown

60°

45°
45°
B 30°
Y Z

Question 3.
(a) Displacement distance graph of two sound waves A and B, travelling in a medium, are as shown
in the diagram below. (2)

20 A
10 B
(0, 0) Distance (cm)
10
20

Study the two sound waves and compare their:


(i) Amplitudes
(ii) Wavelengths
(b) You have three resistors of values 2 , 3  and 5 . How will you join them so that the total
resistance is more than 7 ? (2)
(i) Draw a diagram for the arrangement.
(ii) Calculate the equivalent resistance.
(c) (i) What do you understand by the term nuclear fusion? (2)
(ii) Nuclear power plants use nuclear fission reaction to produce electricity. What is the
advantage of producing electricity by fusion reaction?
Ph 76 2018

(d) (i) What do you understand by free vibrations of a body? (2)


(ii) Why does the amplitude of a vibrating body continuously decrease during damped
vibrations?
(e) (i) How is the emf across primary and secondary coils of a transformer related with the number
of turns of coil in them? (2)
(ii) On which type of current do transformers work? (2)
Answer 3.
(a) (i) AA / AB = 20 / 10 = 2 / 1 = 2 : 1  Amplitudes of A and B are in the ratio 2 : 1
(ii) B > A  Wavelength of A and B are in the ratio 1 : 2
(b) (i) All of these can be connected in series as shown
2 3 5

(ii) R = 2 + 3 + 5 = 10 
(c) (i) It is a process in which two or more smaller nuclei combine to form a bigger nucleus.
(ii) The product formed is not radioactive hence less harmful to humans / energy produced per
nucleon is more them fission.
(d) (i) If a body vibrates with its own frequency it is called a free vibration.
(ii) Energy is used up in overcoming friction.
VP NP
(e) (i) e.m.f. and the number of turns of the coil are directly proportional = V = N
S S

(ii) Alternating current. (A.C.)


Question 4.
(a) (i) How can a temperature in degree Celsius be converted into SI unit of temperature? (2)
(ii) A liquid X has the maximum specific heat capacity and is used as a coolant in Car radiators.
Name the liquid X.
(b) A solid metal weighing 150 g melts at its melting point of 800°C by providing heat at the rate of
100 W. The time taken for it to completely melt at the same temperature is 4 mm. What is the
specific latent heat of fusion of the metal? (2)
(c) Identify the following wires used in a household circuit : (2)
(i) The wire is also called as the phase wire.
(ii) The wire is connected to the top terminal of a three-pin socket.
(d) (i) What are isobars?
(ii) Give one example of isobars.
(e) State any two advantages of electromagnets over permanent magnets. (2)
Answer 4.
(a) (i) 1°C = 273 K
(ii) Water
100  4  60
(b) Using Q = mL, 100 × 4 × 60 = 150 × 10–3 × L or L = = L = 1.6 × 105 J kg–1
150  10 3
Ph 77 2018
(c) (i) Live wire
(ii) Earth wire
(d) (i) Elements having same mass number but different atomic number.
(ii) 14 14 23 23
6 C, 7 N and 12 Mg, 11 Na

(e) (i) Strength can be increased or decreased during working.


(ii) Polarities can be changed during working.

SECTION – II (40 Marks)


(Attempt any four questions from this Section)
Question 5.
(a) (i) Derive a relationship between SI and C.G.S. unit of work. (3)
(ii) A force acts on a body and displaces it by a distance S in a direction at an angle  with the
direction of force. What should be the value of  to get the maximum positive work?
(b) A half metre rod is pivoted at the centre with two weights of 20 gf and 12 gf suspended at a
perpendicular distance of 6 cm and 10 cm from the pivot respectively as shown below: (3)

50 cm

20 gf 12 gf
6 cm 10 cm

(i) Which of the two forces acting on the rigid rod causes clockwise moment?
(ii) Is the rod in equilibrium?
(iii) The direction of 20 kgf force is reversed. What is the magnitude of the resultant moment of
the forces on the rod?
(c) (i) Draw a diagram to show a block and tackle pulley system having a velocity ratio of 3
marking the direction of load (L), effort (E) and tension (T).
(ii) The pulley system drawn lifts a load of 150 N when an effort of 60 N is applied. Find its
mechanical advantage.
(iii) Is the above pulley system an ideal machine or not? (4)
Answer 5.
(a) (i) 1 J = 1 kg × 1 m2 × 1 s–2 = 103 g × 104 cm2 × 1 s–2 g cm2 s–2 = 107 erg
(ii) W = FS cos  = FS (maximum) if cos  = 1 or  = 0°
(b) (i) 12 kgf
(ii) Yes
Ph 78 2018
(iii) 120 + 120 = 240 dyne cm
(c) (i) The diagram is as shown

T
E

T T
T

load 150
(ii) Actual MA number of pulleys in the system = 3. Observed MA = = = 2.5
effort 60
(iii) Since, observed MA is lesser than the theoretical MA therefore, the machine is not an ideal
machine.
Question 6.
(a) A ray of light XY passes through a right angled isosceles prism as shown below. (3)

45°

Y
X

45°
C B

(i) What is the angle through which the incident ray deviates and emerges out of the prism?
(ii) Name the instrument where this action of prism is put into use.
(iii) Which prism surface will behave as a mirror?
(b) An object AB is placed between O and F1 on the principal axis of converging lens as shown in the
diagram. (3)

1F1 F1 B O F2 2F2

LENS
Ph 79 2018
Copy the diagram and by using three standard rays starting from point A, obtain an image of the
object AB.
(c) An object is placed at a distance of 12 cm from a convex lens of focal length 8 cm. Find : (4)
(i) the position of the image
(ii) nature of the image.
Answer 6.
(a) (i) 90°
(ii) Refracting Periscope
(iii) Surface AB
(b) The figure is as shown.

(c) Given u = –12 cm, f = +8 cm, v = ?, nature = ?


1 1 1
Using the lens formula = – we have
f v u

1 1 1 1 1 3 2 1
= f + = + = =
v u 8  12 24 24
Or v = 24 cm,
(ii) Image will be real or inverted.
Question 7.
(a) Draw the diagram of a right angled isosceles prism which is used to make an inverted image
erect. (3)
(b) (3)

Wooden
Sonometer
Box
Paper
rider Tuning Tuning
10 kg
Fork A Fork B
The diagram above shows a wire stretched over a sonometer. Stems of two vibrating tuning
forks A and B are touched to the wooden box of the sonometer. It is observed that the paper rider
(a small piece of paper folded at the centre) present on the wire flies off when the stem of
vibrating tuning fork B is touched to the wooden box but the paper just vibrates when the stem of
vibrating tuning fork A is touched to the wooden box.
(i) Name the phenomenon when the paper rider just vibrates.
(ii) Name the phenomenon when the paper rider flies off.
(iii) Why does the paper rider fly off when the stem of tuning fork B is touched to the box?
(c) A person is standing at the sea shore. An observer on the ship, which is anchored in between a
Ph 80 2018
vertical cliff and the person on the shore, fires a gun. The person on the shore hears two sounds,
2 seconds and 3 seconds after seeing the smoke of the fired gun. If the speed of sound in the air
is 320 m s–1 then calculate :
(i) the distance between the observer on the ship and the person on the shore.
(ii) the distance between the cliff and the observer on the ship. (4)
Cliff
Man
Observer
Ship
Sea shore
Water
Answer 7.
(a) The diagram is as shown.

P
45°
Object
45°
Q
90°
C
Q
45°
Image 45°
P

(b) (i) Forced vibrations


(ii) Resonance
(iii) The wire of the sonometer begins to vibrate with its natural frequency. This is called
resonance. At resonance the amplitude of vibration becomes very large.
(c) Let distance between the observer on the ship and person on the shore be x. For the first sound
which the person hears after 2 seconds, we have
x
v=  x = vt = 320 × 2 = 640 m
t
Let distance between the ship and the cliff be x, then in the second case
2 x  640
v=  2x + 640 = vt = 320 × 3 = 960
t
Or 2x = 960 – 640 = 320 m
320
x= = 160 m
2
Question 8.
(a) (i) A fuse is rated 8 A. Can it be used with an electrical appliance rated 5 kW, 200 V? Give a
Ph 81 2018
reason. (3)
(ii) Name two safety devices which are connected to the live wire of a household electric circuit.
(b) (i) Find the equivalent resistance between A and B.
6 4

A B

3 12 
(ii) State whether the resistivity of a wire changes with the change in the thickness of the wire.
(c) An electric iron is rated 220 V, 2 kW. (4)
(i) If the iron is used for 2 h daily find the cost of running it for one week if it costs ` 4.25 per
kWh.
(ii) Why is the fuse absolutely necessary in a power circuit?
Answer 8.
P 5000
(a) (i) The current which will pass through the electrical device will be I = = = 25 A.
V 200
The fuse cannot be used as it will blow off when the current exceeds 8 A.
(ii) MCB Fuse, switch, ELCB.
(b) (i) 6  and 3  are in parallel therefore,
63 18
= =2
63 9
4  and 12  are in parallel therefore,
4  12 48
= =3
4  12 16
The above two are now in series therefore, R = 2 + 3 = 5
(ii) Resistivity is independent of the dimensions of the wire. Therefore, no change.
(c) (i) V = 220 V, P = 2 kW = 2000 W
P (in watt)  t (in hour) 2000  2  7
Energy consumed E = = = 28 kW h
1000 1000
Hence cost = 28 × 4.25 = ` 119
(ii) Power circuits draw large amount of current, Electric shock in this circuit is very fatal
hence to avoid it fuse is necessary in the power circuit.
Question 9.
(a) (i) Heat supplied to a solid change it into liquid. What is this change in phase called? (3)
(ii) During the phase change does the average kinetic energy of the molecules of the substance
increase?
(iii) What is the energy absorbed during the phase change called?
(b) (i) State two differences between "Heat Capacity" and "Specific Heat Capacity". (3)
Ph 82 2018
(ii) Give a mathematical relation between Heat Capacity and Specific Heat Capacity.
(c) The temperature of 170 g of water at 50°C is lowered to 5°C by adding certain amount of ice to
it. Find the mass of ice added. Given: Specific heat capacity of water = 4200 J kg–1 °C–1 and
Specific latent heat of ice 336000 J kg–1. (4)
Answer 9.
(a) (i) Melting (ii) No (iii) Latent heat of fusion.
(b) (i) The two differences are
Specific heat capacity Heat capacity
1. Heat required to raise the temperature 1. Heat required to raise the temperature
of a unit mass of a substance by 1 degree of a substance by 1 degree Celsius.
Celsius.
2. Does not depends upon the mass of the 2. Depend upon of mass of the substance.
substance.
(ii) The relation is, heat capacity = Specific heat × Mass of substance.
(c) Given mass of ice = mg
Mass of water = 170 g
Initial temperature of water = 50°C
Let the final temperature of the mixture = 5°C
The latent heat required to change m g of ice at 0°C to m g of water at 0°C
= m × 336 = 336 m J
Now, heat required to change m g of water at 0°C to 5°C
= m × 4.2 × (5 – 0) J = 21 m J
Now, heat lost by 170 g of water to reach 5°C from 50°C
= 170 × 4.2 × (50 – 5) = 32 130 J
By principle of calorimetry
Heat lost = Heat gained
336 m + 21 m = 32130
or 357 m = 32130
m = 90 g
Question 10.
(a) The diagram shows a coil wound around a U shape soft iron bar AB.

A B
+ 
Ph 83 2018

(i) What is the polarity induced at the ends A and B when the switch is pressed?
(ii) Suggest one way to strengthen the magnetic field in the electromagnet.
(iii) What will be the polarities at A & B if the direction of current is reversed in the circuit? (3)
(b) The ore of Uranium found in nature contains 92U238 and 92U235. Although both the isotopes are
fissionable, it is found out experimentally that one of the two isotopes is more easily fissionable.
(i) Name the isotope of Uranium which is easily fissionable.
(ii) Give a reason for your answer.
(iii) Write a nuclear reaction when Uranium 238 emits an alpha particle to form a Thorium (Th)
nucleus. (3)
(c) Radiations given out from a source when subjected to an electric field in a direction perpendicular
to their path are shown below in the diagram. The arrows show the path of the radiation A, B and
C. Answer the following questions in terms of A, B and C. (4)
(i) Name the radiation B which is unaffected by the electrostatic field.
(ii) Why does the radiation C deflect more than A?

B
A C

Radioactive
Substance Lead Box

(iii) Which among the three causes the least biological damage externally?
(iv) Name the radiation which is used in carbon dating.
Answer 10.
(a) (i) A South and B South.
(ii) Increasing the strength of current through the coil.
(iii) The Polarities of the two ends become north.
(b) (i) 92U235
(ii) It readily absorbs thermal neutrons and becomes unstable.
(iii) The reaction is
238
92U  90Th234 + 2He4
(c) (i) Gamma radiation (ii) Because C is lighter than A.
(iii) Alpha (A) (iv) Carbon-14
CLASS X (PHYSICS)
(ICSE 2020)
SCIENCE PAPER-1

(Two hours)
Answers to this Paper must be written on the paper provided separately.
You will not be allowed to write during the first 15 minutes.
This time is to be spent in reading the Question paper.
The time given at the head of this Paper is the time allowed for writing the answers.
Section I is compulsory. Attempt any four questions from Section II.
The intended marks for questions or parts of questions are given in brackets [].

SECTION - I (40 marks) of rotation is known as the moment


Attempt all questions from this Section. of force. It is also known as torque.
1. (a) (i) Define moment of force. 2 (ii) Since the SI unit of moment of
(ii) Write the relationship between force is Newton metre, i.e., Nm and
the SI and CGS unit of moment the CGS unit of moment of force
of force. is dyne cm.
(b) Define a kilowatt hour. How is it So, the required relationship
related to joule ? 2 between the SI and CGS unit of
(c) A satellite revolves around a planet moment of force will be
in a circular orbit. What is the work 1 Nm = 105 dyne × 102 cm
done by the satellite at any instant? = 107 dyne cm
Give a reason. 2 (b) One kilowatt hour is defined as the
(d) (i) Identify the class of the lever electrical energy which is consumed by
shown in the diagram below : 2 the appliance of1 kW power in duration
of 1 hour.
 1 kilowatt hour = 1 kilowatt × 1 hour
= 1000 watts × 1 hour
(ii) How is it possible to increase the { 1 kilowatt = 1000 watts}
M.A. of the above lever without = 1000 watts × 3600 s
increasing its length ? { 1 hour = 3600 s}
(e) Give one example of each when : 2  1J 
(i) Chemical energy changes into and  1 watt  
 s
electrical energy.
(ii) Electrical energy changes into J
= 1000 × × 3600 s
sound energy. s
Sol. (a) (i) The product of magnitude of force = 3600000 J
and the perpendicular distance of
line of action of force from the axis 1 kilowatt hour = 3.6 × 106 J

Ph 99 2020
Ph 100 2020
So, this is the required relation between chemical energy into electrical
kilowatt hour and Joule. energy.
(c) It is given that a satellite is revolving (ii) In case of electric bell, the electrical
around the Earth, So, let us represent energy changes into sound energy
the given situation through the as the sounds produced by the
following diagram, at particular instant. electrical signals is converted into
a form of sound energy.
2. (a) A crane ‘A’ lifts a heavy load in 5
seconds, whereas another crane ‘B’
does the same work in 2 seconds.
Compare the power of crane ‘A’ to
that of crane ‘B’. 2
(b) A ray of light falls normally on a
rectangular glass slab. 2
Draw a ray diagram showing the
Note: When the direction of acting
path of the ray till it emerges out of
force on the body is perpendicular to
the slab.
the direction of displacement, then work
done remains zero. So, in this given (c) Complete the path of the mono-
situation, the work done by the satellite chromatic light ray AB incident on
at any instant will be zero. It is because the surface PQ of the equilateral glass
 prism PQR till it emerges out of the
the centripetal force ( Fc ) is perpendi- prism due to refraction. 2
cular to the direction of the displace-
ment/motion.
(d) (i) Consider the following diagram of
lever.

The given diagram is of class III


lever, because Effort (E) is lying
between Fulcrum (F) and Load (L).
(ii) As Mechanical Advantage (d) Where should an object be placed in
front of a convex lens in order to get :
Effort arm
 . 2
load arm (i) an enlarged real image
So, if the distance of the effort from (ii) enlarged virtual image ?
the fulcrum is increased, then the (e) A pond appears to be 2.7 m deep. If
mechanical advantage of the above 4
lever will also be increased without the refractive index of water is ,
3
increasing its length. find the actual depth of the pond. 2
(e) (i) In case of primary cell, chemical Sol. (a) According to question, crane A lifts a
energy changes into electrical heavy load in 5 seconds and crane B
energy as it is used to convert stored does the same work in 2 seconds.
Ph 101 2020

work done (c)


 Power 
time taken
 Power = work done / time taken
W
PA  ...(i)
5s
{ For crane A, t = 5s}
and for crane B, In the given ray diagram,
W i = Angle of incidence
PB  ...(ii) e = Angle of emergence
2s
{ For crane B, t = 2s} r1 and r2 are the angle of refractions.
On dividing eqns. (i) and (ii), we get AB = Incident ray
BC = Refracted ray
W
PA PA 2 CD = Emergent ray.
 5   So, this is the required path of the
PB W PB 5
monochromatic light ray AB incident
2 on the surface PQ of equilateral glass
or PA  0.4 PB prism PQR till it emerges out of the
prism due to refraction.
i.e., the power of crane A is 0.4 times (d) (i) To get an enlarged real image, the
to that of crane B. object should be placed (in front of
(b) When a ray of light falls normally on a a convex lens) between F and 2F.
rectangular glass slab, then it will be
shown as follows :

(ii) To get an enlarged virtual image,


the object should be placed (in front
of a convex lens)between F and O.

So, in the given case, Angle of


incidence = Angle of refraction = Angle
of emergence = 0
i.e., i = r = e = 0
Hence, the above ray diagram is the
required ray diagram which shows the
path of the ray till it emerges out of the (e) Actual depth of pond = ?
slab. Apparent depth of pond = 2.7 m.
Ph 102 2020

4 wavelength of blue light blue


Refractive index of water  = 4.8 × 10–7 cm
3
 Velocity = wavelength × frequency
 Refractive index of medium
 V = f ...(i)
Real depth or Actual depth As we know that frequency of light

Apparent depth does not change when it enters into
a medium, i.e., f is constant.
4 Actual depth  eqn. (i) becomes
 
3 2.7 m V ...(ii)
4 As given
 Actual depth   2.7 m red > blue ...(iii)
3
= 4 × 0.9 m from eqns (ii) and (iii), we get

Vred  Vblue
 Actual depth = 3.6 m
It means that the red colour light
3. (a) The wave lengths for the light of red
will have greater speed in glass.
and blue colours are nearly 7.8 ×
10–7 m and 4.8 × 10–7 m respectively. (b)
(i) Which colour has the greater
speed in a vacuum ?
(ii) Which colour has a greater speed
in glass ? 2
(b) Draw a graph between displacement
from mean position and time for a
body executing free vibration in a
vacuum. 2 This is the required graph between the
(c) A sound wave travelling in water has displacement from mean position and
wavelength 0.4 m. 2 time for a body executing free
Is this wave audible in air ? (The vibrations in a vacuum.
speed of sound in water = 1400 ms–1) (c) It is given that a sound wave is
(d) Why does stone lying in the sun get travelling in water has wavelength of
heated up much more than water 0.4 m, i.e.,  = 0.4 m
lying for the same duration of time ? Speed of sound in water, V =1400 ms–1
2  Velocity = wavelength × Frequency
(e) Why is it not advisable to use a piece  V=×f
of copper wire as fuse wire in an
V 1400 14000
electric circuit ? 2  f    = 3500 Hz
Sol. (a) (i) Since speed does not change in  0.4 4
vacuum, therefore, all colours will As we know that audible frequency
move with the same speed, i.e., with ranges between 20 Hz to 20,000 Hz.
the speed of light (3 × 108 m/s). And the obtained value of frequency,
(ii) It is given that wavelength of red i.e., 3500 Hz lies between this range,
light, Red= 7.8 × 10–7 m and i.e. in between 20 Hz and 20,000 Hz.
Ph 103 2020
So, yes this sound wave is audible in
air.
(d) The stone lying in the sun get heated
much more than water lying for the
same duration of time. It is because the
specific heat capacity of stone is less
than that of water, due to which the
temperature of stone is more.
(e) It is not advisable to use a piece of
copper wire as fuse wire in an electric (i) What is the magnetic polarity of
circuit. It is because copper has low the loop that faces us ?
specific resistance and high melting (ii) With respect to the diagram how
point, due to which it will not can we increase the strength of
break(fuse) during the overloading in the magnetic field produced by
the circuit. this loop ?
4. (a) Calculate the total resistance across Sol. (a) Consider the given diagram,
AB : 2

It can be seen from the figure that R1


and R2 are in parallel.
(b) Two metallic blocks P and Q having
masses in ratio 2:1 are supplied with R1R 2
So, their equivalent, Req1 
the same amount of heat. If their R1 + R 2
temperatures rise by same degree,
3  6 18
compare their specific heat  Req1    2
capacities. 2 36 9
Now, the above given diagram can be
(c) When a current carrying conductor shown as follows
is placed in a magnetic field, it
experiences a mechanical force.
What should be the angle between
the magnetic field and the length of  Req1 and R3 are in series.
the conductor so that the force  equivalent resistance across AB will be
experienced is : 2 RAB = Req1 + R3 = (2 + 5)  = 7.
(i) Zero (b) Consider a metallic block having mass
‘m’, specific heat capacity ‘S’, gains
(ii) Maximum ? heat energy ‘Q’ after rising the
(d) A nucleus 84X202 of an element emits temperature ‘T’, then the relation
an alpha particle followed by a beta among them is given by,
particle. The final nucleus is aY b.  Q = mST ...(i)
Find a and b. 2 For block A, Mass = 2m,
(e) The diagram below shows a loop of Specific heat capacity = SP
wire carrying current I : 2 temperature = T
Ph 104 2020
For block B, Mass = m SECTION - II (40 marks)
Specific heat capacity = SQ Attempt any four questions from this Section.
temperature = T 5. (a) The figure below shows a simple
As per question, pendulum of mass 200 g. It is
QP = QQ ...(ii) displaced from the mean position A
 from eqns. (i) and (ii), we get to the extreme position B. The
potential energy at the position A is
2m SP T = m SQ T
zero. At the position B the pendulum
 2SP = SQ
bob is raised by 5 m. 3
1
 SP  S
2 Q
It means that specific heat capacity of
block P is half of the capacity of block
Q.
(c) It is given that when a current carrying
conductor is placed in a magnetic field,
it experiences a mechanical force
(F = i B L sin ). (i) What is the potential energy of
(i) So, the force experienced will be the pendulum at the position B ?
zero when the angle between the (ii) What is the total mechanical
magnetic field and the length of the energy at point C ?
conductor is 0°. (iii) What is the speed of the bob at
(ii) And the force experienced will be the position A when released
maximum when the angle between from B ?
the magnetic field and the length (Take g = 10 ms–2 and there is no
of conductor is 90°. loss of energy.)
(d) According to the question, a nucleus (b) (i) With reference to the direction of
202 of an element emits an alpha
84X action, how does a centripetal
particle followed by a beta particle force differ from a centrifugal
 emission force during uniform circular
202 198 + 42 He
84X  82Z motion ? 3
198  emission 198 + 0 e (ii) Is centrifugal force the force of
82Z  83Z 1 reaction of centripetal force ?
(iii) Compare the magnitudes of
 a  83 and b  198
centripetal and centrifugal force.
(e) (i) The magnetic polarity of the loop, 3
that faces us will be clockwise, i.e., (c) A block and tackle system of pulleys
south pole. has velocity ratio 4. 4
(ii) We can increase the strength of the (i) Draw a neat labelled diagram of the
magnetic field produced by the system indicating clearly the points
loop in the given diagram, by. of application and direction of load
1. increasing number of turns of and effort.
coil. (ii) What will be its V.R. if the weight of
2. increasing the current. the movable block is doubled ?
Ph 105 2020
Sol. (a) Consider the given diagram, reaction pair of forces acts on two
bodies, but here it is acting on a
single body.
(iii) The magnitudes of centripetal and
centrifugal force are equal.

(c) (i)

It is given that
Mass of simple pendulum, m = 200 g
= 0.2 kg. Potential energy at position A
is zero, i.e., PEA = 0.
(i) Potential energy at position B,
PEB = mgh = 0.2 × 10 × 5 = 10 J.
(ii) Whenever the bob of the pendulum
rotates, then mechanical energy
So, this is the required diagram.
remains same at all the points
according to the law of conser- (ii)  Velocity ratio is always equals
vation of mechanical energy. to n.
 Mechanical energy at point C So, if the weight of movable block
is doubled, then its mechanical
= 10 J.
advantage will get effected but the
(iii) As the bob is rotating, so the
velocity ratio will remain constant.
potential energy of bob at position
6. (a) A diver in water looks obliquely at
B will get converted into kinetic
an object AB in air. 3
energy at point A.
1
 KE at position A, K.EA  m 2
2
1
 10   0.2  2
2
 100 = 2

   10 ms1
(i) Does the object appear taller,
(b) (i) With reference to the direction of shorter or of the same size to the
motion, both centrifugal force and diver ?
centripetal force acts in opposite (ii) Show the path of two rays AC
direction. Centripetal force acts and AD starting from the tip of
towards the centre while centrifugal the object as it travels towards
force acts radially outwards. the diver in water and hence
(ii) No, the centrifugal force is not the obtain the image of the object.
force of reaction at centripetal (b) Complete the path of the ray AB
force. It is because the action- through the glass prism in PQR till it
Ph 106 2020
emerges out of the prism. Given the Hence, from the above diagram, it can
critical angle of the glass as 42°. 3 be said that the object will appear taller.
(b)

(c) A lens of focal length 20 cm forms an


inverted image at a distance 60 cm (c) (i) As per the information given in the
from the lens. 4 question, we can conclude that the
(i) Identify the lens. given lens is a convex lens because
(ii) How far is the lens present in the formed image is inverted.
front of the object’ ? (ii) Given, f = 20 cm
(iii) Calculate the magnification of  = 60 cm
the image. According to lens formula,
Sol. (a) Consider the given diagram, 1 1 1
A  
f  u
1 1 1
  
u  f
1 1 1
  
u 60 20
(i) and (ii) 1 2
 
As we know that when a ray of light u 60
enters from air to water, i.e., from rarer
to denser medium, it bends toward the  u  30 cm
normal. So, the above diagram will  60
become as follows : (iii) Magnification, m    2
u 30
7. (a) Give reasons for the following : 3
During the day :
(i) Clouds appear white.
(ii) Sky appears blue.
(b) (i) Name the system which enables
us to locate underwater objects
by transmitting ultrasonic waves
and detecting the reflecting
impulse. 3
Ph 107 2020
(ii) What are acoustically measurable (ii) Let the man is standing infront of
quantities related to pitch and cliff at a distance of d cm from the
loudness ? cliff.
(c) (i) When a tuning fork [vibrating] So, when he moves closer (after
is held close to ear, one hears a hearing echo) to the cliff by 84 m,
faint hum. The same [vibrating then the distance will be (d – 84) cm.
tuning fork] is held such that its  we know that
stem is in contact with the table
2d
surface, then one hears a loud V
sound. Explain. 4 t
Since velocity will remain same in
(ii) A man standing in front of a
both cases, therefore,
vertical cliff fires a gun. He hears
the echo after 3.5 seconds. On 2d 2 (d  84)

moving closer to the cliff by 84 t1 t2
m, he hears the echo after 3
seconds. Calculate the distance of { t1 = 3.5 s, t2 = 3 s}
the cliff from the initial position 2d 2 ( d  84)
of the man.  
3.5 3
Sol. (a) (i) As clouds scatter all the colours of
2d d  84
sunlight (i.e., VIBGYOR scatter) in  6d = 7d – 588
the sky during the day, therefore, 7 3
due to this reason, the clouds  d  588 m .
appear white.
(ii) Sky appears blue because when the 8. (a) The diagram below shows the core
light enters from the sun into the of a transformer and its input and
sky, the air layers scatter only blue output connections. 3
part of light more than any other
colour.
(b) (i) SONAR, i.e., Sound Navigation
and Ranging is a system which
enables us to locate under water
objects by transmitting ultrasonic
waves and detecting the reflecting (i) State the material used for the
impulse. core.
(ii) Frequency and Amplitude are (ii) Copy and complete the diagram
respectively measurable quantities of the transformer by drawing
related to pitch and loudness. input and output coils.
(c) (i) When a tuning fork is held close to (b) (i) What are superconductors ? 3
ear, it has some sound energy. But (ii) Calculate the current drawn by
when it is held such that its stem is an appliance rated 110 W, 220 V
in contact with the table surface, when connected across 220 V
then due to the larger surface area supply.
of table top, it produces more (iii) Name a substance whose resis-
vibrations with greater sound tance decreases with the increase
energy. in temperature.
Ph 108 2020
(c) (iii) We know that semiconductor’s
resistance decreases with the
increase in temperature.
For examples : Silicon, Graphite,
etc.
(c) Consider the given diagram.

4
The diagram above shows three
resistors connected across a cell of
e.m.f. 1.8 V and internal resistance r.
Calculate :
(i) Cur rent through 3  resistor.
(ii) The internal resistance r. (i) Since 3 and 1.5  resistor are in
parallel combination. Therefore the
Sol. (a) (i) Soft iron core are made of thin
potential difference across them
laminated sheets which are eight
will remain same.
(8) T or U shaped.
Let I 1 be the current passing
(ii) Since the input voltage is greater
through3  resistor and I2 be the
than the output voltage, therefore,
current passing through 1.5 
the given transformer is a step-
resistor.
down transformer. So, the given
diagram will become as follows : V = 3 I1 and V = 1.5 I2
{ V = IR}
 potential difference across them is
same,
 3 I1 = 1.5 I2
 I1 = 0.5 I2
I2
 I1   I2 = 2I1 ...(i)
(b) (i) The substances which have almost 2
zero resistance at low temperatures,  I1 + I2 = I { I = 0.3 A}
are called superconductors.
 I1 + I2 = 0.3 ...(ii)
(ii) Current drawn by the appliance,
From eqns. (i) and (ii), we get
Power  I1 + 2I1 = 0.3
I
Voltage  3I1 = 0.3
P  I1 = 0.1 A and I2 = 2 × 0.1 = 0.2 A
 I
V  I1 = 0.1 A and I2 = 0.2 A.
110 W 1 So, the current through 3  resistor
 I  A = 0.5 A. will be, I1 = 0.1 A.
220 V 2
(ii)  3  and 1.5  resistors are in parallel
 I  0.5 A combination,
Ph 109 2020
(Assume no heat is lost to the
Req1= 33 1.15
 Req 1.5 4.5

4.5
 1 surrounding)
33 1.5
1.5 4.54.5 Specific heat capacity of water
Now, Req1 and 4  are in series = 4.2 Jg–1 k–1
combination, therefore, their
Specific latent heat of fusion of ice
equivalent resistance,
= 336 Jg–1
Req2 = (1 + 4)  = 5 
Sol. (a) (i) The amount of heat which is to be
 Total resistance, Rtot = Req2 supplied to a given mass of a
+ Internal resistance material to produce a unit change
 Rtot = (5 + r)  ...(iii) in its temperature, is known as heat
V 1.8 capacity.
 Rtot    6 ...(iv) (ii) The SI unit of heat capacity is joule
I 0.3
per kelvin.
 from eqn. (iii) and (iv), we get
(iii)  Heat capacity = mass
5 + r = 6  r = 6 – 5 = 1
× specific heat capacity.
 r  1
 C  mS
9. (a) (i) Define heat capacity of a
So, this is the required relationship
substance. 3
between heat capacity and specific
(ii) Write the SI unit of heat capacity. heat capacity of a substance.
(iii) What is the relationship between (b) (i) In the given graph, AB represents
heat capacity and specific heat the specific heat of solid phase and
capacity of a substance ? CD represents the specific heat of
(b) The diagram below shows the change Liquid phase. Since, the specific
of phases of a substance on a temp- heat of solid phase is always greater
erature vs time graph on heating the than that of liquid phase. So, due
substance at a constant rate. 3 to this reason, the slope of CD is
less than that of AB.
(ii) t1 is the melting point because, here
solid is changing into liquid and t2
is the boiling point because, here
liquid is changing into gas.
(c) For Ice, Mass = 60 g
Temperature = 0°C
For water, Mass = 140 g
Temperature = 50°C.
(i) Why is the slope of CD less than
slope of AB ? Let T be the final temperature of water.
(ii) What is the boiling and melting Now, apply Principle of Calorimetry,
point of the substance ?  60 × 336 + 60 × 4.2 × T
(c) A piece of ice of mass 60 g is dropped = 140 × 4.2 (50 – T)
into 140 g of water at 50°C. Calculate  60 × 336 = 4.2 [140 × 50 – 140T – 60T]
the final temperature of water when 60  336
all the ice has melted. 4  = 7000 – 200 T
4.2
Ph 110 2020
60  336  10 (ii) D.C. motor is used to convert
 = 7000 – 200 T electrical energy into mechanical
42
energy, such as in electric fan etc.
 4800 = 7000 – 200 T
(b) (i)
 200 T = 7000 – 4800
2200 Nuclear Fusion Nuclear Fission
 T  T  11º C .
200 1. In this, smaller In this, a heavier nuclei
10. (a) (i) Draw a neat labelled diagram of nuclei combines to breaks into the smaller
a d.c. motor. 3 form a larger one. nuclei.
(ii) Write any one use of a d.c. motor. 2. Very high amount of Low temperature in
(b) (i) Differentiate between nuclear temperature is comparison to that in
fusion and nuclear fission. 3 obtained nuclear fusion is obtained
(ii) State one safety precaution in the
disposal of nuclear waste. (ii) The solid radioactive waste should
be sealed in red bag and placed in
(c) An atomic nucleus A is composed of
a cardboard radioactive waste box.
84 protons and 128 neutrons. The
nucleus A emits an alpha particle and (c) Nucleus A is composed of 84 protons
is transformed into a nucleus B. and 128 neutrons.
4  Mass number = Number of protons
(i) What is the composition of B ? + number of neutrons
(ii) The nucleus B emits a beta = 84 + 128
particle and is transformed into = 212
a nucleus C.  The composition of nucleus A will be
What is the composition of C ? 212 A
84
(iii) What is mass number of the
(i) Since, nucleus A emits an -
nucleus A ?
particle, i.e.,
(iv) Does the composition of C
change if it emits gamma  decay
radiations ?
212 A
84  208 B
82 + 24 He
Sol. (a) (i)  The composition of nucleus B
will be 208
82 B
(ii) Since nucleus B emits a -particle,
i.e.,
 decay
 208
208 B
82
0
83 C + 1e
 The composition of nucleus C will
be 208
83 C.
(iii) The mass number of the nucleus A
is 212.
(iv) If the nucleus C emits gamma
radiations, then there will be no
change in its composition.
ICSE QUESTION PAPER-2012 (SOLVED)

PAPER 2 (CHEMISTRY)
(One hour and A half)
Answer to this Paper must be written on the paper provided separately.
You will not be allowed to write during the first 15 minutes.
This time is to be spent in reading the Question Paper.
The time given at the head of this paper is the time allowed for writing the answers.

Section I is compulsory. Attempt any four questions from Section II.


The intended marks for questions or parts of questions are given in brackets [ ].

SECTION - I (40 MARKS)


(Attempt all questions from this Section)
Question 1.
(a) Name the gas in each of the following :
(i) The gas evolved on reaction of Aluminium with boiling concentrated caustic alkali
solution.
(ii) The gas produced when excess ammonia reacts with chlorine.
(iii) A gas which turns acidified potassium dichromate clear green.
(iv) The gas produced when copper reacts with concentrated nitric acid.
(v) The gas produced on reaction of dilute sulphuric acid with a metallic sulphide.[5]
(b) State one observation for each of the following :
(i) Excess ammonium hydroxide solution is added to lead nitrate solution.
(ii) Bromine vapours are passed into a solution of ethyne in carbon tetrachloride.
(iii) A zinc granule is added to copper sulphate solution.
(iv) Zinc nitrate crystals are strongly heated.
(v) Sodium hydroxide solution is added to ferric chloride solution at first a little and
then in excess. [5]
(c) Some word/words are missing in the following statements. You are required to rewrite
the statements in the correct form using the appropraite word/words :
(i) Ethyl alcohol is dehydrated by sulphuric acid at a temperature of about 170ºC.
(ii) Aqua regia contains one part by volume of nitric acid and three parts by volume of
hydrochloric acid.
(iii) Magnesium nitride reacts with water to liberate ammonia.
(iv) Cations migrate during electrolysis.
(v) Magnesium reacts with nitric acid to liberate hydrogen gas. [5]
(d) Choose the correct answer from the options given below :
Ch 111 2012
Ch 112 2012
(i) An element in period-3 whose electron affinity is zero.
(A) Neon (B) Sulphur
(C) Sodium (D) Argon
(ii) An alkaline earth metal.
(A) Potassium (B) Calcium
(C) Lead (D) Copper
(iii) The vapour density of carbon dioxide [C = 12, O = 16]
(A) 32 (B) 16
(C) 44 (D) 22
(iv) Identify the weak electrolyte from the following :
(A) Sodium Chloride solution (B) Dilute Hydrochloric acid
(C) Dilute Sulphuric acid (D) Aqueous acetic acid.
(v) Which of the following metallic oxides cannot be reduced by normal reducing
agents ?
(A) Magnesium oxide (B) Copper(II) oxide
(C) Zinc oxide (D) Iron(III) oxide [5]
(e) Match the following :
Column A Column B
1. Acid salt A. Ferrous ammonium sulphate
2. Double salt B. Contains only ions
3. Ammonium hydroxide solution C. Sodium hydrogen sulphate
4. Dilute hydrochloric acid D. Contains only molecules
5. Carbon tetrachloride E. Contains ions and molecules [5]
(f) Give the structural formula for the following :
(i) Methanoic acid (ii) Ethanal
(iii) Ethyne (iv) Acetone
(v) 2-methyl propane. [5]
(g) Concentrated nitric acid oxidises phosphorus to phosphoric acid according to the following
equation :
P + 5HNO3 (conc.) H3PO4 + H2O + 5NO2
If 9.3g of phosphorus was used in the reaction, calculate :
(i) Number of moles of phosphorus taken. [1]
(ii) The mass of phosphoric acid formed. [2]
(iii) The volume of nitrogen dioxide produced at STP.
[H = 1, N = 14, P = 31, O = 16] [2]
(h) Give reasons for the following :
(i) Iron is rendered passive with fuming nitric acid.
(ii) An aqueous solution of sodium chloride conducts electricity.
Ch 113 2012
(iii) Ionisation potential of the element increases across a period.
(iv) Alkali metals are good reducing agents.
(v) Hydrogen chloride gas cannot be dried over quick lime. [5]
Answer.
(a) (i) Hydrogen gas (ii) Nitrogen
(iii) Sulphur dioxide (iv) Nitrogen dioxide
(v) Hydrogen sulphide
(b) (i) White ppt. insoluble in excess ammonium hydroxide.
(ii) Ethyne decolorizes the colour of bromine solution.
(iii) the blue colour of copper sulphate solution discharge.
(iv) Reddish brown gas is evolved.
(v) Reddish brown precipitate is formed which in insoluble in excess of NaOH.
(c) (i) To give ethane and water
(ii) Concentrated nitric acid
Concentrated hydrochloric acid
(iii) Annomia gas
(iv) Cation migrates towards cathode during electrolysis
(v) Dilute nitric acid
(d) (i) (D) Argon (ii) (B) Calcium
(iii) (D) 22 (iv) (D) Aqueous acetic acid
(v) (A) Magnesium oxide
(e) Column A Column B
1. Acid salt C. Sodium hydrogen sulphate
2. Double salt A. Ferrous ammonium sulphate
3. Ammonium hydroxide solution E. Contains ions and molecules
4. Dilute hydrochloric acid B. Contains only ions
5. Carbon tetrachloride D. Contains only molecules
(f) (i) HCOOH (ii) CH3CHO
(iii) CH  CH (iv) CH3COCH3
(v) CH3 — CH — CH3

CH3
(g) Given, P + 5HNO3 (conc.) H3PO4 + H2O + 5NO2
9 .3
(i) Number of moles of phosphorous taken = = 0.3 mole
31
(ii) 1 mole of phosphorous gives = 98gm of phosphoric acid
So, 0.3 mole of phosphorous gives = (0.3 × 98gm) of phosphoric acid
Ch 114 2012
= 29.4gm of phosphoric acid
(iii) 1 moles of phosphoric gives = 112L of NO2 gas at STP
So, 0.3 moles of phosphorous gives = (112 × 0.3)L of NO2 gas at STP
= 33.6L of NO2 gas at STP
(h) (i) Due to the formation of oxide film on its surface.
(ii) Due to the presence of free Na+ and Cl– ion.
(iii) Because the atomic radius decreases across a period. Due to this, attraction between the
nucleus and electron increases, this result in increase in the ionization potential.
(iv) Alkali metals are good reducing agents because alkali metals have one valence electron
which they lose to attain stability. Hence, they themselves undergo oxidation causing
reduction of others and are good reducing agents.
(v) Because HCl undergo chemical reaction with quick lime.
CaO + 2HCl CaCl2 + H2O

SECTION - II (40 MARKS)


(Attempt any four questions from this Section)
Question 2.
(a) Some properties of sulphuric acid are listed below. Choose the role played by sulphuric
acid as A, B, C, or D which is responsible for the reactions (i) to (v).
Some role/s may be repeated.
A. Dilute acid B. Dehydrating agent
C. Non-volatile acid (d) Oxidising agent
(i) CuSO45H2O conc.
 H 2SO 4
 CuSO4 + 5H2O
(ii) S + H2SO4 (conc.) 3SO2 + 2H2O
(iii) NaNO3 + H2SO4 (conc.) 
200º C

 NaHSO4 + HCl
(iv) MgO + H2SO4 MgSO4 + H2O
(v) Zn + 2H2SO4 (conc.) ZnSO4 + SO2 + 2H2O [5]
(b) Give balanced equations for the following reactions :
(i) Dilute nitric acid and Copper carbonate.
(ii) Concentrated hydrochloric acid and Potassium permanganate solution.
(iii) Ammonia and Oxygen in the presence of a catalyst.
(iv) Silver nitrate solution and Sodium chloride solution.
(v) Zinc sulphide and Dilute sulphuric acid. [5]
Answer.
(a) (i) B : Dehydrating agent (ii) D : Oxidising agent
(iii) C : Non-volatile acid (iv) A : Dilute acid
(v) D : Oxidising agent
Ch 115 2012
(b) (i) CuCO3 + dil. 2HNO3 Cu (NO3)2 + H2O = CO2
(ii) 16 HCl (conc.) + 2KMnO4 
 2KCl + 2MnCl2 + 5Cl2 + 8H2O

Pt 5 Pt
(iii) 4NH3 + 5O2 4NO + 6H2O 2NH3 + O 2NO + 3H2O + 
800ºC 2 2 800ºC

(iv) AgNO3 (aq) + NaCl (aq) AgCl  + NaNO3 (aq)


(v) ZnS + H2 SO4 (dil.) ZnSO4 + H2S
Question 3.
(a) Select the correct answer from the list given in brackets :
(i) An aqueous electrolyte consists of the ions mentioned in the list, the ion which
could be discharged most readily during electrolysis. [Fe2+, Cu2+, Pb2+, H+].
(ii) The metallic electrode which does not take part in an electrolytic reaction.
[Cu, Ag, Pt, Ni].
(iii) The ion which is discharged at the anode during the electrolysis of copper sulphate
solutions using copper electrodes as anode and cathode. [Cu2+, OH–, SO42–, H+].
(iv) When dilute sodium chloride is electrolysed using graphite electrodes, the cation
is discharged at the cathode most readily. [Na+, OH–, H+, Cl–].
(v) During silver plating of an article using potassium argentocyanide as an electrolyte,
the anode material should be [Cu, Ag, Pt, Fe]. [5]
(b) Match the properties and uses of alloys in List 1 with the appropriate answer from List 2.
List 1 List 2
1. The alloy contains Cu and Zn, is A. Duralumin
hard, silvery and is used in decorative
articles.
2. It is stronger than Aluminium, B. Brass
light and is used in making light tools.
3. It is lustrous, hard, corrosion resistant C. Bronze
and used in surgical instruments.
4. Tin lowers the melting point of the D. Stainless stell
alloy and is used for soldering purpose.
5. The alloy is hard, brittle, takes up E. Solder
polish and is used for making statues. [5]
Answer.
(a) (i) Cu2+ (ii) Pt
2+
(iii) Cu (iv) H+
(v) Ag
(b) List 1 List 2
1. The alloy contains Cu and Zn, is B. Brass
Ch 116 2012
hard, silvery and is used in decorative
articles.
2. It is stronger than Aluminium, A. Duralumin
light and is used in making light tools.
3. It is lustrous, hard, corrosion resistant D. Stainless steel
and used in surgical instruments.
4. Tin lowers the melting point of the E. Solder
alloy and is used for soldering purpose.
5. The alloy is hard, brittle, takes up C. Bronze
polish and is used for making statues.
Question 4.
(a) Identify the anion present in the following compounds :
(i) Compound X on heating with copper turnings and concentrated sulphuric acid
liberates a reddish brown gas.
(ii) When a solution of compound Y is treated with silver nitrate solution a white
precipitate is obtained which is soluble in excess of ammonium hydroxide solution.
(iii) Compound Z which on reacting with dilute sulphuric acid liberates a gas which
turns lime waer milky, but the gas has no effect on acidified potassium dichromate
solution.
(iv) Compound L on reacting with Barium chloride solution gives a white precipitate
insoluble in dilute hydrochloric acid or dilute nitric acid. [4]
(b) State one chemical test between each of the following pairs :
(i) Sodium carbonate and Sodium sulphite
(ii) Ferrous nitrate and Lead nitrate
(iii) Manganese dioxide and Copper(II) oxide [3]
(c) Draw an electron dot diagram to show the structure of hydronium ion. State the type of
bonding present in it. [3]
Answer.
(a) (i) Nitrate ion, NO3– (ii) Chloride ion, Cl–
(iii) Carbonate ion, CO32– (iv) sulphate ion, SO42–
(b) (i) Sodium carbonate and Sodium sulphate :
Add HCl (dil.) to both the compounds and pass the gas liberated through acidified potassium
dichromate solution.

Na2CO3 + 2HCl 2NaCl + H2O + CO2 


(No reaction in potassium dichromate solution)

Na2SO3 + 2HCl 2NaCl + H2O + SO2 


Ch 117 2012
(Gas liberated will turn potassium dichromate solution green)
(ii) Ferrous nitrate and Lead nitrate :
When ammonium hydroxide is added to both the solution, the following reactions take
place.
Fe(NO3)2 + 2NH4OH Fe(OH)2  + 2NH4NO3
(Dirty green precipitates)
Pb (NO3)2 + 2NH4OH Pb (OH)2  + 2NH4NO3
(Chalky white precipitates)
Thus, ferrous nitrate will give dirty green precipitates whereas lead nitrate will give chalky
white precipitates.
(iii) Manganese dioixde and Copper (II) oxide :
To both the compounds add conc. HCl and heat, the following observations helps :


MnO2 + 4HCl   MnCl2 + 2H2O + Cl2 
(conc.)
(Greenish yellow gas is liberated)


CuO + HCl   CuCl2 + H2O
(conc.) (Greenish blue solution)
(No gas is liberated but the solution turns bluish)
H
+
(c) H O
H
It has coordinate (dative covalent) bonding
Question 5.
(a) (i) 67.2 litres of hydrogen combines with 44.8 litres of nitrogen to form ammonia
under specific conditions as :
N2(g) + 3H2(g) 2NH3(g)
Calculate the volume of ammonia produced. What is the other substance, if any,
that remains in the resultant mixture ? [2]
3
(ii) The mass of 5.6 dm of a certain gas at STP is 12.0 g. Calculate the relative molecular
mass of the gas. [2]
(iii) Find the total percentage of Magnesium in magnesium nitrate crystals,
Mg(NO3)2.6H2O.
[Mg = 24; N = 14; O = 16 and H = 1] [2]
(b) Refer to the flow chart diagram below and give balanced equations with conditions, if
any, for the following conversions A to D.
Ch 118 2012

B Iron(II) chloride

Sodium Hydrogen C Ammonium


Chloride Chloride chloride
A
D
Lead chloride

[4]
Answer.
(a) Acc. to the equation
N2(g) + 3H2(g) 2NH3(g)
(i) One mole of nitrogen combine with 3 moles of hydrogen to give 2 moles of ammonia.
Therefore, 3 × 22.4 litre of hydrogen require = 22.4 L of Nitrogen
Since, Nitrogen is present in excess amount so it remains in the resultant
Mixture and volume of ammonia produced = 2 × 22.4 L = 44.8 L
(ii) 5.6 dm3 of gas weighs = 12.0 g

 12.0 
1 dm3 of gas weighs =   gm
 5.6 

 12.0 
22.4 dm3 of gas weighs =   22.4  gm = 48 gm
 5 .6 
Therefore, relative molecular mass of gas = 48 gm
(iii) Molar mass of Mg(NO3)26H2O
= 24 + (14 × 2) + (16 × 6) + (1 × 12) + 6 × 16 = 256 g

24
Mass percent of Magnesium = × 100 = 9.37%
256

(b) [A] NaCl + H2SO4 (conc.) below 200C NaHSO4 + HCl



Sodium chloride Hydrogen chloride

[B] 2HCl (dil.) + Fe Fe Cl2 + H2 


Hydrogen chloride Iron (II) chloride
[C] HCl + NH3 NH4Cl + H 2O
Hydrogen chloride Ammonium chloride
[D] Pb(NO3)2 + 2HCl PbCl2 + 2HNO3
Ch 119 2012
Question 6.
(a) Name the following metals :
(i) A metal present in cryolite other than sodium.
(ii) A metal which is unaffected by dilute or concentrated acids.
(iii) A metal present in period 3, group 1 of the periodic table. [3]
(b) The following questions are relevant to the extraction of Aluminium :
(i) State the reason for addition of caustic alkali to bauxite ore during purification of
bauxite.
(ii) Give a balanced chemical equation for the above reaction.
(iii) Along with cryolite and alumina, another substance is added to the electrolyte
mixture. Name the substance and give one reason for the addition. [3]
(c) The following questions are based on the preparation of ammonia gas in the laboratory:
(i) Explain why ammonium nitrate is not used in the preparation of ammonia.
(ii) Name the compound normally used as a drying agent during the process.
(iii) How is ammonia gas collected ?
(iv) Explain why it is not collected over water. [4]
Answer.
(a) (i) Aluminium (ii) Gold
(iii) Sodium
(b) (i) Caustic alkali is added to bauxite ore during extraction as the insoluble part of the bauxite
is removed and the alumina component is then precipitated.
(ii) The reaction is as follows :
Al 2O32H2O + 2NaOH 2NaAlO2 + 3H2O
(iii) Fluorspar is added along with cryolite and alumina because this helps the mixture to fuse
at 950ºC instead of 2050ºC and the aluminium obtained at this temperature is liquid.
(c) (i) Ammonium nitrate does not undergo a reversible sublimation reaction, it melts and then
decompses into nitrogen oxide gas and water vapour. Thus it is not used in the preparation
of ammonia.
NH4NO3 N2O + 2H2O
(ii) Calcium oxide
(iii) Ammonia is collected in an inverted dry gas jar by downward displacement of air.
(iv) It is highly soluble in water and hence cannot be collected by downward displacement of
water.
Question 7.
(a) From the following organic compounds given below, choose one compound in each case
which relates to the description (i) to (iv) :
[Ethyne, ethanol, acetic acid, ethene, methane]
(i) An unsaturated hydrocarbon used for welding purposes.
(ii) An organic compound whose functional group is carboxyl.
Ch 120 2012
(iii)A hydrocarbon which on catalytic hydrogennation gives a saturated hydrocarbon.
(iv) An organic compound used as a thermometric liquid. [4]
(b) (i) Why is pure acetic acid known as glacial acetic acid ?
(ii) Give a chemical equation for the reaction between ethyl alcohol and acetic acid.
[2]
(c) There are three elements E, F, G with atomic numbers 19, 8 and 17 respectively.
(i) Classify the elements as metals and non-metals. [3]
(ii) Give the molecular formula of the compound formed between E and G and state
the type of chemical bond in this compound. [1]
Answer.
(a) (i) Ethyne (ii) Acetic acid
(iii) Ethene (iv) Ethanol
(b) (i) Pure acetic acid is called glacial acetic acid because at a temperature below 16.5°C is
solidified as icy mass which floats at the surface of acetic acid as a glacier.
(ii) C2H5OH + CH3COOH CH3COOC2H5 + H2O
Ethyl alcohol Acetic acid Ethyl propanoate
(c) (i) E = Metal, F and G = Non-Metal
(ii) The Molecular formula is EG
The bonding is electrovalent bond.
ICSE QUESTION PAPER-2013 (SOLVED)

PAPER 2 (CHEMISTRY)
(One hour and A half)
Answer to this Paper must be written on the paper provided separately.
You will not be allowed to write during the first 15 minutes.
This time is to be spent in reading the Question Paper.
The time given at the head of this paper is the time allowed for writing the answers.

Section I is compulsory. Attempt any four questions from Section II.


The intended marks for questions or parts of questions are given in brackets [ ].

Section I (40 Marks)


Attempt all questions from this Section.
Question 1
(a) From the list given below, select the word (s) required to correctly complete
blanks (i) to (v) in the following passage. The words from the list are to be used
only once. Write the answers as (a) (i), (ii), (iii) and so on. Do not copy the passage.
Ans. (ammonia, ammonium, carbonate, carbon dioxide, hydrogen, hydronium, hydroxide,
precipitate, salt, water) :
(a) A solultion M tuns blue litmus red, so it must contain (i) ............ ions ; another
solution O turns red litmus blue and hence, must contain (ii) ......ions.
(b) When solution M and O are mixed together, the products will be (iii) ............ and
(iv) ............
(c) If a piece of magnesium was put into a solution M, (v) ............ gas would be evolved.(5)
Ans. (a) (i) hydronium, (ii) hydroxide
(b) (iii) salt, (iv) water
(c) (v) hydrogen
(b) Identify the gas evolved in the following reactions when :
(i) sodium propionate is heated with soda lime.
(ii) potassium sulphite is treated with dilute hydrochloric acid.
(iii) sulphur is treated with concentrated nitric acid.
(iv) a few crystals of KNO3 are heated in a hard glass test tube.
(v) concentrated hydrochloric acid is made to react with manganese dioxide. (5)
Ans. (i) Ethane gas (ii) Sulphur dioxide gas
(iii) Nitrogen dioxide gas (iv) Oxygen gas
Ch 121 2013
Ch 122 2013
(v) Chlorine gas
(c) State one appropriate observation for each of the following:
(i) Concentrated sulphuric acid is added drop wise to a crystal of hydrated copper
sulphate.
(ii) Copper sulphide is treated with dilute hydrochloric acid.
(iii) Excess of chlorine gas is reacted with ammonia gas.
(iv) A few drops of dilute hydrochloric acid are added to silver nitrate solution, followed
by addition of ammonium hydroxide solution.
(v) Electricity is passed through molten lead bromide. (5)
Ans. (i) The blue coloured hydrated copper sulphate crystals disintegrate with a hissing sound,
giving off steam and leaving behind white residue.
(ii) A colourless gas with a smell of rotten eggs is given off and a green coloured solution
is formed.
(iii) When chlorine gas is in excess as compared to ammonia gas, the ammonia reduces
chlorine to hydrochloric acid.
(iv) A curdy white precipitate is formed. This precipitate dissolves in excess of ammonium
hydroxide to form a colourless solution.
(v) The molten lead bromide breaks into lead metal which discharges at cathode and bromine
gas which discharged at anode.
(d) Give suitable chemical terms for the following :
(i) A bond formed by a shared pair of electrons with both electrons coming from the
same atom.
(ii) A salt formed by incomplete neutralisation of an acid by a base.
(iii) A reaction in which hydrogen of an alkane is replaced by a halogen.
(iv) A definite number of water molecules bound to some salts.
(v) The process in which a substance absorbs moisture from the atmosphere air to
become moist, and ultimately dissolves in the absorbed water.

Ans. (i) Coordinate bond (ii) Acidic salt


(iii) Subsitution reaction (iv) Water of crystallisation (v) Deliquescence
(e) Give a chemical test to distinguish between the following pairs of compounds :
(i) Sodium chloride solution and sodium nitrate solution.
Ans. Add silver nitrate solution to sodium chloride solution and sodium nitrate solution. In case
of sodium chloride, a curdy white precipitate is formed. In case of sodium nitrate solution
the reaction mixture remains colourless.
(ii) Hydrogen chloride gas and hydrogen sulphide gas.
Ch 123 2013
Ans. Moist lead acetate paper turns black in case of hydrogen sulphide gas, but does not
change its colour in case of hydrogen chloride gas.
(iii) Ethene gas and ethane gas.
Ans. To the given gas add few drops of bromine solution in carbon, tetra-chloride. In case of
ethene gas, the reddish colour of bromine discharges. However, in case of ethane gas the
reddish colour of bromine does not discharge.
(iv) Calcium nitrate solution and zinc nitrate solution.
Ans. To each of the solution add first ammonium hydroxide solution in small amount and then
in excess. In case of calcium nitrate a fine white precipitate is formed, which does not
dissolve in excess of sodium hydroxide. In case of zinc nitrate a gelatin like white
precipitate is formed which dissolves in excess of sodium hydroxide.
(v) Carbon dioxide gas and sulphur dioxide gas.
Ans. To each of the gas add few drops of acidified potassium dichromate solution. In case of
carbon dioxide no change takes place. In case of sulphur dioxide, potassium dichromate
solution turns blue.
(f) Choose the most appropriate answer from the following options :
(i) Among the period 2 elements, the element which has high electron affinity is
(a) Lithium (b) Carbon (c) Chlorine (d) Fluorine
Ans. (d) Fluorine has highest electron affinity
(ii) Among the following compounds identify the compound that has all three bonds
(ionic, covalent and coordinate bond).
(a) Ammonia (b) Ammonium chloride (c)Sodium hydroxide (d) Calcium chloride
Ans. (b) Ammonium chloride has all the three electronic bonds
(iii) Identify the statement that is incorrect about alkanes :
(a) They are hydrocarbons.
(b) There is single covalent bond between carbon and hydrogen
(c) They can undergo both substitution as well as addition reactions
(d) On complete combustion they produce carbon dioxide and water.
Ans. (c) They can undergo both substitution as well as addition reactions.
(iv) Which of these will act as non-electrolyte ?
(a) Liquid carbon tetrachloride (b) Acetic acid
(c) Sodium hydroxide aqueous solution acid. (d) Potassium chloride aqu. solution.
Ans. (a) Liquid carbon tetrachloride is non-electrolyte
(v) Which one of the following will not produce an acid when made to react with
water ?
(a) Carbon monoxide (b) Carbon dioxide (c)Nitrogen dioxide (d) Sulphuric trioxide.
Ans. (a) Carbon monoxide gas
Ch 124 2013
(vi) Identify the metallic oxide which is amphoteric in nature :
(a) Calcium oxide (b) Barium oxide (c) Zinc oxide (d) Copper (II) oxide
Ans. (c) Zinc oxide is an amphoteric oxide
(vii) In the given equation identify the role played by concentrated sulphuric acid
S + 2H2SO4 
 3SO2 + 2H2O :
(a) Non-volatile acid (b) Oxidising agent (c)Dehydrating agent (d) none of these
Ans. (b) Concentrated sulphuric acid acts as oxidising agent
(viii) Nitrogen gas can be obtained by heating :
(a)Ammonium nitrate (b) Ammonium nitrite (c) Magnesium nitric (d) Ammonium chloride
Ans. (b) Ammonium nitrite
(ix) Which of the following is not a typical property of an ionic compound ?
(a) High melting point
(b) Conducts electricity in the molten and in the aqueous solution state
(c) They are insoluble in water
(d) They exist as oppositely charged ions even in the solid state.
Ans. (c) Ionic compounds are generally insoluble in water
(x) The metals zinc and tin are present in the alloy :
(a) Solder (b) Brass (c) Bronze (d) Duralumin. (10)
Ans. Bronze (c)
(g) Solve the following :
(i) What volume of oxygen is required to burn completely 90 dm 3 of butane under
similar conditions of temperature and pressure ? 2C4H10 + 13O2 
 8CO2 + 10H2O

Ans. 2C4H10 + 13O2 


 8CO2 + 10H2O
2Vols. 13Vols. 8 Vols. Nil (By Gay Lussaic’s law)
13
1 Vol Vols.
2
13
90 dm3 × 90 dm3 = 585 dm3 (2)
2
(ii) The vapour density of a gas is 8. What would be the volume occupied by 24.0
g of the gas at STP ? (2)
Ans. V.D. of gas = 8
 Molecular mass of gas = 2 × V.D. = 2 × 8 = 16
 Gram molecular mass of gas = 16 g
Ch 125 2013
16 g of gas at STP occupies = 22.4 dm3
22.4  24
 24 g of gas at STP occupies = = 33.6 dm3.
16
(iii) A vessel contains X number of molecules of hydrogen gas at a certain temperature
and pressure. How many molecules of nitrogen gas would be present in the same
vessel under the same conditions of temperature and pressure ? (1)
Ans. According to Avogadro’s law, equal volumes of all gases under similar conditions of
temperature and pressure contain equal number of molecules.
Number of molecules of nitrogen = X.
SECTION II (40 Marks)

Attempt any four questions from this Section

Question 2
(a)
Group IA IIA IIIA IVA VA VIA VIIA 0
number 1 2 13 14 15 16 17 18
2nd period Li D O J Ne
A Mg E Si H M
R T I Q U Y
* In this table H does not represent hydrogen.
* Some elements are given in their own symbol and position in the periodic table.
* While others are shown with a letter.
With reference to the table answer the following questions.
(i) Identify the most electronegative element. (1)
(ii) Identify the most reactive element of group 1. (1)
(iii) Identify the element from period 3 with least atomic size. (1)
(iv) How many valence electrons are present in Q ? (1)
(v) Which element from group 2 would have the least ionization energy ? (1)
(vi) Identify the noble gas of the fourth period. (1)
(vii) In the compound between A and H what type of bond would be formed and give
the molecular formula for the same. (2)
Ans. (i) J (ii) R (iii) M (iv) 5 (v) T (vi) Y
(vii) A and H form an electrovalent compound and its formula is A2H.
(b) Compare the compounds carbon tetrachloride and sodium chloride with regard to
solubility in water and electrical conductivity. (2)
Ans. Carbon tetrachloride is insoluble in water and is not conducting in nature.
Ch 126 2013
Sodium chloride is soluble in water and is conducting in nature in aqueous state or molten
state.
Question 3
(a) Choosing the substances from the list given below, write balanced chemical
equations for the reactions which would be used in the laboratory to obtain the
following salts :
Dilute Sulphuric acid Copper Copper (II) carbonate
Iron Sodium carbonate
Sodium Sodium chloride
Zinc nitrate
(i) Sodium sulphate (ii) Zinc carbonate
(iii) Copper (II) sulphate (iv) Iron (II) sulphate. (4)
(b) State two relevant observations for each of the following :
(i) Ammonium hydroxide solution is added to copper (II) nitrate solution in small
quantities and then in excess.
(ii) Ammonium hydroxide solution is added to zinc nitrate solution in minimum
quantities and then in excess.
(iii) Lead nitrate crystals are heated in a hard glass test tube.
Ans. (a) (i) Sodium carbonate and dilute sulphuric acid.
Na2CO3 + H2SO4(dil)  Na2SO4 + CO2 + H2O
(ii) Zinc nitrate and sodium carbonate.
Zn(NO3)2(aq) + Na2CO3(aq)  ZnCO3(s) + 2NaNO3(aq)
(iii) Copper carbonate and dilute sulphuric acid.
CuCO3(s) + H2SO4(aq)  CuSO4(aq) + H2O(l) + CO2(g)
(iv) Iron and dilute sulphuric acid.
Fe(s) + H2SO4(aq)  FeSO4(aq) + H2(g)
(b) (i) With small amount of ammonium hydroxide, a bluish white precipitate is formed. This
precipitate dissolves in excess of ammonium hydroxide to form a deep blue solution.
(ii) A white gelatin like precipitate is formed which dissolves in excess of ammonium hydroxide.
(iii) It gives off a reddish brown gas (NO2). The crystals crumble to form a powdery mass,
which is yellow when hot and white when cold.
Question 4
(a) Copper sulphate solution is electrolysed using copper electrodes.
Study the diagram given below and answer the question that follows:
Ch 127 2013

(i) Which electrode to your left or right is known as the oxidising electrode and
why? (2)
Ans. Left (anode ; + ve terminal of the battery). It is called the
oxidising electrode as anions give their electrons to anode,
and thus undergo oxidation.
(ii) Write the equation representing the reaction that occurs. (1)

Ans. At cathode (–ve terminal of battery ; rich in e )
Cu2+ + 2e–  Cu
At anode (+ ve terminal of battery ; deficient in e–)
Cu – 2e–  Cu2+
(iii) State two appropriate observations for the above electrolysis reaction. (2)
Ans. The size of anode gradually decreases and that of cathode gradually increases. However,
there is no change in the colour of copper sulphate solution.
(b)
X Y
Normal Electronic Configuration 2, 8, 7 2, 8, 2
Nature of oxide Dissolves in water and Very low solubility in water.
turns blue litmus red Dissolves in hydrochloric acid
Tendency for oxidising Tends to oxidise elements Tends to act as a reducing
and reducing reactions and compounds agent
Electrical and Thermal Very poor electrical Good Electrical conductor
conductivity conductor; Poor thermal Good Thermal conductor
conductivity
Tendency to form alloys and No tendency to form Forms alloys
amalgums alloys
Using the information above, complete the following :
(i) ............. is the metallic element.
(ii) Metal atoms tend to have a maximum of ............. electrons in the outermost
energy level.
(iii) Non-metallic elements tend to form ............. oxides while metals tend to form
............. oxides.
Ch 128 2013
(iv) Non-metallic elements tend to be ............. conductors of heat and electricity.
(v) Metals tend to ............. electrons and act as ............. agents in their reactions
with elements and compounds. (5)
Ans. (i) Y, (ii) Three (iii) acidic, basic (iv) bad (v) lose, reducing
Question 5
a. Give balanced equations for each of the following :
(i) Reduction of hot Copper (II) oxide to copper using ammonia gas.
Ans. (i) 3CuO + 2NH3  3Cu + 3H2O + N2 (4)
(ii) Oxidation of carbon with concentrated nitric acid.
Ans. C + 4HNO3  2H2O + 4NO2 + CO2
(iii) Dehydration of concentrated sulphuric acid with sugar crystals
Ans. C12H22O11 + 11H2SO4  12C + 11H2O
(b) Copy and complete the following table relating to important industrial process :
Name of the process Temperature Catalyst Equation for the
catalyzed reaction (3)
Haber’s process
Ans.
Temperature Catalyst Equation for catalysed reaction
450o C
450o C Iron containing N2 + 3H2 2NH3 + 22.4 KCal
Fe + Mo
molybdenum
(c) The following questions relate to the extraction of aluminium by electrolysis :
(i) Name the other aluminium containing compound added to alumina and state its
significance.
Ans. The compound is cryolite [Na3AlF6]
The addition of cryolite lowers the melting point of alumina from 2050oC to 950 oC.
Furthermore, it increases the electrical conductivity of the molten alumina.
(ii) Give the equation for the reaction that takes place at the cathode.
Ans. Al3+ + 3e–  Al
(iii) Explain why is it necessary to renew the anode periodically.
Ans. The anode (which is made of carbon) is attacked by nascent oxygen formed due to the
discharge of O2– ions and changes to carbon dioxide. As the anode is gradually consumed,
it is periodically renewed.
Question 6
(a) Give balanced equations for the laboratory preparations of the following organic
compounds : (4)
(i) A saturated hydrocarbon from iodomethane.
Ch 129 2013

Ans. CH3I + 2H (from Zn/Cu couple)  CH4 + HI


(ii) An unsaturated hydrocarbon from an alcohol.
Ans. C2H5OH + H2SO4 (conc) 160 C
 C2H4 + H2O
(iii) An unsaturated hydrocarbon from calcium carbide.
Ans. CaC2 + 2H2O   Ca(OH)2 + C2H2
(iv) An alcohol from ethyl bromide.
Ans. C2H5Br + KOH (aq)   C2H5OH + KBr
(b) Give the structural formulae for the following : (3)
(i) An isomer of n-butane. (ii) 2-propanol.

H H H H OH H
| | | |3 2| 1|
Ans. H — C — C — C — H [Iso-butane] H — C — C — C — H [2-propanol]
| | | | | |
HH—C — H H H H — H
|
H
(iii) Diethyl ether.
Ans.
H H O H H
| | || | |
H—C — C — C — C —C—H [Di-ethyl ether]
| | | |
H H H H

(c) Give reasons for the following :


(i) Methane does not undergo addition reactions, but ethene does.
Ans. All the four covalent bonds between the carbon and hydrogen are fully shared. Thus the
hydrogen atom can only be substituted by more reactive atoms or group of atoms. There
is no scope of addition of reactive atoms in its molecule.
However, in case of ethene there is a double bond between the two carbon atoms. These
bonds are under strain and hence can be easily broken by more reactive atoms to form
other compounds which are saturated in nature.
(ii) Ethyne is more reactive than ethane.
Ans. Ethyne has a triple covalent bond (—C  C —) between two carbon atoms, whereas
ethane has a single covalent bond (—C = C —) between the two carbon atoms. So, the
strain in the bonding of ethyne is far more than ethane. This accounts of the reactivity
of ethyne as its bonds break more easily than that of ethane.
(iii) Hydrocarbons are excellent fuels. (3)
Ans. All the constituents of hydrocarbon (carbon and hydrogen) are highly combustible and do
Ch 130 2013
not have any uncombustible content. So, hydrocarbons are excellent fuels.
Question 7
(a) O2 is evolved by heating KClO3 using MnO2 as a catalyst
2KClO3 MnO
2  2KCl + 3O2
Ans. (a) 2KClO3 MnO
2  2KCl + 3O2 (5)
2[39+35.5+48] g 2[39+35.5]g 3[22.4] lt
245 g 67.2 lt
(i) Calculate the mass of KClO3 required to produce 6.72 litre of O2 at STP. [atomic
masses of K = 39, Cl = 35.5, O = 16] (2)
Ans. 67.2 lt of oxygen at STP is librated from potassium chlorate = 245 g
245  6.72
 6.72 lt of oxygen at STP is liberated from potassium chlorate = = 24.5 g
67.2
(ii) Calculate the number of moles of oxygen present in the above volume and also
the number of molecules. (2)
Ans. 22.4 lt of oxygen and STP = 1 mole.
6.72
 6.72 lt of oxygen at STP = = 0.3 moles
22.4
1 mole of oxygen contains number of molecules = 6 × 1023

6  10 23  0.3
 0.3 mole of oxygen contain number of molecules = = 1.8 × 1023 molecules
1
(iii) Calculate the volume occupied by 0.01 mole of CO2 at STP. (1)
Ans. 1 mole of carbon dioxide gas at STP occupies = 22.4 lt
22.4  0.01
 0.01 mole of carbon dioxide gas at STP occupies = = 0.224 lt = 224 cm3
1
(b) Identify the following substances which are underlined :
(i) An alkaline gas which produces dense white fumes when reacted with hydrogen
chloride gas. Ans. Ammonia gas.
(ii) An acid which is present in vinegar. Ans. Acetic acid or ethanoic acid.
(iii) A gas which does not conduct electricity in the liquid state but conducts electricity
when dissolved in water. Ans. Hydrogen chloride gas.
(iv) A dilute mineral acid which forms a white precipitate when treated with barium
chloride solution. Ans. Dilute sulphuric acid.
(v) The element which has the highest ionization potential.
Ans. Fluorine.
ICSE QUESTION PAPER-2014 (SOLVED)

PAPER 2 (CHEMISTRY)
(Two hours)
Answers to this Paper must be written on the paper provided separately.
You will not be allowed to write during the first 15 minutes.
This time is to be spent in reading the Question Paper.
The time given at the head of this paper is the time allowed for writing the answers.

Section I is compulsory. Attempt any four questions from Section II.


The intended marks for questions or parts of questions are given in brackets [ ].

SECTION - I (40 MARKS)


(Attempt all questions from this Section)
Question 1.
(a) Choose the correct answer from the options given below:
(i) Ionisation Potential increases over a period from left to right because the:
(A) Atomic radius increases and nuclear charge increases
(B) Atomic radius decreases and nuclear charge decreases
(C) Atomic radius increases and nuclear charge decreases
(D) Atomic radius decreases and nuclear charge increases.
(ii) A compound X consists of only molecules. Hence X will have:
(A) A crystalline hard structure
(B) A low melting point and low boiling point
(C) An ionic bond
(D) A strong force of attraction between its molecules.
(iii) When fused lead bromide is electrolysed we observe:
(A) a silver grey deposit at anode and a reddish brown deposit at cathode
(B) a silver grey deposit at cathode and a reddish brown deposit at anode
(C) a silver grey deposit at cathode and reddish brown fumes at anode
(D) silver grey fumes at anode and reddish brown fumes at cathode.
(iv) The main ore used for the extraction of iron is:
(A) Haematite (B) Calamine
(C) Bauxite (D) Cryolite
(v) Heating an ore in a limited supply of air or in the absence of air at a temperature
just below its melting point is known as:
(A) smelting (B) ore dressing

Ch 131 2014


Ch 132 2014

(C) calcination (D) bessemerisation


(vi) If an element A belongs to Period 3 and Group II then it will have,
(A) 3 shells and 2 valence electrons (B) 2 shells and 3 valence electrons
(C) 3 shells and 3 valence electrons (D) 2 shells and 2 valence electrons
(vii) The molecule containing a triple co-valent bond is:
(A) ammonia (B) methane
(C) water (D) nitrogen
(viii) The electrolyte used for electroplating an article with silver is:
(A) silver nitrate solution (B) silver cyanide solution
(C) sodium argentocyanide solution (D) nickel sulphate solution
(ix) Aluminium powder is used in thermite welding because,
(A) it is a strong reducing agent (B) it is a strong oxidising agent
(C) it is corrosion resistant (D) it is a good conductor of heat.
(x) The I.U.P.A.C. name of acetylene is,
(A) propane (B) propyne
(C) ethene (D) ethyne. [10]
(b) Fill in the blanks from the choices given within brackets:
(i) The basicity of Acetic Acid is (3, 1, 4)
(ii) The compound formed when ethanol reacts with sodium is (sodium
ethanoate, sodium ethoxide, sodium propanoate)
(iii) Quicklime is not used to dry HCl gas because (CaO is alkaline, CaO is
acidic, CaO is neutral)
(iv) Ammonia gas is collected by (an upward displacement of air, a downward
displacement of water, a downward displacement of air)
(v) Cold, dilute nitric acid reacts with copper to form (Hydrogen, nitrogen
dioxide, nitric oxide). [5]
(c) Give one word or phrase for the following:
(i) The ratio of the mass of a certain volume of gas to the mass of an equal volume of
hydrogen under the same conditions of temperature and pressure.
(ii) Formation of ions from molecules.
(iii) Electrolytic deposition of a superior metal on a baser metal.
O
(iv) Hydrocarbons containing a C functional group.
(v) The amount of energy released when an atom in the gaseous state accepts an
electron to form an anion. [5]
(d) Match the options A to E with the statements (i) to (v):
A alkynes (i) No. of molecules in 22.4 dm3 of carbon dioxide at
s.t.p.
Ch 133 2014
B alkane (ii) An element with electronic configuration 2, 8,
8, 3
C iron (iii) Cn H2n + 2
D 6.023 × 1023 (iv) Cn H2n – 2
E metal (v) The metal that forms two types of ions [5]
(e) Write balanced equations for the following:
(i) Action of heat on a mixture of copper and concentrated nitric acid.
(ii) Action of warm water on magnesium nitride.
(iii) Action of concentrated sulphuric acid on carbon.
(iv) Action of dilute hydrochloric acid on sodium sulphide.
(v) Preparation of ethane from sodium propionate. [5]
(f) Distinguish between the following pairs of compounds using the test given within
brackets:
(i) Iron (II) sulphate and iron (III) sulphate (using ammonium hydroxide)
(ii) A lead salt and a zinc salt (using excess ammonium hydroxide)
(iii) Sodium nitrate and sodium sulphite (using dilute sulphuric acid)
(iv) Dilute sulphuric acid and dilute hydrochloric acid (using barium chloride solution)
(v) Ethane and ethene (using alkaline potassium permanganate solution) [5]
(g) (i) Oxygen oxidises ethyne to carbon dioxide and water as shown by the equation:
2C2H2 + 5O2  4CO2 + 2H2O
What volume of ethyne gas at s.t.p. is required to produce 8.4 dm 3 of carbon
dioxide at s.t.p.? [H = 1, C = 12, O = 16]
(ii) A compound made up of two elements X and Y has an empirical formula X2Y. If
the atomic weight of X is 10 and that of Y is 5 and the compound has a vapour
density 25, find its molecular formula. [5]
Answer.
(a) (i) (D) Atomic radius decreases and nuclear charge increases.
(ii) (B) A low melting point and low boiling point
(iii) (C) a silver grey deposit at cathode and reddish brown fumes at anode
(iv) (A) Haematite (v) (C) calcination
(vi) (A) 3 shells and 2 valence electrons (vii) (D) nitrogen
(viii)(C) sodium argentocyanide solution (ix) (A) it is a strong reducing agent
(x) (D) ethyne
(b) (i) The basicity of Acetic Acid is 1.
(ii) The compound formed when ethanol reacts with sodium is sodium ethoxide.
(iii) Quicklime is not used to dry HCl gas because CaO is alkaline.
(iv) Ammonia gas is collected by a downward displacement of air.
(v) Cold, dilute nitric acid reacts with copper to form nitric oxide.
Ch 134 2014
(c) (i) The ratio of the mass of a certain volume of gas to the mass of an equal volume of hydrogen
under the same conditions of temperature and pressure Relative vapour density..
(ii) Formation of ions from molecules Ionisation.
(iii) Electrolytic deposition of a superior metal on a baser metal Electroplating.
O
(iv) Hydrocarbons containing a C functional group Ketones.
(v) The amount of energy released when an atom in the gaseous state accepts an electron to
form an anion Electron Affinity..
(d) A alkynes (iv) Cn H2n – 2
B alkane (iii) Cn H2n + 2
C iron (v) The metal that forms two types of ions
23
D 6.023 × 10 (i) No. of molecules in 22.4 dm3 of carbon dioxide at s.t.p.
E metal (ii) An element with electronic configuration 2, 8, 8, 3
(e) (i) Action of heat on a mixture of copper and concentrated nitric acid.
Cu + 4HNO3 Cu(NO3)2 + 2H2O + 2NO2
(ii) Action of warm water on magnesium nitride.
Mg3N2 + 6H2O 3Mg(OH)2 + 2NH3
(iii) Action of concentrated sulphuric acid on carbon.
C + 2H2SO4 CO2 + 2H2O + 2SO2
(iv) Action of dilute hydrochloric acid on sodium sulphide.
Na2S + 2HCl 2NaCl + H2S
(v) Preparation of ethane from sodium propionate.
C2H5 COONa + NaO H C2H6 + Na2CO3
(f) (i) Iron (II) sulphate and iron (III) sulphate (using NH4OH)
Iron (II) sulphate (Fe2+ ion)

FeSO4 + 2NH4OH Fe(OH)2  + (NH4)2 SO4


green dirty green ppt. colourless sol.
3+
Iron (III) Salt (Fe ion)

Fe2(SO4)3 + 6NH4OH 2Fe(OH)3  + 3(NH4)2 SO4


reddish brown ppt.
(ii) A lead salt and a zinc salt (using excess NH4OH)

Pb(NO3)2 + 2NH4OH Pb(OH)2  + 2NH4NO3


Chalky white ppt.
Insoluble in excess of NH4OH

ZnSO4 + 2NH4OH Zn(OH)2  + (NH4)2 SO4


White gelatinous ppt.
Ch 135 2014
With excess of NH4OH ppt. dissolves
Zn(OH)2 + (NH4)2SO4 + 2NH4OH [Zn(NH3)4] SO4 + 4H2O
Tetra amine zinc sulphate
(iii) Sodium nitrate and sodium sulphate (using dil. H2SO4)
NaNO3 + H2SO4 (dil.) 
200º C

 NaHSO4 + HNO3

2NaNO3 + H2SO4 


200º C

 Na2SO4 + 2HNO3

Na2SO3 + H2SO4 Na2SO4 + H2O + SO2 


Sod. sulphite give SO2 gas but NaNO3 does not
(iv) Dilute sulphite acid and dilute hydrochloric acid (using barium chloride solution)

BaCl2 + H2SO4 BaSO4  + 2HCl


White ppt.
BaCl2 + HCl No ppt. formation
(v) Ethane and ethene (using alkaline KMnO4)
Ethene decolourises the colour of alkaline KMnO4 but ethane does not
(g) (i) 2C2H2 + 5O2  4CO2 + 2H2O
2 vol. 5 vol. 4 vol. 2 vol.
4 vol. of O2 required ethyne = 2 vol.
2  8 .4
8.4 dm3 vol. of O2 required ethyne = = 4.2 dm3
4
(ii) Empirical formula = X2Y
Empirical mass = 2 × 10 + 5 = 25
25 = n × 25 (V.D.) Moleculer mass = 2 × V.D. = 2 × 25 = 50

25 Molecules Mass 50
n= =1  2
25 Emperical Mass 25
Molecular formula = n × Empirical formula
2 × X2 Y = X 4 Y2

SECTION - II (40 MARKS)


(Attempt any four questions from this Section)
Question 2.
(a) State your observation in each of the following cases:
(i) When dilute hydrochloric acid is added to sodium carbonate crystals.
(ii) When excess sodium hydroxide is added to calcium nitrate solution.
(iii) At the cathode when acidified aqueous copper sulphate solution is electrolyzed
with copper electrodes.
Ch 136 2014
(iv) When calcium hydroxide is heated with ammonium chloride crystals.
(v) When moist starch iodide paper is introduced into chlorine gas. [5]
(b) Study the figure given below and answer the questions that follow:

Spray
Gay Y

Water

Dropper

Water +
Blue Litmus

(i) Identify the gas Y.


(ii) What property of gas Y does this experiment demonstrate?
(iii) Name another gas which has the same property and can be demonstrated through
this experiment. [3]
(c) (i) Name the other ion formed when ammonia dissolves in water.
(ii) Give one test that can be used to detect the presence of the ion produced. [2]
Answer.
(a) (i) Brisk effervescence of a gas which turns lime water milky.
(ii) A chalky white ppt insoluble in excess sodium hydroxide.
(iii) Reddish brown deposit.
(iv) Gas with a pungent odour evolved.
(v) The paper turns blue black.
(b) (i) Identify the gas Y — It is HCl (Hydrogen chloride)
(ii) Property of Y — Highly solubility in water
(iii) The another gas has same property — NH3 (Ammonia)
(c) (i) When Ammonia dissolves in water.
NH3 + H2O NH4OH
NH4OH NH4+ + OH–
ions formed are ammonium and hydroxyl.
(ii) Add copper sulphate solution - pale blue ppt formed or ferrous sulphate solution - dirty
green ppt formed
Ch 137 2014
Question 3.
(a) State the conditions required for the following reactions to take place:
(i) Catalytic hydrogenation of ethyne.
(ii) Preparation of ethyne from ethylene dibromide.
(iii) Catalytic oxidation of ammonia to nitric oxide.
(iv) Any two conditions for the conversion of sulphur dioxide to sulphur trioxide. [5]
(b) State the main components of the following alloys:
(i) Brass. (ii) Duralumin.
(iii) Bronze. [3]
(c) Give balanced equations for the following:
(i) Laboratory preparation of nitric acid.
(ii) Preparation of ethanol from monochloroethane and aq. sodium hydroxide. [2]
Answer.
(a) (i) Catalytic hydrogenation of ethyne
Ni
C2H2 + H2  C2H4 
Ni
C 2 H6
(200 C)

Ethyne convert into ethene and then to ethane and nickel act as catalyst.
(ii) Preparation of ethyne from ethylene dibromide.
KOH
CH2 – CH2  CH  CH + 2KBr + 2H2O
(alc.)

Br Br
(iii) Catalytic oxidation of ammonia to nitric oxide.

4NH3 + 5O2 Pt.


800º
C
 6H2O + 4NO 
(iv) Two condition for the conversion of SO2 to SO3
Catalyst V205,
Temp. (450 – 500)°C,
Pressure (1 – 2) atms.
(b) Give the main components of following alloys:
(i) Brass — 60 – 70% Cu
40 – 30% Zn
(ii) Duralium — 95% Al
4% Cu
0.5% Mg
0.5% Mn
(iii) Bronze — 80% Cu
18% Sn
2% Zn
Ch 138 2014
(c) (i) Laboratory preparation of nitric acid.
KNO3 + H2SO4 
200º C

 KHSO4 + HNO3

NaNO3 + H2SO4 


200º C

 NaHSO4 + HNO3
(ii) Preparation of ethanol from monochloro ethane and aq. sodium hydroxide.
CH3CH2Cl + NaOH CH3CH2OH + NaCl
Question 4.
(a) Give the structural formula of the following:
(i) ethanol. (ii) 1-propanal.
(iii) ethanoic acid. (iv) 1, 2, dichloroethane. [4]
(b) Draw the structure of the stable positive ion formed when an acid dissolves in water. [2]
(c) State the inference drawn from the following observations:
(i) On carrying out the flame test with a salt P a brick red flame was obtained. What
is the cation in P?
(ii) A gas Q turns moist lead acetate paper silvery black. Identify the gas Q.
(iii) pH of liquid R is 10. What kind of substance is R?
(iv) Salt S is prepared by reacting dilute sulphuric acid with copper oxide. Identify S.
[4]
Answer.
(a) Give the structural formula:
H H

(i) Ethanol H C C OH
H H
H H O

(ii) 1-propanal H C C C H
H H

H O

(iii) Ethanoic acid H C C OH


H

H H

(iv) 1, 2, dichloroethane H C C H
Cl Cl
+ –
(b) HCl + H2O H3O + Cl

H O H
H
Ch 139 2014
(c) (i) Cation in P is Ca2+.
(ii) The gas Q is H2S.
(iii) The substance R is alkaline.
(iv) The compound or salt S is copper sulphate CuSO4
CuO + H2SO4 CuSO4 + H2O
Question 5.
(a) Name the following:
(i) The property possessed by metals by which they can be beaten into sheets.
(ii) A compound added to lower the fusion temperature of electrolytic bath in the
extraction of aluminium.
(iii) The ore of zinc containing its sulphide. [3]
(b) Give one equation each to show the following properties of sulphuric acid:
(i) Dehydrating property. (ii) Acidic nature.
(iii) As a non-volatile acid. [3]
(c) Give balanced chemical equations to prepare the following salts:
(i) Lead sulphate from lead carbonate.
(ii) Sodium sulphate using dilute sulphuric acid.
(iii) Copper chloride using copper carbonate. [4]
Answer.
(a) (i) Mellability (ii) Cryolite (Na3AlF6)
(iii) The ore of zinc containing sulphide ZnS (Zinc Blende).
(b) (i) Dehydrating property.
HCOOH Conc.
 H 2SO 4
 CO + H2O
(ii) Acidic nature.
CuO + H2SO4 CuSO4 + H2O
(iii) As a non-volatile acid.
NaCl + H2SO4 NaHSO4 + HCl
(c) (i) Lead sulphate from lead carbonate.
PbCO3 + 2HNO3 Pb(NO3)2 + H2O + CO2
Pb(NO3)2 + H2SO4 PbSO4 + 2HNO3
(ii) Sodium sulphate using dilute sulphuric acid.
2NaCl + H2SO4 
200º C

 Na2SO4 + HCl
(iii) Copper chloride using copper carbonate.
CuCO3 + 2HCl CuCl2 + H2O + CO2
Question 6.
(a) (i) State Avogadro’s Law.
(ii) A cylinder contains 68g of ammonia gas at s.t.p.
Ch 140 2014
1. What is the volume occupied by this gas?
2. How many moles of ammonia are present in the cylinder?
3. How many molecules of ammonia are present in the cylinder?
[N-14, H-1] [4]
(b) (i) Why do covalent compounds exist as gases, liquids or soft solids?
(ii) Which electrode: anode or cathode is the oxidising electrode? Why? [3]
(c) Name the kind of particles present in:
(i) Sodium Hydroxide solution.
(ii) Carbonic acid.
(iii) Sugar solution. [3]
Answer.
(a) (i) Avogadro’s Law states that “equal vol. of all gases under similar conditions of temperature
and pressure contain the same no. of molecules.”
(ii) 1. Molecular gram atom wt. of NH3 = 17 gm
17 gm of NH3 has vol. at s.t.p. = 22.4 lt.
22.4  68
68 gm of NH3 has vol. at s.t.p. = = 89.6 lt.
17

68
2. No. of moles in 68 gm of NH3 = = 4 moles.
17
3. No. of molecules of ammonia = 4 × 6.023 × 1023 = 24.092 × 1023
(b) (i) The molecules are held together with weak vander wall’s forces. As these forces are
weak, so they are gases, liquids or soft solids.
(ii) Anode - It takes up the electrons from the anions.
(c) (i) Sodium hydroxide sol.  only ions (ii) Carbonic acid  ions and molecules
(iii) Sugar solution  only molecules
Question 7.
(a) An element Z has atomic number 16. Answer the following questions on Z:
(i) State the period and group to which Z belongs.
(ii) Is Z a metal or a non-metal?
(iii) State the formula between Z and Hydrogen.
(iv) What kind of a compound is this? [4]
(b) M is a metal above hydrogen in the activity series and its oxide has the formula M2O.
This oxide when dissolved in water forms the corresponding hydroxide which is a good
conductor of electricity. In the above context answer the following:
(i) What kind of combination exists between M and O?
(ii) How many electrons are there in the outermost shell of M?
(iii) Name the group of which M belongs.
Ch 141 2014
(iv) State the reaction taking place at the cathode.
(v) Name the product at the anode. [5]
Answer.
(a) An element Z has atomic number 16
(i) Period – 3 group – 6 (as E.C. 2, 8, 6)
(ii) Z is a non-metal.
(iii) Formula between Z and hydrogen is H2Z.
(iv) The kind of compound is covalent.
(b) (i) Electrovalent combination (ii) One
(iii) Alkali metals
(iv) Reaction of cathode: M+ + e– M
(v) Oxygen / O2
ICSE QUESTION PAPER-2015 (SOLVED)

PAPER 2 (CHEMISTRY)
(Two hours)
Answers to this Paper must be written on the paper provided separately.
You will not be allowed to write during the first 15 minutes.
This time is to be spent in reading the Question Paper.
The time given at the head of this paper is the time allowed for writing the answers.

Section I is compulsory. Attempt any four questions from Section II.


The intended marks for questions or parts of questions are given in brackets [ ].

SECTION - I (40 MARKS)


(Attempt all questions from this Section)
Question 1.
(a) Select from the list the gas that matches the description given in each case:
[ammonia, ethane, hydrogen chloride, hydrogen sulphide, ethyne]
(i) This gas is used as a reducing agent in reducing copper oxide to copper.
(ii) This gas produces dense white fumes with ammonia gas.
(iii) This gas is used for welding purposes.
(iv) This gas is also a saturated hydrocarbon.
(v) This gas has a characteristic rotten egg smell. [5]
Ans.
(i) Ammonia (ii) Hydrogen chloride
(iii) Ethyne (iv) Ethane (v) Hydrogen sulphide
(b) Choose the most appropriate answer for each of the following:
(i) Among the elements given below, the element with the least electronegativity is:
(A) Lithium (B) Carbon
(C) Boron (D) Fluorine
(ii) Identify the statement which does not describe the property of alkenes:
(A) They are unsaturated hydrocarbons
(B) They decolourise bromine water
(C) They can undergo addition as well as substitution reactions
(D) They undergo combustion with oxygen forming carbon dioxide and water.
(iii) This is not an alloy of copper:
(A) Brass (B) Bronze

Ch 142 2015


Ch 143 2015
(C) Solder (D) Duralumin.
(iv) Bonding in this molecule can be understood to involve coordinate bonding.
(A) Carbon tetrachloride (B) Hydrogen
(C) Hydrogen chloride (D) Ammonium chloride
(v) Which of the following would weigh the least?
(A) 2 gram atoms of Nitrogen. (B) 1mole of Silver
(C) 22.4 litres of oxygen gas at 1 atmospheric pressure and 273K
(D) 6.02 . 1023 atoms of carbon.
[Atomic masses: Ag=108, N=14, O=16, C=12] [5]
Ans.
(i) (A) Lithium
Lithium is an element with the least electronegativity.
(ii) (C) They can undergo addition and substitution reactions.
Alkenes do not undergo substitution reaction.
(iii) (C) Solder
Solder is an alloy of lead and tin.
(iv) (D) Ammonium chloride
The bond formed between the nitrogen atom in ammonia and the chloride ion is a coordinate
bond.
(v) (D) 6.02 . 1023 atoms of carbon.
(c) Complete the following calculations. Show working for complete credit:
(i) Calculate the mass of Calcium that will contain the same number of atom as are present
in 3.2 gm of Sulphur. [Atomic masses: S=32, Ca=40] [2]
(ii) If 6 litres of hydrogen and 4 litres of chlorine are mixed and exploded and if water is
added to the gases formed, find the volume of the residual gas. [2]
(iii) If the empirical formula of a compound is CH and it has a vapour density of 13, find the
molecular formula of the compound. [1]
Ans.
(i) Given:
Mass of Sulphur = 3.2 gm
Solution:
32 g of S  6.022 × 1023 atoms
3.2 g of S  ?
6.022  10 23  3.2
3.2 g of S will contain = = 6.02 × 1022 atom
32

40 g of Ca  6.02 × 1023 atoms


Ch 144 2015

? = 6.02 × 1022 atoms

40  6.02  10 23
Mass of Ca = = 4g
6.02  10 22
Mass of calcium = 4 g
(ii) 6 litres of hydrogen and 4 litres of chlorine are mixed which results in the formation of 8 litres
of HCl gas. When water is added, it results in the formation of hydrochloric acid. So, the
amount of gas left is only 2 litres of hydrogen as chlorine acts as a limiting reagent.
Therefore, the volume of the residual gas will be 2 litres.
(iii) Given:
Empirical formula = CH
Vapour density = 13
Molecular weight = 2 × Vapour density
= 2 × 13 = 26

 Empirical formula of a compound with molecular mass 26 is CH.

Molecular mass 26 26
n = Empirical formula  (12  1)  13 = 2

 Molecular formula of the given compound is 2 × (CH) = (CH)2 = C2H2.


(d) State one relevant observation for each of the following:
(i) When crystals of copper nitrate are heated in a test tube.
(ii) When the gaseous product obtained by dehydration of ethyl alcohol is passed through
bromine water.
(iii) When hydrogen sulphide gas is passed through lead acetate solution.
(iv) When ammonia gas is burnt in an atmosphere of excess oxygen.
(v) At the Anode when aqueous copper sulphate solution is electrolysed using copper elec-
trodes. [5]
Ans.
(i) Reddish brown nitrogen dioxide gas is released and the residue left behind is black copper oxide.
(ii) The reddish brown colour of bromine solution gets decolourised.
(iii) When hydrogen sulphide gas is passed through lead acetate solution, it forms a black precipitate
of lead sulphide.
(iv) Ammonia gas burns to form nitrogen gas and steamy fumes of water.
Also it burns with greenish yellow flame.
(v) The anode slowly dissolves, but the colour of copper sulphate does not change.
(e) Identify the acid which matches the following description (i) to (v):
Ch 145 2015
(i) The acid which is used in the preparation of a non-volatile acid.
(ii) The acid which produces sugar charcoal from sugar.
(iii) The acid which is prepared by catalytic oxidation of ammonia.
(iv) The acid on mixing with lead nitrate solution produces a white precipitate which is
insoluble even on heating.
(v) The acid on mixing with silver nitrate solution produces a white precipitate which is
soluble in excess ammonium hydroxide. [5]
Ans. (i) Concentrated Nitric Acid
(ii) Conc. sulphuric acid
(iii) Conc. Nitric acid
(iv) Dilute sulphuric acid
(v) Dilute hydrochloric acid
(f) Give appropriate scientific reasons for the following statements:
(i) Zinc oxide can be reduced to zinc by using carbon monoxide, but aluminium oxide cannot
be reduced by a reducing agent
(ii) Carbon tetrachloride does not conduct electricity.
(iii) During electrolysis of molten lead bromide graphite anode is preferred to other elec-
trodes.
(iv) The electrical conductivity of acetic acid is less in comparison to the electrical conductiv-
ity of dilute sulphuric acid at a given concentration.
(v) Electrolysis is of molten lead bromide is considered to be a redox reaction. [5]
Ans.
(i) The metals in the middle of the activity series like zinc are moderately reactive, and carbon is a
good reducing agent because of which zinc oxide gets easily reduced by carbon. Oxides of
highly active metals like aluminium have great affinity towards oxygen and so cannot be
reduced by carbon.
(Note: Error in the question. Zinc oxide can be reduced to zinc metal by using carbon, but
aluminium oxide cannot be reduced by a reducing agent.)
(ii) Carbon tetrachloride is made of individual covalently bonded molecules, CCl4. In addition, the
charged particles are absent in CCl4 which could conduct electricity. So, CCl4 does not conduct
electricity.
(iii) During the electrolysis of molten lead bromide, a graphite anode is preferred because graphite
remains unaffected by the reactive bromine vapours which are released at the anode.
(iv) Sulphuric acid is a strong acid compared to acetic acid. A strong acid has more ions than a
weak one, and so, its solution will be a better electrical conductor than a weak acid. So,
electrical conductivity of acetic acid is less in comparison of electric conductivity of sulphuric
acid.
(v) In the electrolysis of molten lead bromide, the following reactions take place:
At the cathode: Pb2+ (l) + 2e-  Pb(l)
Ch 146 2015
At the anode: 2Br- (l)  Br2 (g) + 2e-
Lead (II) ions (Pb2+) are attracted to the negative electrode, and the Pb2+ ions are forced to
accept two electrons. Pb2+ ions are reduced. Bromide ions (Br-) are attracted to the positive
electrode and the bromide ions are forced to give away their extra electron to form bromine
atoms. Thus, bromide ions are oxidised. So, electrolysis of molten lead bromide is a redox
reaction.
(g)
(i) Give balanced chemical equations for the following conversions A, B and C:
A B C
Fe  FeCl3  FeCO3   Fe(NO3)2 [3]
(ii) Differentiate between the terms strong electrolyte and weak electrolyte. [2]
(stating any two differences)
Ans.
(i) A: 2Fe + 3Cl2 Heat
 2FeCl3
2FeCl3 + 3Na2CO3 
 Fe2 (CO3) + 6NaCl
C: FeCO3 + 2HNO3 
 Fe (NO3)2 + H2O + CO2
(ii)
Strong Electrolytes Weak Electrolytes
Electrolytes which allow a large Electrolytes which allow a small
amount of current to flow through amount of current to flow through
them. them.
The solution of a strong electrolyte The solution of a weak electrolyte
contains only free mobile ions. contains ions and molecules.
(h) Answer the following questions:
(i) Explain the bonding in methane molecule using electron dot structure. [2]
(ii) The metal of Group 2 from top to bottom are Be, Mg, Ca, Sr, and Ba.
(1) Which one of these elements will form ions most readily and why?
(2) State the common feature in the electronic configuration of all these elements. [3]
Ans. (i) Formation of methane molecule – Non-polar covalent compound:
Atom Electronic Nearest To attain stable electronic
configuration noble gas configuration of nearest noble
gas
12
Carbon 6
C [2,4] Neon [2,8] Carbon needs four electrons to
complete the octet
1
Hydrogen 1
H [1] Helium [2] Hydrogen needs one electron to
complete the duplet
One atom of carbon shares four electron pairs, one with each of the four atoms of hydrogen.
Ch 147 2015

Before combination (4 [H] and 1 After combination (CH4 – Methane


[C] atom) molecule)
(ii)
(1) In group 2, the atomic size increases down the group. As the atomic size increases, the nuclear
charge decreases. Due to this, electrons of the outermost shell lie further away from the
nucleus making the removal of electrons easy. So, Ba will form ions readily.
(2) All the elements have 2 electrons in their valence shell.

SECTION II (40 Marks)


Attempt any four questions from this Section
Question 2
(a) Arrange the following as per the instructions given in the brackets:
(i) Cs, Na, Li, K, Rb (increasing order of metallic character).
(ii) Mg, Cl, Na, S, Si (decreasing order of atomic size).
(iii) Na, K, Cl, S, Si (increasing order ionization energy)
(iv) Cl, F, Br, I (increasing order of electron affinity) [4]
Ans.
(i) Li < Na < K < Rb < Cs
(ii) Na > Mg > Si > S > Cl
(iii) K < Na < Si < S < Cl
(iv) I < Br < F < Cl
(b) Choose the most appropriate answer from the following list of oxides which fit the
description. Each answer may be used only once:
[SO2, SiO2, Al2O3, MgO, CO, Na2O]
(i) A basic oxide.
(ii) An oxide which dissolves in water forming an acid.
(iii) An amphoteric oxide.
(iv) A covalent oxide of a metalloid. [4]
Ans. (i) MgO (ii) SO2
Ch 148 2015
(iii) Al2O3 (iv) SiO2
(c) Element X is a metal with a valency 2, Y is 3 non-metal with a valency 3.
(i) Write an equation to show how Y from an ion.
(ii) If Y is a diatomic gas, write an equation for the direct combination of X and Y to from a
compound. [2]
Ans.
(i) Y will form an anion by gaining 3 electrons.
–3
The equation is given as Y + 3e–  Y .
(ii) The equation for the direct combination of X and Y to form a compound is
3X + Y2 
 X3Y2
Question 3
(a) Give balanced chemical equations for the following conversions:
(i) Ethanoic acid to ethyl ethanoate.
(ii) Calcium carbide to ethyne.
(iii) Sodium ethanoate to methane. [3]
Ans.
(i) Ethanoic acid to ethyl ethanoate
Conc. H2SO4
CH3COOH + C2H5OH CH3COOC2H5 + H2O
Warm
Ethyl ethanoate
(ii) CaC2 + 2 H2 O 
 Ca(OH)2 + CH  CH
Calcium carbide Calcium hydroxide
CaO
(iii) CH3COONa + NaOH o Na2CO3 + CH4
300 C
Methane
(b) Using their structural formulae identify the functional group by circling them:
(i) Dimethyl ether. (ii) Propanone.

Ans. (i) Dimethyl ether

(ii) Propanone
Ch 149 2015
(c) Name the following:
(i) Process by which ethane is obtained from ethene.
(ii) A hydrocarbon which contributes towards the greenhouse effect.
(iii) Distinctive reaction that takes place when ethanol is treated with acetic acid.
(iv) The property of element by virtue of which atoms of the element can link to each other
in the form of a long chain or ring structure.
(v) Reaction when an alkyl halide is treated with alcoholic potassium hydroxide. [5]
Ans. (i) Hydrogenation (ii) Methane
(iii) Esterification (iv) Catenation
(v) Dehydrohalogenation
Question 4
(a) Identify the anion present in each of the following compounds:
(i) A salt M on treatment with concentrated sulphuric acid produces a gas which fumes in
moist air and gives dense fumes with ammonia.
(ii) A salt D on treatment with dilute sulphuric acid produces a gas which turns lime water
milky but has no effect on acidified potassium dichromate solution.
(iii) When barium chloride solution is added to salt solution E a white precipitate insoluble in
dilute hydrochloric acid is obtained. [3]
Ans. (i) Chloride ion (Cl.) (ii) Carbonate ion (CO32.)
(iii) Sulphate ion (SO42.)
(b) The following table shows the tests a student performed on four different aqueous
solutions which are X, Y, Z and W. Based on the observations provided, identify the
cation present: [4]
Chemical test Observation Conclusion
To solution X, ammonium A dirty white precipitate is (i)
hydroxide is added in minimum formed which dissolves in
quantity first and then in excess. excess to form a clear solution
To solution Y ammonium A pale blue precipitate is (ii)
hydroxide is added in minimum formed which dissolves in
quantity first and then in excess. excess to form a clear inky blue
solution.
To solution W a small quantity of A white precipitate is formed (iii)
sodium hydroxide solution is which remains insoluble.
added and then in excess.
To a salt Z calcium hydroxide A pungent smelling gas turning (iv)
solution is added and then moist red litmus paper blue is
heated. obtained.
Ch 150 2015
Ans. (i) Zn2+ (ii) Cu2+
(iii) Ca2+ (iv) NH4+
(c) Give balanced chemical equations for each of the following:
(i) Lab preparation of ammonia using an ammonium salt
(ii) Reaction of ammonia with excess chlorine.
(iii) Reaction of ammonia with sulphuric acid. [3]
Ans.
(i) Ammonia is prepared in the laboratory by using ammonium chloride.
2NH4Cl + Ca (OH)2 CaO
 CaCl2 + 2H2O + 2NH3
Ammonium chloride Calcium chloride Ammonia gas
(ii) When ammonia reacts with excess of chlorine, it forms nitrogen trichloride and HCl.
NH3 + 3Cl2 
 NCl3 + 3HCl
(Nitrogen trichloride)
(iii) Ammonia reacts with sulphuric acid to form ammonium sulphate.
2NH3 + H2SO4 
 (NH4)2SO4
Ammonium sulphate
Question 5
(a) Consider the following reaction and based on the reaction answer the questions that
follow:
(NH 4 ) 2 Cr2O 7 heat
 N 2 (g)  4H 2 O(g)  Cr2 O 3
Calculate:
(i) the quantity in moles of (NH4)2Cr2O7 if 63gm of(NH4)2Cr2O7 is heated. [1]
(ii) the quantity in moles of nitrogen formed. [1]
(iii) the volume in litres or dm3 of N2 evolved at S.T.P. [1]
(iv) the mass in grams of Cr2O3 formed at the same time. [2]
(Atomic masses: H=1, Cr= 52, N=14]
Ans. The given reaction is as follows:
(NH 4 ) 2 Cr2 O 7 heat
 N 2 (g)  4H 2 O(g)  Cr2 O 3
(i) Given:
Weight of (NH4)2Cr2O7 = 63 gm
Molar mass of (NH4)2Cr2O7
= (2 × 14) + (8 × 1) + (2 × 52) + (7 × 16)
= 28 + 8 + 104 + 112 = 252 gm
1 mole (NH4)2Cr2O7 = 252 gm
63
Hence, 63 gm of (NH4)2Cr2O7 = = 0.25 moles
252
The quantity of moles of (NH4)2Cr2O7 if 63 gm of (NH4)2Cr2O7 is heated is 0.25 moles.
(ii) From the given chemical equation, 1 mole of (NH4)2Cr2O7 produces 1 mole of nitrogen gas.
Ch 151 2015
Hence, 0.25 moles of (NH4)2Cr2O7 can produce 0.25 moles of nitrogen gas.
The quantity in moles of nitrogen formed is 0.25 moles.
(iii) One mole of an ideal gas at S.T.P. occupies 22.4 litres or dm3.
Hence, 0.25 moles of (NH4)2Cr2O7 will occupy 0.25 × 22.4 = 5.6 litres or dm3.
The volume in litres or dm3 of N2 evolved at S.T.P. is 5.6 litres or dm3.
(iv) From the given chemical equation, 1 mole of (NH4)2Cr2O7 produces 1 mole of Cr2O3.
Hence, 0.25 moles of (NH4)2Cr2O7 will produce 0.25 moles of Cr2O3.
Molar mass of Cr2O3
= (2 × 52) + (3 × 16)
= 104 + 48 = 152 gm
1 mole Cr2O3 = 152 gm
Hence, 0.25 moles of Cr2O3 = 0.25 × 152 = 38 gm
The mass in grams of Cr2O3 formed at the same time is 38 gm.
(b) (i) For each of the substance listed below, describe the role played in the extraction of
aluminium.
(1) Cryolite (2) Sodium hydroxide (3) Graphite [3]m 6
(ii) Explain why :
(1) In the electrolysis of alumina using the Hall Heroult’s Process the electrolyte is covered
with powdered coke.
(2) Iron sheets are coated with zinc during galvanization. [2]
Ans. (i) In the extraction of aluminium, the given compounds play the following roles:
(1) Cryolite: It lowers the fusion temperature from 2050°C to 950°C and enhances conductivity.
(2) Sodium hydroxide: Two roles are played by sodium hydroxide in the extraction of aluminium.
First, finely grinded bauxite (ore of aluminium) is heated under pressure with conc. caustic soda
solution (NaOH solution) for 2–8 hours at 140°C to 150°C to produce sodium aluminate. The
chemical equation is as follows:
Al2O3   2H2O + 2NaOH   2NaAlO2 + 3H2O
Second, on diluting sodium aluminate with water and cooling to 50°C, sodium aluminate is
hydrolysed to give aluminium hydroxide as precipitate.
Here, the impurities dissolve in sodium hydroxide.
(3) Graphite: Thick rods of graphite are suspended into the fused electrolyte.
They act as an anode where oxygen gas is discharged.
(ii) (1) In the electrolysis of alumina using the Hall–Héroult process, the electrolyte is covered with
powdered coke as it reduces heat loss by radiation prevents the burning of the anode
(2) Iron sheets are coated with zinc during galvanisation to prevent them from rusting.
Question 6
(a) (i) Give balanced chemical equations for the action of sulphuric acid on each of the
following:
(1) Potassium hydrogen carbonate. (2) Sulphur.
(ii) In the contact process for the manufacture of sulphuric acid give the equations for the
conversion of sulphur trioxide to sulphuric acid. [2]
Ch 152 2015
Ans. (1) Action of sulphuric acid on potassium hydrogen carbonate
2KHCO3 + H2SO4   K2SO4 + 2H2O + 2CO2 
(2) Action of sulphuric acid on sulphur
S + 2H2SO4   3SO2 + 2H2O
(ii) In the contact process for the manufacture of sulphuric acid, the equations for the conversion
of sulphur trioxide to sulphuric acid are
SO3 + H2SO4 (conc.) 
 H2S2O7 (oleum or pyrosulphuric acid)
H2S2O7 + H2O   2H2SO4
(b) (i) Copy and complete the following table:
Anode Electrolyte
Purification of copper [2]
(ii) Write the equation taking place at the anode. [1]
Ans. (i)
Anode Electrolyte
Purification of copper Impure copper Solution of copper
sulphate and dilute
sulphuric acid
(ii) Equation at the anode:
Cu – 2e¯   Cu
2+

(c) Explain the following:


(i) Dilute nitric acid is generally considered a typical acid but not so in its reaction with
metals.
(ii) Concentrated nitric add appears yellow when it is left standing in a glass bottle.
(iii) An all glass apparatus is used in the laboratory preparation of nitric acid. [3]
Ans. (i) Dilute nitric acid is generally considered a typical acid but not in its reaction with metals
because the action of nitric acid on metals depends on the temperature and concentration of
nitric acid. These conditions are not required in case of hydrochloric acid or sulphuric acid.
(ii) Although pure concentrated nitric acid is colourless, it appears yellow when left standing in a
glass bottle due to the dissolution of reddish brown nitrogen dioxide gas in the acid. Nitrogen
dioxide is produced because of the thermal decomposition of a portion of nitric acid.
4HNO3   2H2O + 4NO2 + O2
(iii) An all-glass apparatus is used in the laboratory preparation of nitric acid because nitric acid
vapours corrode rubber and cork.
Question 7
(a) The following questions are pertaining to the laboratory preparation of hydrogen chlo-
ride gas:
(i) Write the equation for its preparation mentioning the condition required. [1]
(ii) Name the drying agent used and justify your choice. [2]
(iii) State a safety precaution you would take during the preparation of hydrochloric acid. [1]
Ch 153 2015
Ans. (i) The equation for the laboratory preparation of hydrogen chloride gas:
NaCl + H2SO4  200 o C
  NaHSO4 + HCl 
Although it is a reversible reaction, it goes to completion as hydrogen chloride continuously
escapes as a gas.
The reaction can occur up to the stage of the formation of sodium sulphate on heating above
200°C.

above 200 C o
NaHSO4 + NaCl    Na2SO4 + HCl 
424

(ii) The drying agent used in the laboratory preparation of hydrochloric acid is conc.sulphuric acid.
Or H2SO4
(iii) Funnel arrangement are to be done as HCl is highly soluble in water and to avoid back suction.
Always wear chemical splash goggles, Chemical Resistant Gloves and Chemical Resistant apron
in the laboratory during the preperation of Hydrochloric acid.
(b) An element L consists of molecules.
(i) What type of bonding is present in the particles that make up L?
(ii) When L is heated with iron metal, it forms a compound FeL. What chemical term would
you use to describe the change undergone by L? [2]
Ans. (i) Covalent bonding is observed in atoms which are similar. Hence, covalent bonding is present
in the particles which make up element L.
(ii) When L is heated with iron metal, it forms a compound FeL.
Here, oxidation of Fe and reduction of L occur as follows:
Fe 
 Fe2+ + 2e¯
(c) From the list of the following salts choose the salt that most appropriately fits the
description given in the following:
[AgCl, MgCl2, NaHSO4, PbCO3, ZnCO3, KNO3, Ca(NO3)2]
(i) A deliquescent salt.
(ii) An insoluble chloride.
(iii) On heating, this salt gives a yellow residue when hot and white when cold.
(iv) On heating this salt, a brown coloured gas is evolved. [4]
Ans.
(i) A deliquescent salt = MgCl2
(ii) An insoluble chloride = AgCl
(iii) On heating, this salt gives a yellow residue when hot and a white residue when cold = ZnCO3
(iv) On heating this salt, a brown-coloured gas is evolved = Ca(NO3)2
ICSE QUESTION PAPER-2016 (SOLVED)

PAPER 2 (CHEMISTRY)
(Two hours)
Answers to this Paper must be written on the paper provided separately.
You will not be allowed to write during the first 15 minutes.
This time is to be spent in reading the Question Paper.
The time given at the head of this paper is the time allowed for writing the answers.

Section I is compulsory. Attempt any four questions from Section II.


The intended marks for questions or parts of questions are given in brackets [ ].

SECTION - I (40 MARKS)


(Attempt all questions from this Section)
Question 1.
(a) Fill in the blanks with the choices given in brackets. [5]
(i) Metals are good ............ (oxidizing agents/reducing agents)
because they are electron ............ (acceptors / donors).
(ii) Electrovalent compounds have ............ (high / low) melting points.
(iii) Higher the pH value of a solution, the more ............ (acidic / alkaline) it is.
(iv) ............ (AgCl / PbCl2), a white precipitate is soluble in excess NH4OH.
(v) Conversion of ethene to ethane is an example of ............ (hydration /
hydrogenation).
Ans. (i) Metals are good reducing agents.
because they are electron donors.
(ii) Electrovalent compounds have high melting points.
(iii) Higher the pH value of a solution, the more alkaline it is.
(iv) AgCl (because of formation of diammine silver chloride), a white precipitate is soluble
in excess NH4OH.
(v) Conversion of ethene to ethane is an example of hydrogenation.
(b) Choose the correct answer from the options given below : [5]
(i) An element with the atomic number 19 will most likely combine chemically with
the element whose atomic number is :
(A) 17 (B) 11
(C) 18 (D) 20
(ii) The ratio between the number of molecules in 2 g of hydrogen and 32 g of
oxygen is :
(A) 1 : 2 (B) 1 : 0.01
(C) 1 : 1 (D) 0.01 : 1 [Given that H = 1, O = 16]
(iii) The two main metals in Bronze are :
Ch 154 2016
Ch 155 2016
(A) Copper and zinc (B) Copper and lead
(C) Copper and nickel (D) Copper and tin
(iv) The particles present in strong electrolytes are :
(A) only molecules (B) mainly ions
(C) ions and molecules (D) only atoms
(v) The main of the Fountains Experimeter is to prove that :
(A) HCl turns blue litmus red (B) HCl is denser than air
(C) HCl is highly soluble in water (D) HCl fumes in moist air.
Ans. (i) (A) 17
(ii) (C) 1 : 1
(iii) (D) Copper and tin
(iv) (B) mainly ions
(v) (C) HCl is highly soluble in water
(c) Write balanced chemical equations for each of the following : [5]
(i) Action of warm water on AIN.
(ii) Action of hot and concentrated Nitric acid on copper.
(iii) Action of Hydrochloric acid on sodium bicarbonate.
(iv) Action of dilute Sulphuric acid on Sodium Sulphite.
(v) Preparation of ethanol from Ethyl Chloride.
Ans. (i) AlN + 3H2O  Al(OH)3 + NH3 
(ii) Cu + 4HNO3  Cu(NO3)2 + 2H2O + 2NO2 
(iii) NaHCO3 + HCl  NaCl + H2O + CO2 
(iv) Na2SO3 + H2SO4  Na2SO4 + H2O + SO2 
(v) C2H5Cl + NaOH(aq)  C2H5OH + NaCl
(d) State your observations when : [5]
(i) Dilute Hydrochloric acid is added to Lead nitrate sodium and the mixture is
heated.
(ii) Barium chloride solution is mixed with Sodium Sulphate Solution.
(iii) Concentrated Sulphuric acid is added to Sugar Crystals.
(iv) Dilute Hydrochloric acid is added to Copper carbonate.
(v) Dilute Hydrochloric acid is added to Sodium thiosulphate.
Ans. (i) Pb(NO3)2 + 2HCl (dil)  PbCl2  + 2HNO3
Lead chloride
(white precipitate)
When dilute hydrochloric acid is added to lead nitrate white precipitates of lead chloride
are formed.
(ii) BaCl2 + Na2SO4  BaSO4  + 2NaCl
(White precipitates)
Ch 156 2016
When sodium sulphate is mixed with barium chloride. White coloured precipitates of
Barium sulphate are formed.
(iii) C12H22O11 (s) Conc.H
 2SO 4
 12C (s) + 11 H2O
(sugar charcoal
Black spongy mass)
When conc. sulphuric acid is added to sugar crystals black spongy mass (sugar
charcoal) is formed.
(iv) CuCO3 + 2HCl  CuCl2 + H2O + CO2
Brisk effervescence
When dilute hydrochloric acid is added to copper carbonate brisk effervescence due to
the liberation of carbon dioxide is observed.
(v) Na2S2O3 + 2HCl  2NaCl + H2O + SO2  + S 
(yellow residue)
When dilute hydrochloric acid is added to sodium thiosulphate yellow residue (due to
formation of sulphur) is formed and a gas with choking odour is formed.
(e) Identify the term/substance in each of the following : [5]
(i) The tendency of an atom to attract electrons to itself when combined in a
compound.
(ii) The method used to separate ore from gangue by preferential wetting.
(iii) The catalyst used in the conversion of ethyne to ethane.
(iv) The type of reactions alkenes undergo.
(v) The electrons present in the outermost shell of an atom.
Ans. (i) Electronegativity (ii) Froth flotation process
(iii) Nickel or platinum or palladium (iv) Addition reactions
(v) Valence electrons
(f) (i) A gas of mass 32 gms has a volume of 20 litres at S.T.P. Calculate the gram
molecular weight of the gas. [5]
(ii) How much Calcium oxide is formed when 82 g of calcium nitrate is heated ? Also
find the volume of nitrogendioxide evolved :
2Ca(NO3)2   2CaO + 4NO2 + O2
(Ca = 40, N = 14, O = 16)
Ans. (i) 20 litres at S.T.P. weighs equal to 32 g
32
1 litre at S.T.P. weighs equal to g
20
22.4 litres at S.T.P.
32
Weighs equal to × 22.4 = 35.84 g
20

(ii) 2Ca (NO3)2   2CaO + 4NO2 + O2


2[40 + 2 (14 + 48)] 2(40 + 16) 4Vol.
Ch 157 2016
328 g = 112 g
82 g ? ?
328 g will decompose to form 112 g of CaO
112
1 g will decompose to form g of CaO
328
112
82g will decompose to form × 82 g = 28 g of CaO
328
328 g will decompose to form 4 vol of NO2
328 g will decompose to form 4 × 22.4 l of NO2
4  22.4
1 g will decompose to form l of NO2
328
4  22.4
82 g will decompose to form = × 82
328
= 22.4 l of NO2
= 1 Volume of NO2.
(g) Match the salts given in Column I with their method of preparation given in Column
II : [5]
Column I Column II
(i) Pb(NO3)2 from PbO (A) Simple displacement
(ii) MgCl2 from Mg (B) Titration
(iii) FeCl3 from Fe (C) Neutralization
(iv) NaNO3 from NaOH (D) Precipitation
(v) ZnCO3 from ZnSO4 (E) Combination
Ans. Column I Column II
(i) Pb(NO3)2 from PbO (C) Neutralization
(ii) MgCl2 from Mg (A) Simple displacement
(iii) FeCl3 from Fe (E) Combination
(iv) NaNO3 from NaOH (B) Titration
(v) ZnCO3 from ZnSO4 (D) Precipitation
(h) (i) Write the IUPAC names of each of the following :
H H H H H
1. H C C C H 2. H C C C C H
H H H
H H
3. H C C O
H
(ii) Rewrite the following sentences by using the correct symbol > (greater than) or
< (less than) in the blanks given :
Ch 158 2016
1. The ionization potential of Potassium is ............ that of Sodium.
2. The electronegativity of Iodine is ............ that of Chlorine.
Ans. (i) 1. prop - 1 - ene 2. but - 2 - yne
3. ethanal
(ii) 1. < (less than) 2. < (less than)

SECTION - II (40 MARKS)


(Attempt any four questions from this Section)
Question 2.
(a) Use the letters only written in the Periodic Table Given below to answer the questions
that follow : [4]
I II GROUPS III IV V VI VII 
1 L
2 Q E G J ZM
Periods

3 R
4 T
5

(i) State the number of valence electrons in atom J.


(ii) Which element shown forms ions with a single negative charge?
(iii) Which metallic element is more reactive than R?
(iv) Which element has its electrons arranged in four shells?
Ans. (i) J  (5 valence electrons)
(ii) M  (7 valence electrons) so it forms a uninegative ion
(iii) T
(iv) T
(b) Fill in the blanks by selecting the correct word from the brackets : [2]
(i) If an element has a low ionization energy then it is likely to be ............ (metallic
/ non-metallic).
(ii) If an element has seven electrons in its outermost shell then it is likely to have
the ............ (largest / smallest) atomic size among all the elements in the same
period.
Ans. (i) If an element has a low ionization energy then it is likely to be metallic.
(ii) If an element has seven electrons in its outermost shell then it is likely to have the
smallest atomic size among all the elements in the same period.
(c) The following table shows the electronic configuration of the elements W, X, Y, Z :
Element W X Y Z
Electronic 2,8,1 2,8,7 2,5 1
Configurations
Answer the following questions based on the table above :
(i) What type of Bond is formed between : [2]
Ch 159 2016
1. W and X 2. Y and Z
(ii) What is the formula of the compound formed between : [2]
1. X and Z 2. W and X
Ans. (i) 1. electrovalent bond 2. covalent bond
(ii) 1. Z+1 X1 2. +1
W
1
X

Z X W X
Question 3.
(a) Write a balanced chemical equation for each of the following : [3]
(i) Burning of ethane in plentiful supply of air.
(ii) Action of water on Calcium carbide.
(iii) Heating of Ethanol at 170°C in the presence of conc. Sulphuric acid.
7
Ans. (i) C 2 H6 + O 
 2CO2 + 3H2O
2 2

2C2H6 + 7O2 
 4CO2 + 6H2O

(ii) CaC2 + 2H2O 


 Ca(OH)2 + C2H2 
(iii) C2H5OH Conc.
 H2SO 4
 C2H4 + H2O
ethanol ethylene
(b) Give the structural formulae of each of the following : [3]
(i) 2-methyl propane (ii) Ethanoic acid
(iii) Butan – 2 – ol

Ans. (i) H H H
H C C C H
H H
H C H [2-Methyl propane]
H

(ii) H O
H C C O H
H [ethanoic acid]

(iii) H H OH H
H C C C C H
H H H H [butan  2  ol]
(c) Equation for the reaction when compound A is bubbled through bromine dissolved in
carbon tetrachloride is as follows : [2]
Ch 160 2016

Br2/CCl4
CH2Br
A
CH2Br
(i) Draw the structure of A.
(ii) State your observationduring this reaction.
H C C H
Ans. (i)
H H
CH2 == CH2 + Br2 CCl
4  CH2 – CH2
(Orangish Br Br
red) 1, 2 dibromo ethane (colourless)
(ii) Bromine water turns colourless.
(d) Fill in the blanks using the appropriate words given below : [2]
(Sulphur dioxide, Nitrogen dioxide, Nitric oxide, Sulphuric acid)
(i) Cold, dilute, nitric acid reacts with copper to given ............ .
(ii) Hot, concentrated nitric acid reacts with sulphur to form ............ .
Ans. (i) Cold, dilute, nitric acid reacts with copper to given nitric oxide.
(ii) Hot, concentrated nitric acid reacts with sulphur to form sulphuric acid.
Question 4.
(a) Identify the gas evolved and give the chemical test in each of the following cases :
[2]
(i) Dilute hydrochloric acid reacts with sodium sulphuric.
(ii) Dilute hydrochloric acid reacts with iron (II) sulphide.
Ans. (i) Na2SO3 + 2HCl  2NaCl + H2O + SO2 
Sulphur
dioxide
Chemical test : Sulphur dioxide decolorises potassium permanganate solution.
(ii) FeS + 2HCl  FeCl2 + H2S
Hydrogen sulphide
Chemical test : Hydrogen sulphide turns lead acetate paper black.
(b) State your observations when ammonium hydroxide solution is added drop by drop
and then in excess to each of the following solutions : [2]
(i) copper sulphate solution (ii) zinc sulphate solution
Ans. (i) CuSO4 + 2NH4OH  Cu(OH)2 + (NH4)2 SO4
Pale Blue Precipitates
Cu(OH)2 + 2(NH4)2SO4 + 2NH4OH  [Cu(NH3)4] SO4 + 4H2O
Deep blue solution
Pale blue precipitates are formed and then these precipitates dissolve in excess of
ammonium hydroxide to form deep blue solution.
(ii) ZnSO4 + NH4OH  Zn(OH)2  + (NH4)2SO4
White gelatinous
Ch 161 2016
Zn(OH)2 + (NH4)2SO4 + 2NH4OH  [Zn(NH3)4]SO4 + 4H2O
Colourless
When ammonium hydroxide is added drop by drop white gelatinous precipitate of zinc
hydroxide is formed and on adding excess of ammonium hydroxide colourless solution
is formed.
(c) Write equations for the reactions taking place at the two electrodes (mentioning
clearly the name of the electrode) during the electrolysis of : [4]
(i) Acidified copper sulphate solution with copper electrodes.
(ii) Molten lead bromide with inert electrodes.
Ans. (i) Dissociation reaction :
CuSO4 Cu2+ + SO42–
H2 O H+ + OH–
H2SO4 2H+ + SO42–
Cations Anions
At cathode (negative terminal of battery)
Cu2+ + 2e–  Cu(s)
copper metal gets deposited [Reduction]
At anode (positive terminal of battery)
Cu  Cu2+ + 2e–
Copper metal gets converted into ions [Oxidation]
(ii) Dissociation reaction
PbBr2 (m) Pb2+ + 2Br–
Cation Anion
 
Cathode Anode
Pb Reduction
At cathode (negative terminal of battery)
Pb2+ + 2e–  Pb
Lead metal is obtained in molten state
At anode (positive terminal of battery)
Br– – le–  Br
Br + Br  Br2
Reddish brown vapours of bromine are formed
(d) (i) Name the product formed at the anode during the electrolysis of acidified water
using platinum electrodes. [2]
(ii) Name the metallic ions that should be present in the electrolyte when an article
made of copper is to be electroplated with silver.
Ans. (i) Hydrogen (ii) Silver ions
Question 5.
(a) A gas cylinder contains 12 × 1024 molecules of oxygen gas. [2]
Ch 162 2016
If Avogadro’s number is 6 × 1023 ; Calculate :
(i) the mass of oxygen present in the cylinder.
(ii) the volume of oxygen at S.T.P. present in the cylinder. [O = 16]
Ans. (i) If 6 × 1023 = 32 g
32
1 molecules are present the mass is =
6  10 23
32
12 × 1024 molecules are present than the mass = × 12 × 1024
6  10 23
= 32 × 20 = 640 g
(ii) 32 g 
 22.4 l at S.T.P..
22.4
1 g will occupy a volume of
32
22.4
640 g will occupy a volume of = × 640 = 448 l
32
(b) A gaseous hydrocarbon contains 82.76% of carbon. Given that its vapours density is
29, find its molecular formula. [C = 12, H = 1] [3]
Ans. Symbol Percentage Atomic mass Relative ratio Simplest ratio Simplest
whole no.
C 82.76 12 6.89 6.89/6.89 = 1 2
H 17.24 1 17.24 17.24/6.89 = 2.5 5
E.F. = C2H5
E.F.M. = 12 × 2 + 5 × 1 = 24 + 5 = 29
V.D. = 29
M.M. = 2 × V.D. = 2 × 29 = 58
M.M 58
n= = =2
E.F.M 29
Molecular formula = (C2H5)2 = C4H10
(c) The equations 4NH3 + 5O2  4NO + 6H2O, represents the catalytic oxidation of
ammonia. If 100 cm3 of ammonia is used calculate the volume of oxygen required to
oxidise the ammonia completely. [3]
Ans. 4NH3 + 5O2  4NO + 6H2O
4 Vol 5 Vol 4 Vol
100 cm3 ?
4 vol requires 5 vol
5
1 vol requires
4
5
100 cm3 requires × 100 = 125 cm3 of O2
4
Ch 163 2016
(d) By drawing an electron dot diagram show the formation of Ammonium Ion [Atomic
No. : N = 7 and H = 1]
Ans. N(7)  2, 5 H(1)  1

H × ××
N × H
×

H + H +
×× ××
H × N × H + H+ H×N×H H N H
× ×
H H
Question 6.
(a) Name the gas evolved when the following mixtures are heated : [2]
(i) Calcium hydroxide and Ammonium Chloride.
(ii) Sodium Nitrite and Ammonium Chloride.
Ans. (i) Ammonia (ii) Nitrogen
(b) Write balanced chemical equations for each of the following : [2]
(i) When excess of ammonia is treated with chlorine.
(ii) An equation to illustrate the reducing nature of ammonia.
Ans. (i) 8NH3 + 3Cl2 
 6NH4Cl + N2

(ii) 3CuO + 2NH3   3Cu + N2 + 3H2O


(c) A, B, C and D summarize the properties of sulphuric acid depending on whether it is
dilute or concentrated. [3]
A = Typical acid property
B = Non-volatile acid
C = Oxidizing agent
D = Dehydrating agent
Choose the property (A, B, C or D) depending on which is relevant to each of the
following :
(i) Preparation of Hydrogen chloride gas.
(ii) Preparation of Copper sulphate from copper oxide.
(iii) Action of conc. Sulphuric acid on Sulphur.
Ans. (i) Preparation of Hydrogen chloride gas. B = Non-volatile acid
(ii) Preparation of Copper sulphate from copper oxide. A = Typical acid property
(iii) Action of conc. Sulphuric acid on Sulphur. C = Oxidizing agent
Ch 164 2016
(d) Give reasons why : [3]
(i) Sodium Chloride will conduct electricity only in fused or aqueous solution state.
(ii) In the electroplating of an article with silver, the electrolyte sodium argento-
cyanide solution is preferred over silver nitrate solution.
(iii) Although copper is a good conductor of electricity, it is a non-electrolyte.
Ans. (i) Sodium chloride is a solid, ionic compound, it has strong electrostatic forces of
attraction. Only in the fused or aqueous solution, it forms ions which are capable of
conducting electricity.
(ii) It is preferred over silver nitrate because the dissociation of AgNO3 will be very fast and
deposit will not be smooth and uniform.
(iii) An electrolyte must have free ions which act as charge carriers where as in copper the
free electrons act as charge carriers.
Question 7.
(a) (i) Name the solution used to react the Bauxite as a first step in obtaining pure
aluminium oxide, in the Baeyer’s process. [5]
(ii) Write the equation for the reaction where the aluminum oxide for the electrolytic
extraction of aluminum is obtained by heating aluminum hydroxide.
(iii) Name the compound added to pure alumina to lower the fusion temperature
during the electrolytic reduction of alumina.
(iv) Write the equation for the reaction that occurs at the cathode during the extraction
of aluminium by electrolysis.
(v) Explain why it is preferable to use a number of graphite electrodes as anode
instead of a single electrode, during the abvoe electrolysis.
Ans. (i) Sodium hydroxide solution

(ii) 2Al (OH)3 
1100 C
 Al2O3 + 3H2O
(iii) Cryolite (Na3Al F6) acts as a solvent and lower the fusion temperature from 2050°C to
950°C.
(iv) Cathode : (rich in electron)
Al3+ + 3e–   Al
(v) It is preferable to use a number of graphite electrodes as anode because anode gets
oxidised by the oxygen evolved.
Hence, if large number of electrodes are used it will keep the process continuous for a
longer time.
(b) State what would you observe when : [2]
(i) Washing Soda Crystals are exposed to the atmosphere.
(ii) The salt ferric chloride is exposed to the atmosphere.
Ans. (i) Na2CO3.10H2O   Na2CO3.H2O + 9H2O
It will loose water of crystallisation and become a monohydrate. The appearance will be
dull and flaky.
(ii) Ferric chloride is a deliquescent substance, thus it will absorb water from atmosphere
Ch 165 2016
and finally dissolve in that water forming a saturated solution.
(c) Identify the cations in each of the following case : [3]
(i) NaOH solution when added to the Solution (A) gives a reddish brown precipitate.
(ii) NH4OH Solution when added to the Solution (B) gives white ppt. which does not
dissolve in excess.
(iii) NaOH Solution when added to Solution (C) gives white ppt. which is insoluble in
excess.
Ans. (i) Fe+3 (ii) Pb2+
(iii) Ca2+
ICSE QUESTION PAPER-2017 (SOLVED)

PAPER 2 (CHEMISTRY)
(Two hours)
Answers to this Paper must be written on the paper provided separately.
You will not be allowed to write during the first 15 minutes.
This time is to be spent in reading the Question Paper.
The time given at the head of this paper is the time allowed for writing the answers.

Section I is compulsory. Attempt any four questions from Section II.


The intended marks for questions or parts of questions are given in brackets [ ].

SECTION - I (40 MARKS)


(Attempt all questions from this Section)
Question 1.
(a) Fill in the blanks from the choices given in brackets. [5]
(i) The energy required to remove an electron from a neutral isolated gaseous
atom and convert it into a positively charged gaseous ion is called .
(electron affinity, ionisation potential, electronegativity)
(ii) The compound that does not have a lone pair of electrons is . (water,,
ammonia, carbon tetrachloride)
(iii) When a metallic oxide is dissolved in water, the solution formed has a high
concentration of ions. (H+, H3O+, OH–)
(iv) Potassium sulphite on reacting with hydrochloric acid releases gas.
(Cl2, SO2, H2S)
(v) The compound formed when ethene reacts with Hydrogen is . (CH4,
C2 H 6 , C 3 H 8 )
Ans. (i) Ionisation potential
(ii) Carbon tetrachloride or CCl4
(iii) OH –
(iv) SO2
(v) C 2H 6
(b) Choose the correct answer from the options given below : [5]
(i) A chloride which forms a precipitate that is soluble in excess of ammonium
hydroxide, is:
(A) Calcium chloride (B) Ferrous chloride
(C) Ferric chloride (D) Copper chloride
(ii) If the molecular formula of an organic compounds is C10H18 it is:
(A) alkene (B) alkane
Ch 166 2017
Ch 167 2017
(C) alkyne (D) Not a hydrocarbon
(iii) Which of the following is a common characteristic of a covalent compound?
(A) high melting point
(B) consists of molecules
(C) always soluble in water
(D) conducts electricity when it is in the molten state
(iv) To increase the pH value of a neutral solution, we should add:
(A) an acid (B) an acid salt
(C) an alkali (D) a salt
(v) Anhydrous iron (III) chloride is prepared by:
(A) direct combination (B) simple displacement
(C) decomposition (D) neutralization
Ans. (i) (D) copper chloride
(ii) (C) alkyne
(iii) (B) consists of molecules
(iv) (C) an alkali
(v) (A) direct combination
(c) Identify the substance underlined, in each of the following cases: [5]
(i) Cation that does not form a precipitate with ammonium hydroxide but forms
one with sodium hydroxide.
(ii) The electrolyte used for electroplating an article with silver.
(iii) The particles present in a liquid such as kerosene, that is a non-electrolyte.
(iv) An organic compound containing – COOH functional group.
(v) A solid formed by reaction of two gases, one of which is acidic and the other
basic in nature.
Ans. (i) Ca 2+
(ii) sodium silver cyanide solution Na[Ag(CN)2]
(iii) Free molecule
(iv) carboxylic acid
(v) ammonium chloride (NH4Cl)
(d) Write a balanced chemical equation for each of the following: [5]
(i) Action of cold and dilute nitric acid on copper.
(ii) Reaction of ammonia with heated copper oxide.
(iii) Preparation of methane from iodomethane.
(iv) Action of concentrated sulphuric acid on sulphur.
(v) Laboratory preparation of ammonia from ammonium chloride.
Ans. (i) 3Cu + 8HNO3 (dil)
 3Cu (NO3)2 + 4H2O + 2NO
(ii) 3CuO + 2NH3  3Cu + 3H2O + N2 
Zn/Cu
(iii) CH3I + 2[H] alcohol
CH4 + HI
Ch 168 2017
(iv) S + 2H2SO4 (conc.)  3SO2 + 2H2O
(v) 2NH4Cl + Ca(OH)2  CaCl2 + 2H2O + 2NH3
(e) State one relevant observation for each of the following reactions: [5]
(i) Addition of ethyl alcohol to acetic acid in the presence of concentrated
sulphuric acid.
(ii) Action of dilute hydrochloric acid on iron (II) sulphide.
(iii) Action of sodium hydroxide solution on ferrous sulphate solution.
(iv) Burning of ammonia in air.
(v) Action of concentrated sulphuric acid on hydrated copper sulphate.
Ans. (i) Fruit smell evolved (Ether).
(ii) H2S evolves with rotten egg smell.
(iii) Dirty green ppt [Fe(OH)2] which is insoluble in excess NaOH.
(iv) Burns with a green flame producing N2 and H2O.
(v) Blue crystal loses its crystalline shape and becomes white amorphous.
(f) (i) Draw the structural formula for each of the following: [5]
1. 2, 3 – dimethyl butane
2. diethyl ether
3. propanoic acid
(ii) From the list of terms given, choose the most appropriate term to match the
given description.
(calcination, roasting, pulverisation, smelting)
1. Crushing of the ore into a fine powder.
2. Heating of the ore in the absence of air to a high temperature.
Ans. (i) 1. 2, 3 – dimethyl butane

H
C H
CH3 CH3 H H H
H3C CH CH CH3 Or H C C C C H
(4) (3) (2) (1)
H H H H
2. diethyl ether
H H H H
C2H5 – O – C2H5 Or H C C O C C H
H H H H
3. Propanoic acid
H H O
H C C C OH
H H
Ch 169 2017
(ii) 1. Pulverisation
2. Calcination
(g) (i) Calculate the number of gram atoms in 4.6 grams of sodium (Na = 23). [5]
(ii) Calculate the percentage of water of crystallization in CuSO4.5H2O
(H = 1, O = 16, S = 32, Cu = 64)
(iii) A compound of X and Y has the empirical formula XY2. Its vapour density is
equal to its empiricial formula weight. Determine its molecular formula.
Ans. (i) 23 g of sodium = 1 gram atom
1
4.6 g of sodium = × 4.6 = 0.2 gram atom
23
(ii) Molecular mass of CuSO4.5H2O
CuSO4 5H2O
Cu = 64 × 1 = 64 H = 1 × 10 = 10
S = 32 × 1 = 32 O = 16 × 5 = 80
O = 16 × 4 = 64
160 g 90 g
 Molecular mas of compound = 160 + 90 = 250 g
 % of water of crystallization
90
= × 100 = 36%
250
(ii) Let V.D. of compound = n
Molecular mass = 2x
Empirical formula weight = x
Molecular mass
 n = Empirical formula weight

2x
= ×2
x
 Molecular formula of the compound = 2(XY2) = X2Y4
(h) Match the atomic number 2, 4, 8, 15, and 19 with each of the following: [5]
(i) A solid non-metal belonging to the third period.
(ii) A metal of valency 1.
(iii) A gaseous element with valency 2.
(iv) An element belonging to Group 2.
(v) A rare gas.
Ans. (i) 15 (ii) 19
(iii) 8 (iv) 4
(v) 2
Ch 170 2017
SECTION - II (40 MARKS)
(Attempt any four questions from this Section)
Question 2.
(a) Arrange the following as per the instruction given in the brackets: [4]
(i) He, Ar, Ne (Increasing order of the number of electron shells)
(ii) Na, Li, K (Increasing ionisation energy)
(iii) F, Cl, Br (Increasing electronegativity)
(iv) Na, K, Li (increasing atomic size)
Ans. (i) He, Ne, Ar (ii) K, Na, Li
(iii) Br, Cl, F (iv) Li, Na, K
(b) State the type of Bonding in the following molecules: [2]
(i) Water (ii) Calcium oxide
Ans. (i) Water – Covalent bond
(ii) Calcium oxide – Electrovalent bond
(c) Answer the following questions: [2]
(i) How will you distinguish between ammonium hydroxide and sodium hydroxide
using copper sulphate solutions?
(ii) How will you distinguish between dilute hydrochloric acid and dilute sulphuric
acid using lead nitrate solution?
Ans. (i) Using NaOH, it forms blue ppt which is insoluble in excess NaOH. With NH4OH it
forms blue ppt which is soluble in excess NH4OH.
(ii) Using lead nitrate solution, only HCl will form curdy white ppt and not H2SO4.
(d) Identify the salts P and Q from the observations given below: [2]
(i) On performing the flame test salt P produces a lilac coloured flame and its
solution gives a white precipitate with silver nitrate solution, which is soluble
in ammonium hydroxide solution.
(ii) When dilute HCl is added to a salt Q, a brisk effervescence is produced and
the gas turns lime water milky.
When NH4OH solution is added to the above mixture (after adding dilute
HCl), it produces a white precipitate which is soluble in excess NH4OH
solution.
Ans. (i) Salt ‘P’ is potassium chloride (KCl)
(ii) Salt ‘Q’ is zinc carbonate (ZnCO3)
Question 3.
(a) Draw an electron dot diagram to show the formation of each of the following
compounds: [4]
(i) Methane (ii) Magnesium chloride
[H = 1, C = 6, Mg = 12, Cl = 17]
Ans. (i) Methane [H = 1, C = 6]
Ch 171 2017
Hydrogen atom: Z = 1
Configuration: 1
Carbon atom: Z = 6
Configuration: 2, 4
H
+ H H
H + C + H  H C H H C H
+ H H
H
(ii) Magnesium chloride
Mg atom: Z = 12
Configuration: 2, 8, 2
Cl atom: Z = 17
Configuration: 2, 8, 7
Cl + Mg + Cl  (Mg) + 2( Cl )  MgCl2
++

(b) State the observations at the anode and at the cathode during the electrolysis of:
[4]
(i) fused bromide using graphite electrodes.
(ii) copper sulphate solution using copper electrodes.
Ans. (i) At cathode, Pb2+ migrate to form silvery grey metallic lead.
At anode, Br– migrate forming bromide atom and then bromine molecule, ultimately
forming bromine vapours. These are Reddish Brown Fumes.
(ii) At cathode, Cu2+ ions accept electrons and form neutral copper atom deposition here
size of Anode gets decreases.
At anode, neither SO42– nor OH–, are discharged and copper atoms of the anode lose
electrons and enter the solution as copper ions.
(c) Select the ion in each case, that would get selectively discharged from the
aqueous mixture of the ions listed below: [2]
(i) SO42–, NO3–, and OH–
(ii) Pb2+, Ag+ and Cu2+
Ans. (i) OH – (ii) Ag+
Question 4.
(a) Certain blank spaces are left in the following table and these are labelled as A, B,
C, D and E. Identify each of them. [5]
Lab Reactants Products Drying agent Method of
preparation of used formed collection
(i) HCl gas NaCl + H2SO4 ___A___ conc. H2SO4 ___B___
(ii) NH3 gas ___C___ Mg(OH)2 ___D___ ___E___
NH3
Ch 172 2017
Ans. (i) A  NaHSO4 + HCl
B  Upward displacement of air
(ii) C  Mg3N2 + H2O
D  CaO (Calcium oxide)
E  By downward displacement of air
(b) Write balanced chemical equations to show: [3]
(i) The oxidizing action of conc. sulphuric acid on carbon.
(ii) The behaviour of H2SO4 as an acid when it reacts with magnesium.
(iii) The dehydrating property of conc. sulphuric acid with sugar.
Ans. (i) C + 2H2SO4  CO2 + 2H2O + SO2
(conc)
(ii) Mg + H2SO4  MgSO4 + H2
H2SO4
(iii) C12H22O11 (conc) 12C + 11H2O
(c) Write balanced chemical equations to show how SO3 is converted to sulphuric
acid in the contact process. [2]
Ans. SO3 + H2SO4  H2S2O7
H2S2O7 + H2O  2H2SO4
Question 5.
(a) (i) Propane burns in air according to the following equation: [4]
C3H8 + 5O2  3CO2 + 4H2O.
What volume of propane is consumed on using 1000 cm3 of air, considering
only 20% of air contains oxygen?
(ii) The mass of 11.2 litres of a certain gas at s.t.p. is 24 g. Find gram molecular
mass of the gas.
Ans. (i) According to the equation
C3H8 + 5O2  3CO2 + 4H2O
1 vol : 5 vol : : 3 vol : 4 vol
20
Reacting oxygen in 1000 cm3 of air is × 1000 = 200 cm3
100
Vol of O2  5 Vol  200 cm3
 1 Vol  40 cm3
 Vol of propane consumed = 1 Vol = 1 × 40 = 40 cm3
(ii) 11.2 litre of gas  24 g
 22.4 litre (molar volume)
24
= × 22.4 = 48 g
11.2
 Gram molecular mass is 48 g.
Ch 173 2017
(b) A gas cylinder can hold 1 kg of hydrogen at room temperature and pressure:
[4]
(i) Find the number of moles of hydrogen present.
(ii) What weight of CO2 can the cylinder hold under similar conditions of
temperature and pressure? (H = 1, C = 12, O = 16)
(iii) If the number of molecules of hydrogen in the cylinder is x, calculate the
number of CO2 molecules in the cylinder under the same conditions of
temperature and pressure.
(iv) State the law that helped you to arrive at the above result.
Ans. (i) Molecular mass of hydrogen = 2 × 1 g
2 g is present in 1 mole
1
 1000 g (1 kg)  × 1000  500 moles
2
(ii) Molecular mass of CO2 = 12 + 16 × 2 = 44 g
1 mole = 44 g
 500 moles = 44 × 500 = 22000 g = 22 kg
(iii) The number of molecules of CO2 in the cylinder is X.
(iv) Avogadro’s Law – Under same condition of temperature and pressure, equal volumes
of gases contain the same number of molecules.
(c) Write a balanced chemical equation for the preparation of each of the following
salts: [2]
(i) Copper carbonate (ii) Ammonium sulphate crystals
Ans. (i) CuCl2 + Na2CO3  2NaCl + CuCO3 
(ii) (NH4)2CO3 + H2SO4  (NH4)2SO4 + H2O + CO2
Question 6.
(a) Give a balanced chemical equation for each of the following: [4]
(i) Action of conc. nitric acid on sulphur.
(ii) Catalytic oxidation of ammonia.
(iii) Laboratory preparation of nitric acid.
(iv) Reaction of ammonia with nitric acid.
Ans. (i) S + 6HNO3  H2SO4 + 6NO2 + 2H2O
(conc)
Pt
(ii) 4NH3 + 5O2 
700 – 800 C
 4NO + 6H2O + 

(iii) KNO3 + H2SO4  200 C


 KHSO4 + HNO3
(conc)
(iv) NH3 + HNO3  NH4NO3
(b) Identify the term or substance based on the descriptions given below: [4]
Ch 174 2017
(i) Ice like crystals formed on cooling an organic acid sufficiently.
(ii) Hydrocarbon containing a triple bond used for welding purposes.
(iii) The property by virtue of which the compound has the same molecular
formula but different structural formulae.
O
(iv) The compound formed where two alkyl groups are linked by C group.
Ans. (i) Glacial acetic acid (ii) Acetylene
(iii) Isomerism (iv) Ketone
(c) Give a balanced chemical equation for each of the following: [2]
(i) Preparation of ethane from sodium propionate.
(ii) Action of alcoholic KOH on bromoethane.

Ans. (i) C2H5COONa + NaOH CaO
C2H6 + Na2CO3
(ii) C2H5Br + KOH  C2H4 + KBr + H2O
alcoholic
Question 7.
(a) Name the following: [4]
(i) The process of coating of iron with zinc.
(ii) An alloy of lead and tin that is used in electrical circuits.
(iii) An ore of zinc containing its sulphide.
(iv) A metal oxide that can be reduced by hydrogen.
Ans. (i) Galvanisation (ii) Solder
(iii) Zinc sulphide or Zinc blende (ZnS) (iv) CuO
(b) Answer the following questions with respect to the electrolytic process in the
extraction of aluminum: [3]
(i) Identify the components of the electrolyte other than pure alumina and the
role played by each.
(ii) Explain why powdered coke is sprinkled over the electrolytic mixture.
Ans. (i) Cryolite – lower fusion temperature.
Fluorspar – acts as solvent for electrolytic mixture and increase conductivity.
(ii) To reduce heat loss by radiation and prevent the burning of carbon anode.
(c) Complete the following by selecting the correct option from the choices given:
[3]
(i) The metal which does not react with water or dilute H2SO4 but reacts with
concentrated H2SO4 is . (Al/Cu/Zn/Fe)
(ii) The metal whose oxide, which is amphoteric, is reduced to metal by carbon
reduction . (Fe/Mg/Pb/Al)
(iii) The divalent metal whose oxide is reduced to metal by electrolysis of its
fused salt is . (Al/Na/Mg/K)
Ans. (i) Cu (ii) Pb (iii) Mg
ICSE QUESTION PAPER-2018 (SOLVED)

PAPER 2 (CHEMISTRY)
(Two hours)
Answers to this Paper must be written on the paper provided separately.
You will not be allowed to write during the first 15 minutes.
This time is to be spent in reading the Question Paper.
The time given at the head of this paper is the time allowed for writing the answers.

Section I is compulsory. Attempt any four questions from Section II.


The intended marks for questions or parts of questions are given in brackets [ ].

SECTION - I (40 MARKS)


(Attempt all questions from this Section)
Question 1.
(a) Choose the correct answer from the options given below: [5]
(i) The salt solution which does not react with ammonium hydroxide is:
(A) Calcium Nitrate (B) Zinc Nitrate
(C) Lead Nitrate (D) Copper Nitrate
(ii) The organic compound which undergoes substitution reaction is:
(A) C 2 H 2 (B) C 2 H 4
(C) C 10 H 18 (D) C 2 H 6
(iii) The electrolysis of acidified water is an example of:
(A) Reduction (B) Oxidation
(C) Redox reaction (D) Synthesis
(iv) The IUPAC name of dimethyl ether is:
(A) Ethoxy methane (B) Methoxy methane
(C) Methoxy ethane (D) Ethoxy ethane
(v) The catalyst used in the Contact Process is:
(A) Copper (B) Iron
(C) Vanadium pentoxide (D) Manganese dioxide
Ans. (i) (A) Calcium Nitrate
(ii) (D) C 2H 6
(iii) (C) Redox reaction
(iv) (B) Methoxy methane
(v) (C) Vanadium pentoxide
(b) Give one word or a phrase for the following statements: [5]
(i) The energy released when an electron is added to a neutral gaseous isolated
Ch 175 2018
Ch 176 2018
atom to form a negatively charged ion.
(ii) Process of formation of ions from molecules which are not in ionic state.
(iii) The tendency of an element to form chains of identical atoms.
(iv) The property by which certain hydrated salts, when left exposed to
atmosphere, lose their water of crystallization and crumble into powder.
(v) The process by which sulphide ore is concentrated.
Ans. (i) Electron affinity (ii) Ionisation
(iii) Catenation (iv) Efflorescence
(v) Froth floatation process.
(c) Write a balanced chemical equation for each of the following: [5]
(i) Action of concentrated sulphuric acid on carbon.
(ii) Reaction of sodium hydroxide solution with iron (III) chloride solution.
(iii) Action of heat on aluminium hydroxide.
(iv) Reaction of zinc with potassium hydroxide solution.
(v) Action of dilute hydrochloric acid on magnesium sulphite.
Ans. (i) C + 2H2SO4  2H2O + 2SO2  + CO2 
(ii) FeCl3 + 3NaOH  Fe(OH)3 + 3NaCl
(iii) 2Al(OH)3 
Δ Al2O3 + 3H2O
(iv) Zn + 2KOH  K2ZnO2 + H2O
(v) MgSO3 + 2HCl  MgCl2 + H2O + SO2 
(d) (i) Give the IUPAC name for each of the following:
H H H
1. H C O 2. H C C C OH
H H H H
H H
3. H3C C C CH3
(ii) Write the structural formula of the two isomers of butane.
Ans. (i) 1. Methanal 2. Propanol
3. But-2-ene

H H H H H H H
(ii) H C C C C H H C C C H
H H H H H H
(n-butane) H C C
Butane
H
(Isobutane)
2 Methyl butane

(e) State one relevant observation for each of the following: [5]
Ch 177 2018
(i) Lead nitrate solution is treated with sodium hydroxide solution drop wise till
it is in excess.
(ii) At the anode, when molten lead bromide is electrolyzed using graphite
electrodes.
(iii) Lead nitrate solution is mixed with dilute hydrochloric acid and heated.
(iv) Anhydrous calcium chloride is exposed to air for sometime.
(v) Barium chloride solution is slowly added to sodium sulphate solution.
Ans. (i) Pb(NO3)2 + 2NaOH  Pb(OH)2  + 2NaNO3
white precipitate
Pb(OH)2 + 2NaOH  Na2PbO2 + 2H2O
(Soluble)
When drop by drop of sodium hydroxide is added, chalky white precipitates appear.
These precipitates dissolve in excess of sodium hydroxide forming sodium plumbite.
(ii) PbBr2 Pb2+ + 2Br–
Cation Anion
 
(Cathode) (Anode)
At Anode
2Br– – 2e–  2Br
Br + Br  Br2 
At anode reddish vapours of bromine escape in air from lead bromide.
(iii) Pb(NO3)2 + 2HCl  
Δ  PbCl2  + 2HNO3
(white precipitates)
White precipitates of lead chloride are formed which are soluble in hot water and
insoluble in cold water.
(iv) Colourless solution or solid CaCl2 turns into solution.
(v) A white ppt is formed.
(f) Give a reason for each of the following: [5]
(i) Ionic compounds have a high melting point.
(ii) Inert gases do not form ions.
(iii) Ionisation potential increases across a period, from left to right.
(iv) Alkali metals are good reducing agents.
(v) Conductivity of dilute hydrochloric acid is greater than that of acetic acid.
Ans. (i) Ionic compounds have ions held strongly by electrostatic forces of attraction. These
strong forces need more energy to be broken apart. Hence, they have high melting
point.
(ii) Inert gases do not form ions because they have completely filled octet. They are
extremely stable. Hence, they neither loose, nor gain electrons.
(iii) As we move from left to right a period, the atomic size decreases due to the increase
in nuclear charge thus more energy is required to remove the electron, hence
ionisation potential increase.
Ch 178 2018
(iv) Alkali metals have free electrons. They can easily loose electrons to form positive
ions. The loss of electron is known as oxidation and the substance/element that lose
electrons is said to be reducing agent.

Na (II)  1e

Na
+

2, 8, 1 2, 8
Loss of electron
Oxidation
Reducing
agent

(v) Hydrochloric acid is a strong acid. It splits to give more hydrogen ions as compared
to acetic acid. Hence, conductivity of dilute hydrochloric acid is more than that of
acetic acid.
(g) Name the gas that is produced in each of the following cases: [5]
(i) Sulphur is oxidized by concentrated nitric acid.
(ii) Action of dilute hydrochloric acid on sodium sulphide.
(iii) Action of cold and dilute nitric acid on copper.
(iv) At the anode during the electrolysis of acidified water.
(v) Reaction of ethanol and sodium.
Ans. (i) Nitrogen dioxide (ii) Hydrogen sulphide
(iii) Nitrogen monoxide (iv) Oxygen (v) Hydrogen
(h) Fill up the blanks with the correct choice given in brackets. [5]
(i) Ionic or electrovalent compounds do not conduct electricity in their
state. (fused/solid)
(ii) Electrolysis of aqueous sodium chloride solution will form at the
cathode. (hydrogen gas/sodium metal)
(iii) Dry hydrogen chloride gas can be collected by displacement of air..
(downward/upward)
(iv) The most common ore of iron is . (calomine/haematite)
(v) The salt prepared by the method of direct combination is .
(iron (II) chloride/ iron (III) chloride)
Ans. (i) solid (ii) hydrogen gas
(iii) upward (iv) haematite
(v) iron (III) chloride

SECTION - II (40 MARKS)


(Attempt any four questions from this Section)
Question 2.
(a) (i) What do you understand by a lone pair of electrons? [3]
Ch 179 2018
(ii) Draw the electron dot diagram of Hydromium ion. (H = 1, O = 8)
Ans. (i) The unshared pair of electron that does not normally take part in a chemical reaction
is known as lone pair.

lone
X X pair of
electron
H X N X H
X
bond pair
H

(ii) Hydronium ion


Formation of proton :
H(1) – 1e–  H + H(1)  H•
××
Structure of water O (8)  ×× O ××
X X X X
H X O X H H O H
X X X X
Formation of hydronium ion

+
XX X X
+
H O H + H H O H
XX

H
Hydronium ion

(b) In Period 3 of the Periodic Table, element B is placed to the left of element A.
[3]
On the basis of this information, choose the correct word from the brackets to
complete the following statements:
(i) The element B would have (lower/higher) metallic character than A.
(ii) The element A would probably have (lesser/higher) electron affinity than B.
(iii) The element A would have (greater/smaller) atomic size than B.
Ans. (i) Higher (ii) Higher
(iii) Smaller
(c) Copy and complete the following table which refers to the conversion of ions to
neutral particles. [4]
Ch 180 2018

Conversion Ionic Equation Oxidation/Reduction


Chloride ion to chlorine molecule (i) (ii)
Lead (II) ion to lead (iii) (iv)

Ans. Conversion Ionic Equation Oxidation/Reduction


Chloride ion to chlorine molecule (i) Cl– – le–  Cl (ii) Oxidation
3
Cl + Cl  Cl2
Lead (II) ion to lead (iii) Pb2+ + 2e–  Pb (iv) Reduction

Question 3.
(a) (i) Write the balanced chemical equation to prepare ammonia gas in the
laboratory by using an alkali. [3]
(ii) State why concentrated sulphuric acid is not used for drying ammonia gas.
(iii) Why is ammonia gas not collected over water?
Ans. (i) 2NH4Cl + Ca(OH)2 Δ
 CaCl2 + 2H2O + 2NH3 
Ammonium Calcium hydroxide
chloride
(ii) Concentrated sulphuric acid reacts with ammonia / form ammonium sulphate or NH
being basic combines with concentrated H2SO4 / correct chemical equation.
2NH3 + H2SO4  (NH4)2SO4
(iii) Ammonia is highly soluble gas one volume of water can dissolve 702 volumes of
ammonia at 20°C and at 1 atmospheric pressure. Hence, it is not collected over
water.
(b) (i) Name the acid used for the preparation of hydrogen chloride gas in the
laboratory. Why is this particular acid preferred to other acids? [3]
(ii) Write the balanced chemical equation for the laboratory preparation of
hydrogen chloride gas.
Ans. (i) Concentrated sulphuric acid
[It is preferred to other acids because it is non-volatile acid]
(ii) NaCl + H2SO4 below C
 200
  NaHSO4 + HCl
(c) For the preparation of hydrochloric acid in the laboratory: [2]
(i) Why is direct absorption of hydrogen chloride gas in water not feasible?
(ii) What arrangement is done to dissolve hydrogen chloride gas in water?
Ans. (i) Direct absorption of hydrogen chloride gas in water is not feasible as it leads to back
suction.
(ii) “Inverted funnel arrangement” is done to dissolve hydrogen chloride gas in water.
(d) For the electro-refining of copper [2]
(i) What is the cathode made up of ?
(ii) Write the reaction that takes place at the anode.
Ch 181 2018
Ans. (i) Cathode is made-up of thin sheets of pure copper connected in parallel.
(ii) Cu – 2e–  Cu2+
Copper anode undergoes oxidation forming Cu2+ ions which pass into the solution.
Question 4.
(a) The percentage composition of a gas is: [2]
Nitrogen 82.35%, Hydrogen 17.64%
Find the empirical formula of the gas. [N = 14, H = 1]
Ans. Symbol Percentage Atomic wt. Relative Ratio Simplest Ratio

82.35 5.88
Nitrogen 82.35% 14  5.88 1
14 5.88

17.64 17.64
Hydrogen 17.64% 1  17.64 3
1 5.88

Empirical Formula = N1H3 = NH3


(b) Aluminium carbide reacts with water according to the following equation: [4]
Al4C3 + 12H2O  4Al(OH)3 + 3CH4
(i) What mass of aluminium hydroxide is formed from 12 g of aluminium carbide?
(ii) What volume of methane at s.t.p. is obtained from 12 g of aluminium carbide?
[Relative molecular weight of Al4Cl3 = 144; Al(OH)3 = 78]
Ans. (i) Al4C3 + 12H2O  4Al(OH)3 + 3CH4
[4 × 27 + 3 × 12] 4[27 + 17 × 3]
108 + 36 4[27 × 51]
144 g 4 × 78
312 g
144 g of Al4Cl3 produce 312 g of Al(OH)3
312
1 g of Al4C3 produce g of Al(OH)3
144

312
12 g of Al4C3 produce × 12 = 26 g of Al(OH)3
144
(ii) Al4C3 + 12H2O  4Al(OH)3 + 3CH4
144 g 3 × 22.4 l
67.2 l
144 g of Al4C3 will produce 67.2 l of methane
67.2
1 g of Al4C3 will produce l of methane
144
67.2
12 g of Al4C3 will produce × 12 = 5.6 l
144
Ch 182 2018
(c) (i) If 150 cc of gas A contains X molecules, how many molecules of gas B will be
present in 75 cc of B? [2]
The gases A and B all under the same conditions of temperature and
pressure.
(ii) Name the law on which the above problem is based.
Ans. (i) 150 cc of gas A contain X molecules
150 cc of gas B will also contain X molecules
X
75 cc of gas B will contain molecules under the same conditions of temperature
2
and pressure.
(ii) Avogadro’s law
(d) Name the main component of the following alloys: [2]
(i) Brass (ii) Duralumin
Ans. (i) Brass  Copper, zinc and tin.
(ii) Duralumin  Aluminium, copper, magnesium and manganese.
Question 5.
(a) Complete the following table which relates to the homologous series of
hydrocarbons. [6]
General IUPAC name of the Characteristic bond IUPAC name of the
formula homologous series type first member of the series
CnH2n – 2 (A) (B) (C)
CnH2n + 2 (D) (E) (F)

Ans. (A) Alkynes (B) Triple covalent bond


(C) Ethyne (D) Alkane
(E) Single covalent bond (F) Methane
(b) (i) Name the most common ore of the metal aluminium from which the metal is
extracted. Write the chemical formula of the ore. [4]
(ii) Name the process by which impure ore of aluminium gets purified by using
concentrated solution of an alkali.
(iii) Write the equation for the formation of aluminium at the cathode during the
electrolysis of alumina.
Ans. (i) Bauxite (Al2O3·2H2O) (ii) Baeyer’s process
3+ 2–
(iii) Al2O3 2Al + 3O
Cation Anion

(Cathode)
Cathode
Al3+ + 3e–  Al
Ch 183 2018
Question 6.
(a) A compound X (having vinegar like smell) when treated with ethanol in the
presence of the acid Z, gives a compound Y which has a fruity smell. [4]
The reaction is:
z
C2H5OH + X 
 Y + H 2O
(i) Identify Y and Z.
(ii) Write the structural formula of X.
(iii) Name the above reaction

H2SO4
Ans. CH3 COO H + C2H5 OH CH3 COO C2H 5 + H 2O
X Z Ethyl ethanoate

(i) Y is ethyl ethanoate and Z is concentrated sulphuric acid.


H O
(ii) H C C OH
H
(iii) Esterification reaction
(b) Ethane burns in oxygen to form CO2 and H2O according to the equation: [4]
2C2H6 + 7O2  4CO2 + 6H2O
If 1250 cc of oxygen is burnt with 300 cc of ethane.
Calculate:
(i) the volume of CO2 formed.
(ii) the volume of unused O2.
Ans. 2C2H6 + 7O2  4CO2 + 6H2O
2 vol 7 vol
300 cc
(i) 2C2H6 + 7O2  4CO2 + 6H2O
2 vol 4 vol
300 cc
2 vol. of ethane forms 4 volumes of carbon dioxide
4
1 vol. of ethane forms volumes of carbon dioxide.
2

4
300 cc will form × 300 = 600 cc of CO2.
2
(ii) 2 vol. of ethane requires 7 volumes of oxygen
Ch 184 2018

7
2 vol. of ethane requires volumes of oxygen
2
7
300 cc of ethane requires × 300 = 1050 cc
2
Total volume of oxygen = 1250 cc
Volume of oxygen used = 1050 cc
Unused oxygen = (1250 – 1050) cc
= 200 cc
(c) Three solutions P, Q and R have pH value of 3.5, 5.2 and 12.2 respectively. [2]
Which one of these is a :
(i) Weak acid? (ii) Strong alkali?
Ans. P = 3.5
Q = 5.2
R = 12.2
1 14
7
Strongly 3.5 5.2 12.2 Strongly
Acidic Basic
weak Strong
acid alkali
(i) Weak acid = Q (ii) Strong alkali = R
Question 7.
(a) Give a chemical test to distinguish between the following pairs of chemicals: [4]
(i) Lead nitrate solutioni and Zinc nitrate solution.
(ii) Sodium chloride solution and Sodium nitrate solution.
Ans. (i) Add ammonium hydroxide to the solutions of lead nitrate and zinc nitrate dropwise
and then in excess.
Lead Nitrate Zinc Nitrate
1. On adding ammonium hydroxide chalky 1. On adding ammonium hydroxide
white precipitates of lead hydroxide are gives colourless precipitates of zinc
formed. hydroxide are formed.
Pb(NO3)2 + 2NH4OH  Pb(OH)2 Zn(NO3)2 + 2NH4OH  Zn(OH)2
+ 2NH4NO3 + 2NH4NO3
2. On adding excess of ammonium 2. On adding excess of ammonium
hydroxide, the precipitates do not hydroxide, the precipitates dissolves
dissolve. forming a soluble complex.
Zn(OH)2 + 2NH4NO3 + 2NH4OH 
[Zn(NH3)4] (NO3)2 + 4H2O
Ch 185 2018
(ii) Add silver nitrate solution to both sodium chloride and sodium nitrate solution and
observe.
Sodium Chloride Sodium Nitrate
1. Add silver nitrate solution to sodium 1. Add silver nitrate solution to sodium
chloride nitrate.
NaCl + AgNO3  AgCl  + NaNO3 No reaction will take place.
You will observe curdy white NaNO3 + AgNO3  No reaction
precipitates of silver chloride.

(b) Write a balanced equation for the preparation of each of the following salts: [2]
(i) Copper sulphate from Copper carbonate
(ii) Zinc carbonate from Zinc sulphate.
Ans. (i) CuCO3 + H2SO4  CuSO4 + H2O + CO2 
(ii) ZnSO4 + Na2SO3  ZnCO3 + Na2SO4
(c) (i) What is the type of salt formed when the reactants are heated at a suitable
temperature for the preparation of Nitric acid? [2]
(ii) State why for the preparation of Nitric acid, the complete apparatus is made
up of glass.
Ans. (i) Acid salt.
(ii) All glass apparatus is used because nitric acid vapours are corrosive and may attack
rubber, cork or metal.
(d) Which property of sulphuric acid is shown by the reaction of concentrated sulphuric
acid with : [2]
(i) Ethanol? (ii) Carbon?
Ans. (i) Dehydrating agent
conc. H2SO4
C2H5OH 160°170°C
C2H4
ethanol ethene
(ii) Oxidising agent
C + 2H2SO4  CO2 + 2H2O + 2SO2
ICSE QUESTION PAPER-2019 (SOLVED)
PAPER 2 (CHEMISTRY)
(Two hours)
Answers to this Paper must be written on the paper provided separately.
You will not be allowed to write during the first 15 minutes.
This time is to be spent in reading the Question Paper.
The time given at the head of this paper is the time allowed for writing the answers.

Section I is compulsory. Attempt any four questions from Section II.


The intended marks for questions or parts of questions are given in brackets [ ].

SECTION - I (40 MARKS)


(Attempt all questions from this Section)
Question 1.
(a) Choose the correct answer from the options given below: [5]
(i) An electrolyte which completely dissociates into ions is:
A. Alcohol B. Carbonic acid
C. Sucrose D. Sodium hydroxide
(ii) The most electronegative element from the following elements is:
A. Magnesium B. Chlorine
C. Aluminium D. Sulphur
(iii) The reason for using Aluminium in the alloy duralumin is:
A. Aluminium is brittle. B. Aluminium gives strength.
C. Aluminium brings lightness. D. Aluminium lowers melting point.
(iv) The drying agent used to dry HCl gas is:
A. Conc. H2SO4 B. ZnO
C. A12O3 D. CaO
(v) A hydrocarbon which is a greenhouse gas is:
A. Acetylene B. Ethylene
C. Ethane D. Methane
(b) Fill in the blanks with the choices given in brackets: [5]
(i) Conversion of ethanol to ethene by the action of concentrated sulphuric acid
is an example of____________.
(dehydration / dehydrogenation / dehydrohalogenation)

Ch 186 2019


Ch 187 2019
(ii) When sodium chloride is heated with concentrated sulphuric acid below 200°C,
one of the products formed is____________. (sodium hydrogen sulphate /
sodium sulphate / chlorine)
(iii) Ammonia reacts with excess chlorine to form____________. (nitrogen /
nitrogen trichloride / ammonium chloride)
(iv) Substitution reactions are characteristic reactions of ____________.
(alkynes / alkenes / alkanes)
(v) In Period 3, the most metallic element is ____________.
(sodium /magnesium / aluminium)
(c) Write a balanced chemical equation for each of the following reactions: [5]
(i) Reduction of copper (II) oxide by hydrogen
(ii) Action of dilute sulphuric acid on sodium hydroxide
(iii) Action of dilute sulphuric acid on zinc sulphide
(iv) Ammonium hydroxide is added to ferrous sulphate solution
(v) Chlorine gas is reacted with ethene.
(d) State one observation for each of the following: [5]
(i) Concentrated nitric acid is reacted with sulphur.
(ii) Ammonia gas is passed over heated copper (II) oxide
(iii) Copper sulphate solution is electrolysed using copper electrodes
(iv) A small piece of zinc is added to dilute hydrochloric acid
(v) Lead nitrate is heated strongly in a test tube.
(e) (i) Calculate : [5]
1. The number of moles in 12g of oxygen gas. [O = 16]
2. The weight of 1022 atoms of carbon. [C = 12, Avogadro’s No. = 6 × 1023]
(ii) Molecular formula of a compound is C6H18O3. Find its empirical formula.
(f) (i) Give the IUPAC name of the following organic compounds: [5]
H H O
| | ||
1. H—C—C=C—H 2. H—C—C—H
| |
H H
(ii) What is the special feature of the structure of ethyne?
(iii) Name the saturated hydrocarbon containing two carbon atoms.
(iv) Give the structural formula of Acetic acid.
(g) Give the appropriate term defined by the statements given below: [5]
Ch 188 2019
(i) The formula that represents the simplest ratio of the various elements
present in one molecule of the compound.
(ii) The substance that releases hydronium ion as the only positive ion when
dissolved in water.
(iii) The tendency of an atom to attract electrons towards itself when combined
in a covalent compound.
(iv) The process by which certain ores, specially carbonates, are converted to
oxides in the absence of air.
(v) The covalent bond in which the electrons are shared equally between the
combining atoms.
(h) Arrange the following according to the instructions given in brackets: [5]
(i) K, Pb, Ca, Zn. (In the increasing order of the reactivity)
(ii) Mg2+, Cu2+, Na1+, H1+ (In the order of preferential discharge at the cathode)
(iii) Li, K, Na, H (In the decreasing order of their ionization potential)
(iv) F, B, N, O (In the increasing order of electron affinity)
(v) Ethane, methane, ethene, ethyne.
(In the increasing order of the molecular weight) [H = 1, C = 12]
Answer :
(a) (i) D Sodium hydroxide (ii) B Chlorine
(iii) C Aluminium brings lightness (iv) A Conc. H2SO4
(v) D Methane
(b) (i) Dehydration (ii) Sodium hydrogen sulphate
(iii) Nitrogen trichloride (iv) Alkanes
(v) Sodium
(c) (i) CuO + H2  Cu + H2O
(ii) 2NaOH + H2SO4 (dil)  Na2SO4 + 2H2O
(iii) ZnS + H2SO4 (dil) ZnSO4 + H2S
(iv) FeSO4 + 2NH4OH Fe(OH)2 + (NH4)2SO4
(v) C2H4 + Cl2  C2H4Cl2
(d) (i) Reddish brown gas is observed
(ii) Black solid copper oxide changes into reddish brown solid
(iii) At cathode : Pink or reddish brown Cu is deposited
At anode : anode keep dissolving
Or Size of cathode increases and size of anode decreases.
Or (iv) a gas released which burns with pop sound
(v) Reddish brown gas or yellow oxide i.e. PbO
Ch 189 2019
(e) (i) 1. GMM O2 = 32 gm
32 gm of O2 contains = 1 mole molecules
1
1 gm of O2 contains =
32

1 3
12 gm of O2 contains =  12   0  375
32 8
23
2. 6  10 atoms of carbon has mass = 12 gm
12
1 atom of carbon has mass =
6  1023

12 2
1022 atoms of carbon has mass =  1022   0  2gm
6  1023 10
(ii) Empirical formula of C6H18O3 = C2H6O1
(f) (i) Propyne (ii) Ethanal (iii) Triple bond
H O
| ||
(iv) Ethane C2H6 (v) H—C—C—O—H
|
H
(g) (i) Empirical formula (ii) Acid (iii) Electronegativity
(iv) Calcination (v) Non polar covalent bond
(h) (i) Pb < Zn < Ca < K (ii) Na+ < Mg2+ < H+ < Cu2+
(iii) H > Li > Na > K (iv) B < N < O < F
(v) CH4 < C2H2 < C2H4 < C2H6

SECTION II (40 Marks)


Attempt any four questions from this Section
Question 2.
(a) Draw the electron dot structure of: [3]
(i) Nitrogen molecule [N = 7]
(ii) Sodium chloride [Na = 11, Cl = 17]
(iii) Ammonium ion [N = 7, H = l]
(b) The pH values of three solutions A, B and C are given in the table. Answer the
following questions: [3]
Ch 190 2019

Solution pH value
A 12
B 2
C 7
(i) Which solution will have no effect on litmus solution?
(ii) Which solution will liberate CO2 when reacted with sodium carbonate?
(iii) Which solution will turn red litmus solution blue?
(c) Study the extract of the Periodic Table given below and answer the questions
that follow. Give the alphabet corresponding to the element in question. DO
NOT repeat an element. [4]

A
C D E
B G F

(i) Which element forms electrovalent compound with G?


(ii) The ion of which element will migrate towards the cathode during electrolysis?
(iii) Which non-metallic element has the valency of 2?
(iv) Which is an inert gas?
Answer.
(a) (i) Nitrogen molecules N2
N N N N

(ii) Sodium chloride NaCl Na Cl Na+ — Cl –

(iii) Ammonium ion +


H

H N H
H

(b) (i) C (ii) B (iii) A


(c) (i) B (ii) C (iii) E(iv) F
Question 3.
(a) Name the particles present in: [3]
Ch 191 2019
(i) Strong electrolyte (ii) Non- electrolyte (iii) Weak electrolyte
(b) Distinguish between the following pairs of compounds using the reagent given in
the bracket. [3]
(i) Manganese dioxide and copper (II) oxide, (using concentrated HCl)
(ii) Ferrous sulphate solution and ferric sulphate solution, (using sodium
hydroxide solution)
(iii) Dilute hydrochloric acid and dilute sulphuric acid, (using lead nitrate solution)
(c) Choose the method of preparation of the following salts, from the methods given
in the list: [4]
[List: A. Neutralization B. Precipitation C. Direct combination D. Substitution]
(i) Lead chloride (ii) Iron (II) sulphate
(iii) Sodium nitrate (iv) Iron (III) chloride
Answer.
(a) (i) Strong electrolyte — only ions
(ii) Non electrolyte — only molecules
(iii) Weak electrolyte — ions as well as molecules
(b) (i) MnO2 and CuO (using conc. HCl)
MnO2 + Conc. 4HCl  MnCl2 + 2H2O + Cl2
[greenish yellow gas chlorine is obserbved]
CuO + 2HCl  CuCl2 + H2O
[but not coloured gas is observed]
(ii) FeSO4 and Fe(SO4)3 (using NaOH sol.)
FeSO4 + 2NaOH (sol.)  Fe(OH)2 + Na2SO4
dirty green ppt.
Fe2(SO4)3 + 6NaOH  2Fe(OH)3 + 3Na2.SO4
[Reddish brown ppt.]
(iii) Dil. HCl and Dil. H2SO4 (using Pb (NO3)2)
Pb(NO3)2 + 2HCl  PbCl2 + 2HNO3
white ppt. of PbCl2 dissolve on heating
Pb(NO3)2 + H2SO4  PbSO4 + 2HNO3
[Insoluble ppt. of PbSO4]
(c) (i) Lead chloride — Precipitation (ii) Iron (II) sulphate — Substitution
(iii) Sodium Nitrate — Neutralization (iv) Iron (III) chloride — Direct combination
Question 4.
(a) Complete the following equations: [3]
Ch 192 2019
(i) S + conc. HNO3  (ii) C + conc. H2SO4 
(iii) Cu + dil. HNO3 
(b) Write a balanced chemical equation for the preparation of: [3]
(i) Ethene from bromoethane (ii) Ethyne using calcium carbide
(iii) Methane from sodium acetate.
(c) Name the following organic compounds: [4]
(i) The compound with 3 carbon atoms whose functional group is a carboxyl.
(ii) The first homologue whose general formula is CnH2n.
(iii) The compound that reacts with acetic acid to form ethyl ethanoate.
(iv) The compound formed by complete chlorination of ethyne.
Answer.
(a) (i) S + Conc. 6HNO3  H2SO4 + 2H2O + 6NO2
(ii) C + Conc. 2H2SO4  CO2 + 2H2O + 2SO2
(iii) 3Cu + dil. 8HNO3  3Cu(NO3)2 + 4H2O + 2NO
(b) (i) Ethene from bromoethane
C2H5Br + KOH  C2H4 + KBr + H2O
Bromothane alcoholic ethene
(ii) Ethyne using calcium carbide
CaC2 + 2H2O  Ca(OH)2 + C2H2
Calcium carbide ethyne
(iii) Methane from sodium acetate
CH3COONa + NaOH  CH4 + Na2CO3
Sodium acetate Methane
O
||
(c) (i) CH3CH2 – C – OH — Propanoic acid
(ii) C2H4 — Ethene
(iii) C2H5OH — Ethanol
Cl Cl
| |
(iv) 1, 1, 2, 2, tetrachloroethane — H – C – C – H
| |
Cl Cl
Ch 193 2019
Question 5.
(a) Give the chemical formula of : [3]
(i) Bauxite (ii) Cryolite (iii) Sodium aluminate
(b) Answer the following questions based on the extraction of aluminium from [3]
alumina by Hall-Heroult’s Process.:
(i) What is the function of cryolite used along with alumina as the electrolyte?
(ii) Why is powdered coke sprinkled on top of the electrolyte?
(iii) Name the electrode, from which aluminium is collected.
(c) Match the alloys given in column I to the uses given in column II: [4]
COLUMN I COLUMN II
(i) Duralumin A. Electrical fuse
(ii) Solder B. Surgical instruments
(iii) Brass C. Aircraft body
(iv) Stainless Steel D. Decorative articles
Answer.
(a) (i) Bauxite — Al2O32H2O (ii) Cryolite — Na3AlF6
(iii) Sodium aluminate — Na3AlO3
(b) (i) 1. to lower the fusion temperature of alumina
2. Enhance conductivity of Alumina
(ii) 1. prevent burning of anode
2. reduces the heat loss by radiation
(iii) Cathode
(c) Match the alloys
(i) Duralumin — C Aircraft body
(ii) Solder — A Electric fuse
(iiii) Brass — D Decorative material
(iv) Stainless steel — B Surgical instrument
Question 6.
(a) Identify the substances underlined: [3]
(i) The catalyst used to oxidise ammonia.
(ii) The organic compound which when solidified, forms an ice like mass.
(iii) The dilute acid which is an oxidizing agent.
(b) Copper sulphate solution reacts with sodium hydroxide solution to form a
precipitate of copper hydroxide according to the equation: [3]
Ch 194 2019
2NaOH + CuSO4  Na2SO4 + Cu (OH)2 
(i) What mass of copper hydroxide is precipitated by using 200 gm of sodium
hydroxide? [H = 1, O = 16, Na = 23, S = 32, Cu = 64]
(ii) What is the colour of the precipitate formed?
(c) Find the empirical formula and the molecular formula of an organic compound
from the data given below: [4]
C = 75.92%, H = 6.32% and N = 17.76%.
The vapour density of the compound is 39.5. [C=12, H=l, N=14]
Answer.
(a) (i) Platinum
(ii) Acetic acid or Ethanoic acid, (CH3COOH)
(iii) Nitric acid
(b) (i) 2NaOH + CuSO4  Na2SO4 + Cu(OH)2
2  40 = 80 gm 98 gm
80 gm of NaOH is used to produce ppt. Cu(OH)2 =98 gm

98
1 gm of NaOH is used to produce ppt. =
80

98
200 gm of NaOH is used to produce ppt. =  200  245gm.
80
(ii) the colour of ppt. is pale blue.
(c) Element %age Atomic Atomic Simplest
mass ratio ratio

75  92 6  32
C 75.92 12  6  32 5
12 1 27

6  32 6  32
H 6.32 1  6.32 5
1 1 27

17  76 1 27
N 17.76 14  1.27 1
14 1.27
Empirical formula = C5H5N.
Empirical formula mass = 5  12 + 5  1 + 14 = 60 + 5 + 14 = 79
Vapour density = 39.5
Molecular mass = 2  39.5 = 79
Ch 195 2019

Molecular mass 79
n = Empiricalformula mass = 1
79

So, Empirical formula is Molecular formula C5H5N.


C5 H5 N
H H H
H | | |
C=C–C=C–C  N
H
2, 4-Pentadienenitrile
Question 7.
(a) Name the gas evolved in each of the following cases: [3]
(i) Alumina undergoes electrolytic reduction.
(ii) Ethene undergoes hydrogenation reaction.
(iii) Ammonia reacts with heated copper oxide.
(b) Study the flow chart given and give balanced equations to represent the
reactions A, B and C : [3]
B
A
Mg 3 N 4  
 NH3  NH 4 Cl
C

(c) Copy and complete the following table which refers to the industrial method
for the preparation of ammonia and sulphuric acid: [4]
Name of the compound Name of the process Catalytic equation
(with the catalyst)
Ammonia (i)____________ (ii)____________
Sulphuric acid (iii)___________ (iv)___________
Answer.
(a) Carbon dioxide (b) Ethane (c) Nitrogen
(b) M3N2 + 6H2O  3Mg(OH)2 + 2NH3
(A)
NH3 + HCl  NH4Cl
(B)

NH4Cl + heat NH3 + HCl


(C)
Ch 196 2019
(c) Ammonia
(i) Haber’s Process

Fe (catalyst)/Mo (promoter)
  2NH3 + heat

(ii) Na2 + 3H2 
450C — 500°C above 200 atm.

Sulphuric acid
(i) Contact Process
(ii) S + O2  SO2

2 5 V O /Pt. V O /Pt.
2 5
Catalytic eqn.  SO2 + O2  
 SO3 or SO2 + O2  
400–450°C 2SO3
SO3 + H2SO4  H2S2O7
H2S2O7 + H2O  2H2SO4
ICSE QUESTION PAPER-2020 (SOLVED)
PAPER 2 (CHEMISTRY)
(Two hours)
Answers to this Paper must be written on the paper provided separately.
You will not be allowed to write during the first 15 minutes.
This time is to be spent in reading the Question Paper.
The time given at the head of this paper is the time allowed for writing the answers.

Section I is compulsory. Attempt any four questions from Section II.


The intended marks for questions or parts of questions are given in brackets [ ].

SECTION - I (40 MARKS)


(Attempt all questions from this Section)
Question 1.
(a) Choose the correct answer from the options given below: [5]
(i) The element with highest ionization potential is :
A. Hydrogen B. Caesium
C. Radon D. Helium
(ii) The inert electrode used in the electrolysis of acidified water, is :
A. Nickel B. Platinum
C. Copper D. Silver
(iii) A compound with low boiling point, is :
A. Sodium chloride B. Calcium chloride
C. Potassium chloride D. Carbon tetrachloride
(iv) The acid which can produce carbon from cane sugar, is :
A. Concentrated Hydrochloric acid B. Concentrated Nitric acid
C. Concentrated Sulphuric acid D. Concentrated Acetic acid
(v) The organic compound having a triple carbon-carbon covalent bond, is
A. C3H4 B. C3H6
C. C3H8 D. C4H10
(b) State one relevant observation for each of the following reactions : [5]
(i) Action of concentrated nitric acid on copper.
(ii) Addition of excess ammonium hydroxide into copper sulphate solution.
(iii) A piece of sodium metal is put into ethanol at room temperature.
Ch 197 2019
Ch 198 2019
(iv) Zinc carbonate is heated strongly.
(v) Sulphide ore is added to a tank containing oil and water and then strirred or
agitated with air.
(c) Write a balanced chemical equation for each of the following : [5]
(i) Reaction of carbon powder and concentrated nitric acid.
(ii) Reaction of excess ammonia with chloride.
(iii) Reaction of lead nitrate solution with ammonium hydroxide.
(iv) Producing ethane from bromoethane using Zn\Cu couple in alcohol.
(v) Complete combustion of ethane.
(d) (i) Draw the structural formula for each of the following : [5]
1. 2, 2 dimethyl pentane
2. methanol
3. Iso propane
(ii) Write the IUPAC name for the following componds :
1. acetaldehyde 2. acetylene
(e) State one relevant reason for each of the following : [5]
(i) Graphite anode is preferred to platinum in the electrolysis of molten lead
bromide.
(ii) Soda lime is preferred to sodium hydroxide in the laboratory preparation of
methane.
(iii) Hydrated copper sulphate crystals turn white on heating.
(iv) Concentrated nitric acid appears yellow, when it is left for a while in a glass
bottle.
(v) Hydrogen chloride gas fumes in moist air.
(f) Calculate : [5]
(i) The amount of each reactant required to produce 750 ml of carbon dioxide,
when two volumes of carbon monoxide combine with one volume of oxygen
to produce two volumes of carbon dioxide.
2CO + O2  2CO2
(ii) The volume occupied by 80 g of carbon dioxide at STP.
(iii) Calculate the number of molecules in 4.4 gm of CO2.
[Atomic mass of C = 12, O = 16]
(iv) State the law associated in question no. (f) (i) above.
(g) Give one word or a phrase for the following statements : [5]
(i) The chemical bond formed by a shared pair of electrons, each bonding atom
contributing one electron to the pair.
Ch 199 2019
(ii) Electrode used as cathode in electrorefining of impure copper.
(iii) The substance prepared by adding other metals to a base metal in appropriate
proportions to obtain certain desirable properties.
(iv) The tendency of an atom to attract electrons to itself when combined in a
compound.
(v) The reaction in which carboxylic acid reacts with alcohol in the presence of
conc. H2SO4 to form a substance having a fruity smell.
(h) Fill in the blanks from the choices given in brackets : [5]
(i) The polar covalent compound in gaseous state that does not conduct
electricity is ....................... (carbon tetrachloride, ammonia, methane)
(ii) A salt prepared by displacement reaction is .....................
(ferric chloride, ferrous chloride, silver chloride)
(iii) The number of moles in 11gm of nitrogen gas is .....................
(0.39, 0.49, 0.29) [atomic mass of N = 14]
(iv) An alkali which completely dissociates into ions is .....................
(ammonium hydroxide, calcium hydroxide, lithium hydroxide)
(v) An alloy used to make statues is .....................
(bronze, brass, fuse metal)
Answer :
(a) (i) D Helium (ii) B Platinum
(iii) D Carbon tetrachloride (iv) C Concentrated Sulphuric acid
(v) A C3H4
(b) (i) When concentrated nitric acid reacts with copper, a reddish brown gas having pungent
smell is evolved.

3Cu + 4HNO3(conc.)   Cu(NO3)2 + 2H2O + 2NO2
(ii) When excess ammonium hydroxide is added into copper sulphate solution, a deep blue
solution of tetraamine copper (II) sulphate is formed.
CuSO4 + 2NH4OH  Cu(OH)2 + (NH4)2SO4 + 4H2O
Cu(OH)2 + (NH4)2SO4 + 2NH4OH  [Cu(NH3)4]SO4 + 4H2O
Tetraamine copper (II)
sulphate
(iii) When a piece of sodium metal is put into ethanol at room temperature, it gives off
hydrogen gas with bubbles and a colourless solution of sodium ethoxide is formed :
2C2H5OH + 2Na  2C2H5ONa + H2 
Sodium ethoxide
(iv) Zinc carbonate on heating strongly decomposes to zinc oxide and carbon dioxide gas

ZnCO3 
 ZnO + CO2 
Ch 200 2019
(v) When sulphide ore is added to a tank containing oil and water and then stirred or agitated
with air,the froth is formed, the ore particles are wetted by oil and the gangue particles
are wetted by water.
(c) (i) C + 4HNO3(conc.)  CO2 + 4NO2 + 2H2
Nitrogen oxide
(ii) 8NH3(g) + 3Cl2(g)  N2(g) + 6NH4Cl(g)
Nitrogen Ammonium Chloride
(iii) Pb(NO3)2 + 2NH4OH  Pb(OH)2 + 2NH4NO3
Lead (II) hydroxide Ammonium nitrate

(iv) C2H5Br + 2[H] 


Zn/Cu in
alcohol
C2H6 + HBr
(v) 2C2H6 + 7O2  4CO2 + 6H2O + Energy
H CH 3 H H H
| | | | |
(d) (i) (1) 2, 2 dimethyl pentane : H—C—C——C—C—C—H
| | | | |
H CH 3 H H H

H
|
(2) Methanol : H—C—O—H
|
H

H H H
| | |
(3) Iso-propane : H—C———C———C—H
| | |
H H—C—H H
|
H
(ii) (1) Acetaldehyde—Ethanal (2) Acetylene—Ethyne
(e) (i) Graphite anode is preferred to platinum in the electrolysis of molten lead bromide because
graphite is unaffected by the bromine vapour.
(ii) Soda lime is preferred to sodium hydroxide in the laboratory preparation of methane
because sodium hydroxide absorbs water from the atmosphere white soda lime (NaOH +
CaO) is stable and absorbs less amount of water.
(iii) When hydrated copper sulphate is heated it turns white because it loses its crystalline
water and becomes anhydrous.
(iv) When concentrated nitric acid is left for a while in a glass bottle it decomposes to give out
nitrogen dioxide, which dissolves in this and appears yellow in colour.
(v) Hydrogen chloride gas fumes in moist air because it is highly soluble and forms a mist of
droplets of HCl that appears as white fumes.
(f) (i) 2CO + O2  2CO2
2 Volume 1 Volume 2 Volume (750 ml)
At STP volume = 22400 ml
Ch 201 2019

2 × 22400 ml CO is used to form 2 × 22400 ml CO2


2  22400  750
 750 ml CO2 is formed from = 750 ml CO used to form 750 ml of CO2.
2  22400
2 × 22400 CO2 is formed from 1 × 22400 ml O2
1  22400  750
 750 ml CO2 is formed from = 375 ml O2 is used to form 750 ml of CO2.
2  22400
(ii) Molar mass of CO2 = 12 + 16 × 2 = 44
If 44 gm of CO2 contain 22400 l at STP
22400  80
 80 gm of CO2 contains = = 40.72 l.
44

Mass of the substance


(iii) No. of molecules = × N0
Molar mass of the substance
Mass of the substance = 4.4 gm
N0 = 6.022 × 1023
Molar mass of the substance (CO2) = 12 + 16 × 2 = 44 gm
4.4
No. of molecules = × 6.022 × 1023
44
= 0.1 × 6.022 × 1023
= 6.022 × 1022
(iv) Gay Lussac’s law.
(g) (i) Covalent bond (ii) Pure copper metal
(iii) Alloy (iv) Electronegativity
(v) Esterification
(h) (i) ammonia (ii) ferrous chloride

Mass of the substance 11gm


(iii) No. of moles in 11 gm of N2 = = = 0.39
Molar mass 28 gm
(iv) Ca(OH)2, calcium hydroxide
(v) Bronze

SECTION II (40 Marks)


Attempt any four questions from this Section
Question 2.
(a) The following table represents the elements and the atomic number. [3]
With reference to this, answers the following using only the alphabets given in
the table.
Ch 202 2019

Element Atomic number


P 13
Q 7
R 10
(i) with hydrogen to form a basic gas ?
(ii) Which element has an electon affinity zero ?
(iii) Name the element, which forms an ionic compound with chlorine.
(b) Draw the electron dot diagram for the compounds given below. Represent the
electron by (  ) and (×) in the diagram.
[Atomic No.: Ca = 20, O = 8, Cl = 17, H = 1] [3]
(i) Calcium oxide
(ii) Chlorine molecule
(iii) Water molecule
(c) Choose the correct word which refers to the process of electrolysis from A to E,
to match the description (i)to (iv) :
A. Oxidation; B. Cathode; C. Anode; D. An electrolyte; E. Reduction [4]
(i) Conducts electricity in aqueous or in molten state.
(ii) Loss of electron takes place at anode.
(iii) A reducing electrode.
(iv) Electrode connected to the positive end or terminal of the battery.
Answer :

(a) Element Atomic number


P 13 2, 8,3
Q 7 2, 5
R 10 2, 8

(i) Q (The base formed is NH3 because it dissolves in water and forms basic
solution of ammonium hydroxide.)
(ii) R (because it has complete octet)
(iii) P (It has 3 valence electrons which are completely transferred to chlorine and
form trichloride)
(b) Electronic configuration of elements :
20Ca = 2, 8, 8, 2 8O = 2, 6
17Cl = 2, 8, 7 1H = 1
(i) Calcium oxide (CaO)
2–
2+
Ca + O  Ca O
Ch 203 2019
(ii) Chlorine molecule (Cl2)

Cl Cl  Cl—Cl

(iii) Water molecule (H2O)

H O H  H—O—H or O
H H
(c) (i) An electrolyte (ii) Oxidation
(iii) Anode (iv) Cathode
Question 3.
(a) Baeyer’s process is used to concentrate bauxite to alumina. [3]
Give balanced chemical equations for the reaction taking place for its conversion
from bauxite to alumina.
(b) Complete the following by selecting the correct option from the choices given:[3]
(i) pH of acetic acid is greater than dilute sulphuric acid. So acetic acid contains
....................... concentration of H+ ions. (greater, same, low)
(ii) The indicator which does not change colour on passage of HCl gas is
....................... (methyl orange, moist blue limits, phenolphthalein)
(iii) The acid which cannot act as an oxidizing agent is .......................
(conc. H2SO4, conc. HNO3, conc. HCl)
(c) Match the gases given in column I to the identification of the gases mentioned
in column II. [4]
Column I Column II
(i) Hydrogen sulphide A. Turns acidified potassium dichromate
solution green.
(ii) Nitric oxide B. Turns lime water milky.
(iii) Carbon dioxide C. Turns reddish brown when it reacts with oxygen.
(iv) Sulphur dioxide D. Turns moist lead acetate paper silvery black.
Answer :
(a) Bayer’s method for extraction of alumina from bauxite ore.
Al2O3.2H2O 
NaOH
 2NaAlO2 + 3H2O
Bauxite Sodium aluminate
NaAlO2 + 2H2O  NaOH + Al(OH)3
Aluminium hydroxide

2Al(OH)3 
143 K
 Al2O3 + 3H2O
Alumina (pure)
(b) (i) low (ii) phenolphthalein (iii) Conc. HCl
Ch 204 2019
(c)
Column I Column II
(i) Hydrogen sulphide Turns moist lead acetate paper to silvery black.
(ii) Nitric oxide Turns reddish brown when it reacts with oxygen.
(iii) Carbon dioxide Turns lime water milky.
(iv) Sulphur dioxide

Question 4.
(a) Differentiate between the following pairs based on the information given in the
brackets. [3]
(i) Conductor and electrolyte (conducting particles)
(ii) Cations and anions (formation from an atom)
(iii) Acid and Alkali (formation of type of ions)
(b) Draw the structure of isomers of pentane. [3]
(c) Hydrogen chloride gas is prepared in the laboratory using concentrated sulphuric
acid and sodium chloride. Answer the questions that follow based on this reaction.
[4]
(i) Give the balanced chemical equation for the reaction with suitable
condition(s), if any.
(ii) Why is concentrated suphuric acid used instead of concentrated nitric acid.
(iii) How is the gas collected ?
(iv) Name the drying agent not used for drying the gas.
Answer :
(a) (i) Conductor Electrolyte
Conducting particles are free electrons. Conducting particles are ions.

(ii) Cations Anions


These are positively charged species which are These are negatively charged species which are
formed by loss of electrons. e.g. Na + , Ca 2+ , etc. formed by gain of electrons. e.g. Cl –, SO4 2–, etc.

(iii) Acid Alkali


The compound which when dissolved in water The compound which when dissolved in water
gives hydronium ion (H + or H 3O + ). gives hydroxide ion (OH –).

(b) Molecular formula of pentane C5H12 isomers of pentane are :

(i) H H H H H
| | | | |
H—C—C—C—C—C— H n-pentane
| | | | | (Pentane)
H H H H H
Ch 205 2019

H H H H
(ii) | | | |
H—C———C———C—C—H Iso-pentane
| | | | (2-methyl butane)
H H—C—H H H
|
H

H
(iii) |
H H—C—H H
| | |
H—C———C———C—H Neopentane
| | | ( 2, 2-dimethyl propane)
H H—C—H H
|
H

above
(c) (i) 2 NaCl + H2SO 4 NaH SO4 + 2 HCl
Rock salt conc 200° C Sodium sulphate Hydrogen
chloride gas

below
Or NaCl + H2SO4 NaHSO4 + HCl
200°C
(ii) Conc. sulphuric acid acts as dehydrating agent while nitric acid is a strong oxidising
agent.
(iii) The gas is collected by upward displacement of air as it is heavier than air.
(iv) Phosphorous pentaoxide (P2O5) and calcium oxide (CaO).
Question 5.
(a) Distinguish between the following pairs of compounds using a reagent as a
chemical test : [3]
(i) Calcium nitrate and Zinc nitrate solution.
(ii) Ammonium sulphate crystals and Sodium sulphuric crystals.
(iii) Magnesium chloride and Magnesium nitrate solution.
(b) Calculate the percentage of : [3]
(i) Fluorine
(ii) Sodium and
(iii) Aluminium
in sodium aluminium fluoride [Na3AlF6], to the nearest whole number.
[Atomic mass : Na = 23, Al = 27, F = 19]
(c) (i) State the volume occupied by 40 gm of methane at STP, if its vapour density
(V.D.) is 8. [4]
(ii) Calculate the number of moles present in 160 gm of NaOH.
[Atomic mass : Na = 23, H = 1, O = 16].
Ch 206 2019
Answer :
(a) (i) By ammonium hydroxide

Calcium nitrate Zinc nitrate


When ammonium hydroxide is added in excess When ammonium hydroxide is added in excess
no precipitate of calcium hydroxide occurs. white gelatinous precipitate of zinc hydroxide is
formed which is soluble in excess NH4OH.

(ii) By sodium hydroxide

Ammonium sulphate crystals Sodium sulphate crystals


When sodium hydroxide is added white Sodium sulphate does not react with sodium
gelatinous precipitate of zinc hydroxide is hydroxide.
formed.

(iii) By silver nitrate

Magnesium chloride Magnesium nitrate


On adding silver nitrate solution white On adding silver nitrate no reaction occurs.
precipitate of silver chloride is formed.

(b) 3AlF6].
= 3 × 23 +27 + 6 × 19 [Atomic mass : Na = 23, Al = 27, F = 19]
= 69 + 27 + 114
= 210 gm.

Mass of the fluorine


(i) Percentage of fluorine = Molecular mass of Na AlF × 100
3 6

19  6
= 100  54.29%
210
It has to be rounded up to nearest whole number so 54%.
Mass of the sodium
(ii) Percentage of sodium = Molecular mass of Na AlF × 100
3 6

23  3
=  100  32.5%
210
It has to be rounded up to nearest whole number so 33%.
Mass of the aluminium
(iii) Percentage of aluminium = × 100
Molecular mass of Na 3AlF6
27
=  100  12.85%
210
It has to be rounded up to nearest whole number so 13%.
Ch 207 2019
(c) (i) V.D. of Methane = 8
Molecular mass of methane (CH4) = 2 × V.D = 16 gm
16 gm of methane occupies of 22.4 l at STP
22.4
1 gm of methane occupies a volume of
16

22.4
40 gm of methane occupies a volume of × 40 = 56 gm
16
(ii) Mass of the NaOH = 160 gm
Molecular mass of NaOH = 23 + 16 + 1 = 40

Mass of the substance 160


Number of moles = = = 4 moles
Molecular mass 40
Question 6.
(a) Identify the salts P, Q, R from the following observations : [3]
(i) Salt P has light bluish green colour. On heating , it produces a black coloured
residue. Salt P produces brisk effervescence with dil. HCl and the gas
evolved turns lime water milky, but no action with acidified potassium
dichromatic solution.
(ii) Salt Q is white in colour. On strong heating, it produces buff yellow residue
and liberates reddish brown gas. Solution of salt Q produces chalky white
insoluble precipitate with excess of ammonium hydroxide.
(iii) Salt R is black in colour. On reacting with concentrated HCl, it liberates a
pungent greenish yellow gas which turns moist starch iodide paper blue
black.
(b) Identify the substance underlined in each of the following : [3]
(i) The electrode that increases in mass during the electro-refining of silver.
(ii) The acid that is a dehydrating as well as a drying agent.
(iii) The catalyst used to oxidize ammonia into nitric oxide.
(c) Copy and complete the following paragraph using the options given in brackets :
[4]
Alkenes are a homologous series of (i) ....................... (saturated / unsaturated)
hydrocarbons characterized by the general formula (ii) ....................... (CnH2n+2
/ CnH2n), Alkenes undergo (iii) ....................... (addition / substitution) reactions
and also undergo (iv) ....................... (hydrogenation / dehydrogenation) to form
alkanes.
Ch 208 2019
Answer :

(a) (i) Black residue


P Dil. HCl Brisk effervescence  Lime water turns milky
Light blue
No action
K2Cr2O7

Copper salts are blue in colour. It produces a gas with HCl, which turns lime water milky.
It must be a carbonate. Hence, P is copper carbonate.

CuCO3   CuO + CO2
Black
CuCO3 + HCl  CuCl2 + CO2  + H2O
dil.

CO2 + Ca(OH)2  CaCO3  + H2O


Lime water milky solution

(ii)

Nitrate compounds always produce reddish brown gas. Buff yellow ppt. is of lead oxide.
Hence, Q is lead nitrate.

2 Pb ( NO 3 )2 (g ) 2 Pb O + 4 NO 2 (g ) + O 2 (g )
Lead nitrate Lead oxide Nitrogen
white yello w d ioxid e
(Brown fumes)

Pb(NO3)2 + 2NH4OH  Pb(OH)2  + 2NH4NO3


(Chalky white)

(iii) R 

conc. HCl
greenish yellow gas 
Moist starch iodide
 Blue black
Black
The evolved gas is greenish yellow gas, this confirms that the gas is chlorine.
Black salt is manganese dioxide which reacts with HCl to produce chlorine gas.
MnO2(s) + 4HCl(aq.)  Cl2(g) + MnCl2(aq) + 2H2O
greenish yellow
Cl2(g) + Starch iodide paper Blue black
(Moist)
(b) (i) Cathode (ii) Conc. H2SO4 (iii) Pt
Ch 209 2019
(c) (i) unsaturated (ii) CnH2n
(iii) addition (iv) hydrogenation
Question 7.
(a) Write balanced chemical equations, for the preparation of the given salts (i) to
(iii) by using the methods A to C respectively :
A. Neutralization B. Precipitation C. Titration [3]
(i) Copper sulphate
(ii) Zinc carbonate
(iii) Ammonium sulphate
(b) Name the following elements : [3]
(i) An alkaline earth metal present in group 2 and period 3.
(ii) A trivalent metal used to make light tools.
(iii) A monovalent non-metal present in fluorspar.
(c) An aqueous solution of nickel (II) sulphate was electrolyzed using nickel
electrodes. Observe the diagram and answer the question that follow : [4]

Power supply

– +
Nickel Nickel
cathode anode

Aqueous
nickel (II) sulphate

(i) What do you observe at the cathode and anode respectively ?


(ii) Name the cation that remains as a spectator ion in the solution.
(iii) Which equation for the reaction at the anode is correct ?
1. Ni  Ni2+ + 2e– 2. Ni + 2e–  Ni2+
3. Ni2+  Ni + 2e– 4. Ni2+ + 2e–  Ni
Answer.
(a) (i) Copper sulphate—by neutralization :
CuO + H2SO4  CuSO4 + H2O
Copper oxide dil. Copper sulphate
(Base) (Acid)
Ch 210 2019
(ii) Zinc carbonate—by precipitation
Na2CO3 + ZnSO4  ZnCO3 + Na2SO4
Sodium carbonate Zinc sulphate Zinc carbonate Sodium sulphate

(iii) Ammonium sulphate—by titration


H2SO4 + 2NH4OH  (NH4)2SO4 + 2H2O
Strong acid Weak base Ammonium sulphate
(b) (i) Mg (Magnesium) (Atomic number : 12; electronic configuration : 2, 8, 2)
(ii) Al (Aluminium) (Atomic number : 13; electronic configuration : 2, 8, 3)
(iii) F (Fluorine) (Atomic number : 9; electronic configuration : 2, 7)
(c) (i) At cathode-reduction and at anode oxidation take place.
(ii) H+ ion as a spectator ion.
(iii) 1. Ni  Ni2+ + 2e–.
ICSE QUESTION PAPER-2012 (SOLVED)
Answers to this Paper must be written on the paper provided separately.
You will not be allowed to write during the first 15 minutes.
This time is to be spent in reading the Question Paper,
The time given at the head of this Paper is the time allowed for writing the answers.
Attempt all questions from Section I and any four questions from Section II.
The intended marks for questions or parts of questions are given in brackets [ ].
SECTION–I (40 MARKS)
(Attempt all questions from this section)
Question 1.
(a) Name the following :
(i) The phenomenon by which living or dead plant cells absorb water by surface attraction.
(ii) The phase of cardiac cycle in which the auricles contract.
(iii) The organ where urea is produced.
(iv) The hormone that helps increase the reabsorption of water from the kidney tubules.
(v) Chemical substances produced by micro organisms that can kill or inhibit the growth of
other micro organisms. [5]
(b) Choose the correct answer from the four options given below each statement :
(i) BCG vaccine is used to build immunity against :
(a) Poliomyelitis (b) Tuberculosis
(c) Malaria (d) Whooping cough
(ii) A plant is kept in a dark cupboard for about 48 hours before conducting any experiment
on photosynthesis to :
(a) Remove starch from the plant.
(b) Ensure that starch is not translocated from the leaves.
(c) Remove chlorophyll from the leaf of the plant.
(d) Remove starch from the experimental leaf.
(iii) The part of the human eye where rod cells and cone cells are located is the :
(a) Retina (b) Cornea
(c) Choroid (d) Sclera
(iv) A reflex arc in man is best described as movement of stimuli from :
(a) Receptor cell, sensory neuron, relaying neuron, effector muscles.
(b) Receptor cell, efferent nerve, relaying neuron, muscles of the body.
(c) Receptor cell, spinal cord, motor neuron, relaying neuron.
(d) Receptor cell, synapse, motor neuron, relaying neuron.
(v) NADP is expanded as :
(a) Nicotinamide, adenosine dinucleotide phosphate.
Bio 211 2012
Bio 212 2012
(b) Nicotinamide, adenine dinucleotide phosphate
(c) Nicotinamide, adenine dinucleous phosphate
(d) Nicotinamide, adenosine dinucleous phosphate. [5]
(c) State the main function of the following :
(i) Chordae tendinae (ii) Lymphocytes
(iii) Seminiferous tubule (iv) Thylakoids
(v) Beta cells of pancreas [5]
(d) Give the exact location of :
(i) Lenticels (ii) Prostate gland
(iii) Thyroid gland (iv) Centrosome
(v) Mitral valve [5]
(e) given below are sets of five terms each. In each case rewrite the terms in logical sequence as
directed at the end of each statement. An example has been done for you:
Example :
Cortical cells, root hair, xylem, soil water, endodermis (absorption of water by the plants)
Answer : Soil water, root hair, cortical cells, endodermis, xylem.
(i) Active immunity, antigen, antibody, bacteria, lymphocytes (defence mechanisms of the
body).
(ii) Implantation, parturition, ovulation, gestation, fertilisation (stages leading to formation
of foetus and birth).
(iii) Oval window, tympanum, cochlea, auditory canal, ear ossicles (path through which a
vibration of sound is transferred in the human ear).
(iv) Karyokinesis, S-phase, cytokinesis, G1-phase, G2-phase (cell cycle).
(v) Renal vein, renal artery, afferent arteriole, efferent arteriole, glomerulus (pathway of
blood through a glomerulus). [5]
(f) Study the following diagram carefully and then answer the questions that follow. The diagram
is depicting a defect of the human eye :

(i) Identify the defect shown in the diagram.


(ii) Give two possible reasons for the above defect.
(iii) Draw a neat labelled diagram to show how the above defect can be rectified.
(g) Match the items in Column A with that which is most appropriate in Column B. Rewrite the
matching pairs.
Bio 213 2012
Column A Column B
(1) Potometer (a) Antiseptic
(2) Hypothalamus (b) Disinfectants
(3) Formalin (c) Vasectomy
(4) Contraception in males (d) Sudden change in genes
(5) Mutation (e) Pituitary gland
(f) Tubectomy
(g) Transpiration
(h) Thyroid gland
(i) Alleles
(j) Photosynthesis [5]
(h) Given below are six sets with four terms each. In each set a term is an odd one and cannot be
grouped in the same category to which the other three belong. Identify the odd one in each set
and name the category to which the remaining three belong. The first has been done for your
as an example.
No. Set Odd one Category
e.g.: Cell wall, large vacuole, centrosome Parts of plant cell
plastids, centrosome
(i) Cerebrum, cerebellum, thalamus
hypothalamus
(ii) Ovary, ureter, fallopian tube,
(iii) Adrenal gland, liver, thyroid
gland, pituitary gland
(iv) Malleus, pinna, incus, stapes
(v) Haemophilia, colour blindness,
albinism, night blindness

Answer.
(a) (i) Imbibition (ii) Systole
(iii) Liver (iv) Vasopressin (ADH)
(v) Antibiotics
(b) (i) (b) tuberculosis
(ii) (a) Remove starch from the plant
(iii) (a) Retina
(iv) (a) Receptor cell, sensory neuron, relaying neuron, effector muscles.
(v) (b) Nicotinamide, adenine, dinucleotide phosphate
Bio 214 2012
(c) (i) Chordae tendinae connect the papillary muscles to the tricuspid valve and the mitral
valve in the heart.
(ii) Lymphocytes produce antibodies and provide immunity to the body.
(iii) Seminiferous tubule produce sperms (male gametes).
(iv) Thylakoids are the site for light reaction during photosynthesis.
(v) Beta cells of pancreas secrete the hormone insulin.
(d) (i) Lenticels – in the stem of woody plants.
(ii) Prostate gland – a bilobed structure surrounding the urethra close to its origin from the
bladder.
(iii) Thyroid gland– a butterfly shaped gland is situated in front of the neck below the larynx.
(iv) Centrosome – situated next to the nucleus.
(v) Mitral valve – situated between left atrium and the left ventricle of the heart.
(e) (i) Bacteria, lymphocytes, antibody, antigen, active immunity.
(ii) Ovulation, fertilisation, implantation, gestation, parturition.
(iii) Tympanum, ear ossicles, oval window, cochlea, auditory canal.
(iv) Karyokinesis, G1-phase, S-phase, G2-phase, Cytokinesis.
(v) Renal artery, afferent arteriole, glomerulus, efferent arteriole, renal vein.
(f) (i) Long sightedness
(ii) • To flattering of the lens.
• Shortening of the eyeball.
(iii)

(g) (1) — (g) (2) — (e)


(3) — (b) (4) — (c)
(5) — (d)
(h) No. Set Odd one Category
e.g.: Cell wall, large vacuole, centrosome Parts of plant cell
plastids, centrosome
Bio 215 2012

No. Set Odd one Category


(i) Cerebrum, cerebellum, thalamus Cerebellum Part of fore-brain
hypothalamus
(ii) Ovary, ureter, fallopian tube, Ureter Part of female reproductive system
(iii) Adrenal gland, liver, thyroid Liver Endocrine glands
gland, pituitary gland
(iv) Malleus, pinna, incus, stapes Pinna Middle ear
(v) Haemophilia, colour blindness, Night blindness Hereditary, diseases
albinism, night blindness

SECTION–II (40 MARKS)


(Attempt any four questions from this section)
Question 2.
(a) Given below is an experimental set up to demonstrate a particular process. Study the same and
answer the questions that follow.

Cobalt
Chloride Paper

DORSAL SURFACE VENTRAL SURFACE

(i) Name the physiological process being studied.


(ii) Explain the process mentioned above.
(iii) What is the aim of the above experiment ?
(iv) What would you observe in the experimental set-up after an hour ? Give a reason to
support your answer.
(v) Mention any three adaptations found in plants to overcome the physiological process
mentioned in (i) above. [5]
(b) Give biological/technical terms for the following :
(i) A membrane which allows the passage of molecules selectively.
(ii) The suppressed allele of a gene.
(iii) Structure that carries visual stimuli from retina to the brain.
(iv) WBCs squeeze through the walls of the capillaries into the tissue.
Bio 216 2012
(v) Protective coverings located round the human brain and spinal cord.
(vi) Eye lens losing flexibility resulting in a kind of long sightedness in elderly people.
(vii) Hormones which stimulate other endocrine glands to produce their specific hormones.
(viii) The phase in the menstrual cycle in which the remnant of follicle in the ovary turns to
Corpus luteum.
(ix) Statistical study of human population.
(x) Artificially introducing weekend germs or germ substance into the body for developing
resistance to a particular disease. [5]
Answer.
(a) (i) Transpiration in plants.
(ii) Transpiration is the loss of water vapour from the surface of the plant, mainly from the
leaves.
(iii) To compare the transpiration on the two sides or surfaces of a dorsiventral leaf.
(iv) After an hour, the paper on the lower surface of the leaf turns pink whereas the paper on
the upper surface takes more time to do so. This proves that more transpiration takes
place on the lower surface due to the presence of numerous stomata on it.
(v) • Leaves with thick cuticle
• Sunken stomata
• Leaves modified to spines
(b) (i) Semi permeable membrane. (ii) Recessive
(iii) Optic nerve (iv) Diapedesis
(v) Meninges (vi) Astigmatism
(vii) Hormones of pituitary gland (viii) Secretory phase
(ix) Demography (x) Vaccination
Question 3.
(a) Given below in the diagram of a cell as seen under the microscope after having been placed in
a solution :

1
2
3
4
Bio 217 2012
(i) What is the technical term used for the state/condition of the cell given above ?
(ii) Give the technical term for the solution in which the cell was placed.
(iii) Name the parts numbered 1 to 4.
(iv) Is the cell given above a plant cell or an animal cell ? Give two reasons in support of your
answer as evident from the diagram.
(v) What would you do to bring this cell back to its original condition ? [5]
(b) Differentiate between the following pairs on the basis of what is mentioned in brackets :
(i) Natality and mortality (definition)
(ii) Stoma and Storma (describe its structure)
(iii) Acromegaly and Cretinism (symptoms)
(iv) Transpiration and Guttation (structure involved)
(v) Diabetes mellitus and Diabetes insipidus (reason/cause) [5]
Answer.
(a) (i) Plasmolysed cell
(ii) Hypertonic solution
(iii) 1. Nucleus
2. Chloroplast
3. Vacuole
4. Cell sap
(iv) It is a plant cell, because
• There are chloroplasts in the cell.
• The shape of a cell is hexagonal i.e., a plant cell.
(v) We can place it in a hypotonic solution.
(b) (i) Natality is the number of births per 1000 individuals in the population per year.
Mortality is the number of deaths per 1000 individuals in the population per year.
(ii) Stoma is a pore present in the epidermis of leaves.
Stroma is a fluid in chloroplast containing enzymes that use ATP.
(iii) Acromegaly is a chronic condition developing in adulthood due to over production of
growth hormone.
Cretinism is a condition that results from inadequate secretion of thyroid hormones
during fetal life.
(iv) Transpiration takes place through stomata while guttation takes place though hydathodes.
(v) Diabetes mellitus is caused due to insufficient secretion of insulin. Diabetes insipidus is
caused due to the excess secretion of insulin.
Question 4.
(a) The diagram below shows the Exretory System of a Human being. Study the same and then
answer the questions that follow :
Bio 218 2012

1 2

4
5

7
8

(i) Name the parts labelled 1, 2, 3 and 4.


(ii) Give the main function of the parts labelled 5, 6, 7 and 8.
(iii) Name the endocrine gland which could be added in the diagram and state its location/
position. [5]
(b) Briefly explain the following :
(i) Osmosis (ii) Allele
(iii) Pulse (iv) Reflex action
(v) Synapse [5]
Answer.
(a) (i) 1. Posterior Vena cava 2. Aorta
3. Left renal artery 4. Left renal vein
(ii) 5. Ureters convey the urine from the kidneys to the bladder.
6. Urinary bladder is large sac in which urine is stored or collected.
7. Sphincter controls the release of urine from the bladder.
8. Urethra let the urine drain from the bladder.
(iii) Adrenal gland, it is located at the top of each kidney.
(b) (i) Osmosis is the net movement of water molecules from a region of their higher
concentration to a region of their lower concentration through a semipermeable membrane.
(ii) A pair of genes that controls the two alternative expressions of the same character and
has the same loci in the homologous chromosomes is called the allele.
Bio 219 2012
(iii) The rhythmic contraction of the heart is felt in certain areas like the wrist in the form of
pulsations, and this is called pulse.
(iv) Reflex actions are involuntary actions controlled by the spinal cord. It is a rapid automatic
response to a stimulus.
(v) Synapse is a junctional region between two neurons where information from one neuron
is transmitted or relayed (handed over) to another neuron, but there is no protoplasmic
connection between the two neurons.
Question 5.
(a) Study the diagram given below and then answer the question that follow :

(i) Name the part labelled A. Name any two hormones produced by the part labelled A.
(ii) What happens to the part labelled B–
(1) If fertilisation takes place ? (2) If fertilisation does not take place ?
(iii) Where does fertilisation occur ?
(iv) Draw a neat diagram of the human sperm as seen under high magnification and label the
following parts.
(1) Acrosome (2) Mitochondria
(b) A homozygous plant having round (R) and yellow (Y) seed is crossed with homozygous plant
having wrinkled (r) and green (y) seeds :
(i) Give the scientific name of the plant on which Mendel conducted his hybridization
experiments.
(ii) Give the genotype of the F1 generation.
(iii) Give the dihybrid phenotypic ratio and the phenotype of the offspring of F2 generation
when two plants of the F1 generation are crossed.
(iv) Name and state the law which explains the dihybrid ratio.
(v) Give the possible combinations of gametes that can be obtained from F1 hybrid. [5]
Answer.
(a) (i) Ovary
Ovary produces the hormones Estrogen and Progestrone.
Bio 220 2012
(ii) (1) Endomentrium secretes nutrients in preparation for implantation.
(2) If fertilisation does not take place, it degenerates or shed from the body.
(iii) Fertilisation takes place in the ampulla of the fallopian tube.
(iv)

(b) (i) Pisum sativum


(ii) Tt (Heterozygous)
(iii) Dihybrid phenotypic ratio = 9 : 3 : 3 : 1
Phenotype of F2 generation =
9 round and yellow
3 round and green
3 wrinkled and yellow
1 wrinkled and green
(iv) Law of independent assortment : When more than one pair of contrasting characters
are brought together in an individual, the entry of one pair of allele (controlling one
character) separates independently and assorts independently of any other pair of alleles
(controlling another character).
(v) F1 Hybrid = Tt
Possible combinations of gametes = T, t
Question 6.
(a) The diagram given below is an experiment conducted to study a factor necessary for
Photosynthesis. Observe the diagrams and then answer the following questions :
BEFORE THE EXPERIMENT AFTER THE EXPERIMENT

Green Portion
White Portion
Bio 221 2012
(i) What is the aim of the experiment ?
(ii) Name the test performed on the leaf and the solution used for test.
(iii) What type of leaf was used for the experiment ? Give an example.
(iv) What is the expected result of the above test on the parts labelled A and B ?
(v) Give a balanced chemical to represent the process of Photosynthesis. [5]
(b) The diagrams given below show the cross section of two kinds of blood vessels :

(i) Identify the blood vessels A and B. In each case give a reason to support your answer.
(ii) Name the parts numbered 1 and 2.
(iii) When are the sounds "LUBB" and "DUP" produced during a heartbeat ?
(iv) Name the blood vessel that
(1) begins and ends in capillaries. (2) supplies blood to the walls of the heart.
Answer.
(a) (i) To demonstrate that chlorophyll is essential for photosynthesis.
(ii) Starch test, iodine is used for this test.
(iii) Variegated leaf, for example coleus.
(iv) Part A will remain unchanged.
Part B will turn blue-black in colour.
Light energy
(v) 6CO2 + 6H2O Chlorophyll
C6H12O6 + 6O2  + 6H2O

(b) (i) 'A' is a artery while 'B' is a vein.


The size of lumen in A is small, so it is an artery.
The size of lumen in B is larger, so it is a vein.
(ii) 1–Endothelium (Tunica Interna). 2–Lumen of the vessel
(iii) The sound 'LUBB' occurs due to the closure of the atrioventricular valves. While 'DUP'
sound occurs as the aortic and pulmonary valves close to prevent the back flow of the
blood.
(iv) (1) Hepatic portal vein (2) Coronary arteries
Question 7.
(a) Answer the following :
(i) Draw a well labelled diagram of a 'Neuron' and name the following parts :
Bio 222 2012
(1) Node of Ranvier (2) Nissil granules
(3) Cyton
(ii) Name the part of the human brain which is concerned with the following :
(1) Seat of memory (2) Coordinates muscular activity
(iii) Mention any three major activities of the WHO.
(b) (i) Draw a well labelled diagram to show the metaphase state of Mitosis in an animal cell
having four chromosomes.
(ii) Mention any two reasons for the population explosion in INDIA.
(iii) Give biological reasons for the following :
(1) Pituitary gland is also known as the master gland.
(2) Gametes have a haploid number of chromosomes.
Answer.
(a) (i)

(ii) (1) Cerebrum (2) Cerebellum


(iii) WHO • Promotes medical education and training.
• Promotes research and development of international standard for pharmaceutical
products.
• To lay pharmaceutical standards for important drugs, ensure purity and size of
the dose.
(b) (i) Metaphase plate
Centrosome

Chromosome

Cell membrane
Cytoplasm

(ii) • Desire to have a male child. • Lack of education


(iii) (1) Pituitary gland influences almost all the endocrine glands of the body, so it is also
called the master gland.
(2) Gametes contain half the normal number of chromosomes.
ICSE QUESTION PAPER-2013 (SOLVED)
BIOLOGY
SCIENCE Paper – 3
(One hour and a half)
Answers to this Paper must be written on the paper provided separately.
You will not be allowed to write during the first 15 minutes.
This time is to be spent in reading the Question Paper,
The time given at the head of this Paper is the time allowed for writing the answers.
Attempt all questions from Section I and any four questions from Section II.
The intended marks for questions or parts of questions are given in brackets [ ].
SECTION–I (40 MARKS)
(Attempt all questions from this Section)
Question 1.
(a) Name the following :
(i) The cell body of a nerve cell.
(ii) The waxy layer on the epidermis of the leaf meant to reduce transpiration.
(iii) A non-biodegradable pesticide.
(iv) The physical expression of genes in an individual.
(v) Knot-like mass of blood capillaries inside the bowman's capsule. [5]
(b) State the exact location of the following :
(i) Chloroplast.
(ii) Incus.
(iii) Corpus callosum.
(iv) Guard cells.
(v) Pulmonary semilunar valve. [5]
(c) Give below are six sets with four terms each. In each set a term is an oldd one and cannot be
grouped in the same category to which the other three belong. Identify the odd one in each set
and name the category to which the remaining three belong. The first one has been done as an
example :
Example : Fructose, Sucrose, Glucose, Calcium.
Odd term : Calcium
Category : Carbohydrates.
(i) Cabonic acid, Acetic acid, benzoic acid, boric acid.
(ii) Saliva, bile, sweat, tears.
(iii) Cretinism, Myxedema, Simple goitre, Acromegaly.
(iv) Sneezing, coughing, blinking, typing.
(v) Semicircular canals, Cochlea, tympanum, utriculus. [5]
Bio 223 2013
Bio 224 2013
(d) Match the items in Column A with that which is most approriate in Column B. Rewrite the
matching pair.

Column A Column B

(1) Testis (a) Kidney


(2) Poliomyelitis (b) Water vapour
(3) Transpiration (c) Prostate gland
(4) Clotting of blood (d) Iron
(5) Uriniferous tubule (e) Uterus
(f) Gonad
(g) Salk's vaccine
(h) Water droplet
(i) Calcium
(j) TAB vaccine [5]

(e) Choose the correct answer from the four options given below :
(i) The cell component visible only during cell division.
A. Mitochondria
B. Chloroplast
C. Chromosome
D. Chromatin.
(ii) Pulse wave is mainly caused by the :
A. Systole of atria
B. Diastole of atria.
C. Systole of the left ventricle.
D. Systole of the right ventricle.
(iii) The recessive gene is one that expresses itself in :
A. Heterozygous condition
B. Homozygous condition
C. F2 generation
D. Y-linked inheritance.
(iv) A gland which secretes both hormone and enzyme is the :
A. Pituitary
B. Pancreas
C. Thyroid
D. Adrenal
Bio 225 2013
(v) The ventral root ganglion of the spinal cord contains cell bodies of the :
A. Motor neuron
B. Sensory neuron
C. Intermediate neuron
D. Association neuron. [5]
(f) Given below is an example of certain structures and their special functional activities.
For example : Eye and vision. On a similar pattern complete the following :
(i) Neutrophils : .......................................
(ii) Ureter : ..............................................
(iii) Neurotransmitters : .............................
(iv) Iris of the eye : ...................................
(v) Placenta : ........................................... [5]
(g) The figure given below represents an experiment to demonstrate a particular aspect of
photosynthesis. The alphabet 'A' represents a certain condition inside the flask.

(i) What is the aim of the experiment ?


(ii) Identify the special condition inside the flask.
(iii) Name an alternative chemical that can be used instead of KOH.
(iv) In what manner do the leaves 1 and 2 differ at the end of the starch test ? [5]
(h) Given below are five groups of terms. In each group arrange and rewrite the terms in the
correct order so as to be in a logical sequence.
For example :
Question : Implantation, Parturition, Ovulation, Gestation, Gertilization.
Answer : Ovulation, Fertilization, Implantation, Gestation, Parturition.
(i) Spongy cells, Upper epidermis, Stoma, Palisade tissue, Substomatal space.
(ii) Spinal cord, Motor neuron, Receptor, Effector, Sensory neuron.
Bio 226 2013
(iii) Endodermis, Cortex, Soil water, Xylem, Root hair.
(iv) Metaphase, Telophase, Prophase, Anaphase, Cytokinesis.
(v) Intestine, Liver, Intestinal artery, Hepatic Vein, Hepatic Portal Vein. [5]
Answer.
(a) (i) Perikaryon or cyton
(ii) Cuticle
(iii) D.D.T., B.H.C.
(iv) Phenotype
(v) Glomerulus/Glomerular capillaries
(b) Exact location :
(i) Mesophyll cells between upper and lower epidermis and palisade and spongy cells of
leaves.
(ii) Bone in middle ear.
(iii) In cerebrum part of brain.
(iv) In stoma of leaves.
(v) This valve is located at the opening of right ventricle into the pulmonary artery.
(c) Odd one out :
(i) Odd term : Boric acid
Category : Inorganic acid, organic acids (all others)
(ii) Odd term : Bile
Category : Germ killing secretions of our body (all others)
(iii) Odd term : Acromegaly
Category : Due to malfunctioning of thyroid gland (all others)
(iv) Odd term : Typing
Category : Natural (inborn) Reflex
(v) Odd term : Tympanum
Category : Part of inner ear (parts of inner ear all others)
(d) Column A Column B
(1) Testis (f) Gonad
(2) Poliomyelitis (g) Salk's vaccine
(3) Transpiration (b) Water vapour
(4) Clotting of blood (i) Calcium
(5) Uriniferous tubule (a) Kidney
(e) (i) Chromosome
(ii) Systole of the left ventricle
(iii) Homozygous condition
(iv) Pancreas
Bio 227 2013

(v) Sensory neuron


(f) Functional activities :
(i) Phagocytosis
(ii) Ureter : Passes the urine from kidney to urinary bladder.
(iii) Neurotransmitters : The end portion of axon have swollen ends like bulbs which store
chemicals called neurotransmitters and help in transmission of nerve impulse.
(iv) Iris of the eye : Regulates the amount of light entering the eye.
(v) Placenta : It is the link between the mother and the foetus (embryo).
(g) (i) Aim : Carbon dioxide is necessary for photosynthesis.
(ii) Flask must have KOH solution to absorb CO2.
(iii) Lime water.
(iv) Leaf 2 which has exposed to atomspheric air becomes blue black and leaf 2 will not
become blue black.
(h) Arrange the terms :
(i) Upper epidermis, Palisade tissue, Spongy cells, Substomatal space, Stoma.
(ii) Receptor, Sensory neuron, Spinal cord, Motor neuron, Effector.
(iii) Soil water, Root hair, Cortex, Endodermis, Xylem.
(iv) Prophase, Metaphase, Anaphase, Telophase, Cytokinasis.
(v) Intestinal artery, Intestine, Hepatic Portal Vein, Liver, Hepatic Vein.

SECTION–II (40 MARKS)


(Attempt any four questions from this section)
Question 2.
(a) Given below is a diagram of the lateral section of a testis of a man. Study the same and answer
the questions that follow :

(i) Label the parts numbered 1 to 4 of the diagram.


(ii) State the functions of the parts labelled 1 and 3.
Bio 228 2013
(iii) What is the significance of the testis being located in the scrotal sac outside the abdomen?
(iv) What is the role played by the inguinal canal ?
(v) What is semen ? [5]
(b) Give the biological/technical terms for the following :
(i) Chemicals found in the blood which act against antigens.
(ii) A constituent that causes pollution.
(iii) The onset of menstruation in a young girl.
(iv) Structure which connects the placenta with the foetus.
(v) The fluid present between the layers of meninges.
(vi) Permanently open structures seen on the bark of an old woody stem.
(vii) The biological process which is the starting point of the food chain.
(viii) The change in an organism resulting due to stimulus.
(ix) An Antiseptic substance present in tears.
(x) A solution in which the relative concentration of water molecules and the solute on
either side of the cell membrane is the same. [5]
Answer.
(a) (i) (1) Seminiferous tubule
(2) Lobule
(3) Epididymis
(4) Sperm duct
(ii) Function of Part 1 :
In seminiferous tubule the sperms are produced.
Function of Part 3 :
Epididymis stores the sperms for some days during which they mature and become
motile.
(iii) Since maturation of sperm takes place 2° to 3° lower than the body temperature hence
located in the scrotal sac outside the abdomen.
(iv) Inguinal canal allows the descent of testis along with their ducts blood vessels nerves
etc.
(v) Semen : Seminal vesicles, prostate gland, cowper's glands add their secretions to the
sperms and collectively form milky fluid the semen.
(b) (i) Antibodies produced by lymphocytes
(ii) CO2, SO2, CO
(iii) Menarche
(iv) Umbilical Cord
(v) Cerebrospinal fluid
(vi) Lenticels
(vii) Photosynthesis
Bio 229 2013
(viii) Response
(ix) Lysozyme
(x) Isotonic
Question 3.
(a) Draw a diagram of the human eye as seen in a vertical section and label the parts which suits
the following descriptions relating to the :
(i) photosensitive layer of the eye.
(ii) structure which is responsible for holding the eye lens in its position.
(iii) structure which maintains the shape of the eye ball and the area of no vision.
(iv) anterior chamber seen in front of the eye lens.
(v) Outer most transparent layer seen in front of the eye ball. [5]
(b) Differentiate between the following pairs on the basis of what is mentioned within brackets :
(i) Photolysis and Photophosphorylation. (Definition)
(ii) Bicuspid valve and Tricuspid valve. (Function)
(iii) Vasectomy and Tubectomy. (Explain)
(iv) Cerebrum and Spinal cord. (Arrangement of nerve cells)
(v) Bowman's capsule and Malpighian capsule. (parts included) [5]
Answer.
(a)

(i) Retina
(ii) Suspensory ligament
(iv) Aqueous chamber
(iii) Vitreous chamber
(v) Conjunctiva

(i) Retina.
(ii) Suspensary ligament.
(iii) Vitreous chamber
(iv) Aqueous chamber.
(v) Conjunctiva.
Bio 230 2013
(b) (i) Photolysis : Splitting of water by the action of light during photosynthesis.
Photophosphorylation : Addtion of phosphate (phosphoryl) group to a molecule such
as ADP to ATP.
(ii) Bicuspid and tricuspid valves regulate the flow of blood in a single direction.
(iii) Vasectomy : It is a surgical method for female for population control. In this method
abdomen is opened and the fallopian tubes (oviducts) are cut or tied with nylon thread to
close the passage of the egg.
Tubectomy : This is the surgical method in male for population control. In this surgery
a small cut is made in scrotum, vas deferens (sperm duct) is ligated and small piece
between the two ligatures is removed. This surgery is easier, quicker and safer.
(iv) Cerebrum and spinal cord (arrangement of nerves).
Cerebrum : Inner portion of cerebrum consists of 'white matter' which mainly contains
the axons (nerve fibres) of the neuron.
The outer portion of cerebrum contains cell bodies of neurons is called 'gray matter'.
Spinal cord : In spinal cord arrangement of white and gray matter is reversed from that
in brain.
(v) Bowman's Capsule : It is a rounded, cup-shaped body.
Malpighian Capsule : The Bowman's capsule and the glomerulus together are called
Malpighian capsule.
Question 4.
(a) Given below is a schematic diagram showing Mendel's Experiment on sweet pea plants having
axial flowers with round seeds (AARR) and Terminal flowers with wrinkled seeds (aarr).
Study the same and answer the questions that follow :

(i) Give the phenotype of F1 progeny.


(ii) Give the phenotypes of F2 progeny produced upon by the self-pollination of F1 progeny.
(iii) Give the phenotypic ratio of F2 progeny.
(iv) Name and explain the law induced by Mendel on the basis of the above observation.
[5]
Bio 231 2013

(b) Complete the following table by filling in the blanks from 1 to 10 with appropriate terms :
S.No. Gland Secretion Function / Effect on body
1. Thyroid 1 2
2. 3 Vasopressin 4
3. 5 6 Promotes glucose utilization
by the body cells.
4. Lacrimal gland 7 8
5. Adrenal medulla 9 10 [5]

Answer.
(a) (i) F1 Progeny — Axial Round
(ii) AR aR
Ar ar
AR Ar aR ar
AR AARR AARr AaRR AaRr
Ar AARr AArr AaRr Aarr
aR AaRR AaRr aaRR aaRr
ar AaRr Aarr aaRr aarr
(iii) Axial round Axial wrinkled Terminal Round T.W.
9 : 3 : 3 : 1
(iv) Law of independent assortment. When there are two pairs of contrasting characters the
distribution of the members of one pair into the gamete is independent of the distribution of the
other pair.
(b) S.No. Gland Secretion Function / Effect on body

1. Thyroid Thyroxine Regulates basal metabolism.

2. Posterior pituitary Vasopressin Increase reabsorption of H2O


from kidney tubule.

3. Adrenal cortex Glucocorticoids Promotes glucose utilization


Langerhans by the body cells.

4. Lacrimal gland Lysozyme Kills the germs

5. Adrenal medulla Adrenaline increases heart beat, increases

blood supply to the muscle


Bio 232 2013
Question 5.
(a) The diagram given below represents the human heart in one phase of its functional activities.
Study the same and answer the questions that follow :

2 3
1

(i) Name the phase.


(ii) Label the parts 1, 2 and 3.
(iii) Which part of the heart is contracting in this phase ? Give a reason to support your
answer.
(iv) Draw well labelled diagrams of part 1 and 2 to show the structural differences between
them. [5]
(b) Give biological reasons for the following :
(i) The wall of the ventricle is thicker than the auricles.
(ii) The renal cortex has a dotted appearance.
(iii) Wooden frames of doors get jammed during the monsoon season.
(iv) Throat infections can lead to ear infections.
(v) The hand automatically shows the direction to turn a cycle without thinking. [5]
Answer.
(a) (i) Phase : Auricular systole.
(ii) 1. Pulmonary artery (to the left lung)
2. Superior vena cava
3. Aorta
(iii) Auricles are contracting in this phase because bicuspid (operative) and tricuspid values
(operative) are open to allow the blood to flow from auricles to ventricles.
Bio 233 2013
(iv)

ARTE Thin muscular wall VEIN

Thick muscular wall Wide lumen


Narrow lumen

(b) (i) Walls of ventricle is thicker because they have to pump blood to long distance.
(ii) Each human kidney possesses about one million of functional and structural units known
as nephrons or kidney tubules.
(iii) Wooden doors get jammed during monsson season because of imbibition substance
which are made up of cellulose are hydrophilic.
(iv) Throat infection can lead to ear infection due to the link of nasolacrimal duct between
both.
(v) Hand automatically show the direction due to conditioned (Acquired) reflexes.
Question 6.
(a) The figure given below shows the epidermal cells of an onion bulb. This cell was then transferred
to a drop of sugar solution.

(i) Draw a well labelled diagram of the epidermal cell as it would appear after immersion in
a strong sugar solution.
(ii) What scientific term is used for the changes as shown in (i) above ?
(iii) What should be done to resotre the cell back to its original condition ?
(iv) Give the scientific term for the recovery of the cell as a result of the step taken in (iii)
above.
(v) Define the term osmosis. [5]
(b) Briefly explain the following terms :
(i) Genes. (ii) Cytokinesis in plant cells.
(iii) Guttation. (iv) Diabetes insipidus.
(v) disinfectants. [5]
Bio 234 2013
Answer. Cytoplasm
(a)
Cell wall

Plasma /Cell Nucleus


membrane
Shrunken
protoplasm
Hypertonic solution

(i) Cytoplasm will shrink and plasma membrane will withdraw from cell wall.
(ii) Plasmolysis. (iii) Cell is to be placed in distilled water. (iv) Deplasmolysis.
(v) Osmosis is the diffusion of water molecule across semipermeable membrane from a
more dilute solution to less dilute solution.
(b) (i) Genes : The specific part of chromosomes which are composed of D.N.A. Genes
determine hereditary characters that transfer from parents to offsprings.
(ii) Cytokinesis in plant cells : A cell plate is laid down in the cytoplasm at equatorial plane.
(iii) Guttation : Guttation is the loss of water as droplet along the margins of leaves through
hydathodes. e.g. Banana, strawberry.
(iv) Diabetes insipidus : When concentration of glucose decrease in the blood as well as in
urine due to hypersecretion of insulin.
(v) Disinfectants : Strong chemical sulestances that are applied on spots and places where
germs thrive and multiply. e.g. Phenol, lysol, lime, 40% formalin.
Question 7.
(a) (i) Draw a well labelled diagram to show the anaphase stage of mitosis in a plant cell having
four chromosomes.
(ii) State any two harmful effects of acid rain.
(iii) Expand the following biological abbreviations : (1) NADP, (2) ACTH [5]
(b) (i) List any two major activities of the Red Cross.
(ii) Write any two major reasons for the population explosion in the world.
(iii) Write the names of four nitrogenous bases in a DNA molecule. [5]
Answer.
Cell
(a) (i) wall

Chromatid

Centriole

(ii) Acid rain is caused due to pollutant gases CO2, SO2, CO causing much danger to
agricultural crops and stone monuments.
(iii) (1) NADP : Nicotinamide Adenine Dinucleotide Phosphate.
(2) ACTH : Adrenocorticotropic.
(b) (i) (1) Red cross extend relief and help to the victims of any calamity – flood, fire,
famine, earthquakes etc.
(2) To extend all possible first aid in any accident.
(ii) Two major reasons for population growth in world are illiteracy, traditional beliefs,
social customs and low marriage age.
(iii) These bases are as follow :
Adenine (A), Guanine (G), Thymine (T) and Cytosine (C)
ICSE QUESTION PAPER-2014 (SOLVED)
BIOLOGY
SCIENCE Paper – 3
(One hour and a half)
Answers to this Paper must be written on the paper provided separately.
You will not be allowed to write during the first 15 minutes.
This time is to be spent in reading the Question Paper,
The time given at the head of this Paper is the time allowed for writing the answers.
Attempt all questions from Section I and any four questions from Section II.
The intended marks for questions or parts of questions are given in brackets [ ].
SECTION–I (40 MARKS)
(Attempt all questions from this Section)
Question 1.
(a) Name the following :
(i) The part of the brain associated with memory.
(ii) The ear ossicle which is attached to the tympanum.
(iii) The type of gene, which in the presence of a contrasting allele is not expressed.
(iv) The hormone secreted by islets of langerhans.
(v) The process of conversion of ADP into ATP during photosynthesis. [5]
(b) State the main function of the following :
(i) Cerebrospinal fluid.
(ii) Eustachian tube.
(iii) Suspensory ligament of the eye.
(iv) Sperm duct.
(v) Linticels. [5]
(c) Copy and complete the following by filling the blanks 1 to 5 with appropriate words :
The human female gonads are ovaries. A maturing egg in the ovary is present in a sac of cells
called .......... (1). As the egg grows larger, the follicle enlarges and gets filled with a fluid and
is now called the .......... (2) follice. The process of releasing the egg from the ovary is called
.......... (3). The ovum is picked up by the oviducal funnel and fertilization takes place in the
.......... (4). In about a week the blastocyst gets fixed in the endometrium of the uterus and this
process is called .......... (5).
(d) Given below are six sets with four terms each. In each set one term is odd and cannot be
grouped in the same category to which the other three belong. Identify the odd one in each set
and name the category to which the remaining three belong. The first one has been done as an
example.
Example : Calyx, Corolla, Stamens, Midrib
Odd term : midrib

Bio 235 2014


Bio 236 2014
Category : parts of a flower.
(i) Haemoglobin, Glucagon, Iodopsin, Rhodopsin.
(ii) Urethra, Uterus, Urinary bladder, Ureter.
(iii) Transpiration, Photosynthesis, Phagocytosis, Guttation.
(iv) Cyton, Photon, Axon, Dendron.
(v) Oxytocin, Insulin, Prolactin, Progesterone. [5]
(e) The figure given below represents an experimental set up with a weighing machine to demonstrate
a particular process in plants. The experimental set up was placed in bright sunlight. Study the
diagram and answer the following questions :

Test
Tube
A Gummed Oil
Paper B
Water
A B
B

(i) Name the process intended for study.


(ii) Define the above mentioned process.
(iii) When the weight of the test tube (A and B) is taken beofre and after the experiment,
what is observed? Give reasons to justify your observation in A and B.
(iv) What is the purpose of keeping the test tube B in the experimental set up?
(f) Match the items given in Column A with the most appropriate ones in Column B and rewrite the
correct matching pairs from Column A and Column B :

Sl. No. Column A Column B

1. Pituitary gland a. Testosterone


2. Sulphur dioxide b. Calcium
3. Seminiferous tubules c. Growth hormone
4. Clotting of blood d. Acid rain
5. Guttation e. Sperms
f. Global warming
g. Magnesium
h. Hydathodes [5]
Bio 237 2014
(g) Choose the correct answer from the four options given below :
(i) Cretinism and Myxoedema are due to :
A. Hyper secretion of thyroxin
B. Hyper secretion of growth hormone
C. Hyposecretion of thyroxin
D. Hyposecretion of growth hormone.
(ii) Which of the following is not a natural reflex action?
A. Knee-jerk.
B. Blinking of eyes due to strong light
C. Salivation at the sight of food
D. Sneezing when any irritant enters the nose.
(iii) After mitotic cell division, a female human cell will have :
A. 44 + xx chromosome.
B. 44 + xy chromosome
C. 22 + x chromosome
D. 22 + y chromosome.
(iv) The antibiotic penicillin is obtained from :
A. Protozoan
B. Bacteria
C. Virus
D. Fungus
(v) The site of maturation of human sperms is the :
A. Seminiferous tubule
B. Interstitial cells
C. Epididymis
D. Prostate gland. [5]
(h) State the exact location of the following :
(i) Tricuspid value.
(ii) Amnion.
(iii) Yellow spot.
(iv) Seminal vesicle
(v) Adrenal gland. [5]
Answer.
(a) (i) Cerebrum
(ii) Malleus/Hammer
(iii) Recessive gene
(iv) Insulin, Glucagon.
Bio 238 2014
(v) Photophosphorylation.
(b) (i) Provide mechanical protection to brain/provide nourishment to brain.
(ii) Maintain/Equalize air pressure on either sides of the ear drum.
(iii) Hold the lens in position.
(iv) Transport sperm from the epididymis to penis for discharge.
(v) To facilitate transpiration in older stem.
(c) The human female gonads are ovaries. A maturing egg in the ovary is present in a sac of cells
called follicle (1). As the egg grows larger, the follicle enlarges and gets filled with a fluid and
is now called the Graafian (2) follice. The process of releasing the egg from the ovary is
called ovulation (3). The ovum is picked up by the oviducal funnel and fertilization takes place
in the oviduct (4). In about a week the blastocyst gets fixed in the endometrium of the uterus
and this process is called implantation (5).
(d) (i) Odd term : Glucagon
Category : Pigments
(ii) Odd term : Uterus
Category : Parts of excretory system
(iii) Odd term : Phagocytosis
Category : Processes taking place in plants
(iv) Odd term : Photon
Category : Parts of a neuron.
(v) Odd term : Insulin
Category : Female hormones.
(e) (i) Transpiration.
(ii) It is the process in which water is lost in the form of vapour from the aerial parts of the
plant.
(iii) After the experiment, we observe test tube A is raised up i.e., a loss of weight. In test
tube B, the weight remains unchanged.
Reason : In test tube A, loses weight due to the plant transpires and loses water. In B
there is no plant so there is no transpiration and the oil on the surface prevents direct
evaporation and so weight remains unchanged.
(iv) It acts as a Control Set Up.
(f) Column A Column B
1. Pituitary gland c. Growth hormone
2. Sulphur dioxide d. Acid rain
3. Seminiferous tubules e. Sperms
4. Clotting of blood b. Calcium
5. Guttation h. Hydathodes
(g) (i) (C) Hyposecretion of thyroxine.
(ii) (C) Salivation at the sight of food.
Bio 239 2014

(iii) (A) 44 + xx chromosome.


(iv) (D) Fungus.
(v) (C) Epididymis.
(h) Exact location :
(i) Tricuspid value : Between right auricle and right ventricle.
(ii) Amnion : Around the foetus in the uterus.
(iii) Yellow spot : Centre of horizontal axis of eye ball on retina.
(iv) Seminal vesicle : Between posterior surface of the urinary bladder and rectum in male.
(v) Adrenal gland : Above each kidney fitted like a cap.

SECTION–II (40 MARKS)


(Attempt any four questions from this section)
Question 2.
(a) Differentiate between the following pairs on the basis of what is mentioned within brackets :
(i) Spinal nerves and Cranial nerves (Number of nerves)
(ii) Near vision and Distant Vision (shape of the eye lens)
(iii) Corpus callosum and Corupus luteum. (function).
(iv) Turgor pressure and wall pressure. (Explain).
(v) Disinfectant and Antiseptic (Definition). [5]
(b) The diagram below represents the simplified pathway of the circulation of blood. Study the
same and answer the questions that follow :

(i) Name the blood vessels labelled 1 and 2.


(ii) State the function of blood vessels labelled 5 and 8.
(iii) What is the importance of the blood vessel labelled 6?
Bio 240 2014
(iv) Which blood vessel will contain a high amount of glucose and amino acids after a meal?
(v) Draw a diagram of the different blood cells as seen in a smear of human blood. [5]
Answer.
(a) (i) Spinal nerves : 31 pairs.
Cranial nerves : 12 pairs.
(ii) Near vision : More convex/too curved.
Distant vision : Less convex/too flat.
(iii) Corpus callosum : Transmitting impulses from one cerebral hemisphere to other.
Corpus luteum : Secrete female sex hormones Oestrogen and progesterone.
(iv) Turgor pressure : The pressure exerted by cell contents of a turgid cell on the cell wall.
Wall pressure : The pressure exerted by the wall of a turgid cell on the cell contents.
(v) Disinfectant : These are chemicals of strong concentration applied on spots and areas
to kill germs.
Antiseptic : These are chemicals of mild concentration applied on the body to kill
germs or harmful microbes.
(b) (i) (1) Anterior/Superior venacava.
(2) Aorta.
(ii) Function of 5 : Supply oxygenated blood from Aorta to liver.
Function of 8 : Carry deoxygenated blood from posterior parts of the body to the right
auricle of heart.
(iii) Importance of the blood vessel labelled 6 : Blood vessel 6 is called Hepatic portal
vein. It carries deoxygenated blood from intestine to liver. This blood contains excess
glucose, some toxic substances etc. which are sent to liver where they are detoxified
and the excess glucose is converted to glycogen and stored. This prevents these
substances from directly entering the heart and damaging the heart.
(iv) Hepatic Portal Vein.
(v) RBC
WB

Plasma

Platelet
Question 3.
(a) A candidate in order to study the process of osmosis has taken 3 potato cubes and put them in
3 different beakers containing 3 different solutions. After 24 hours, in the first beaker the
Bio 241 2014
potato cube increased in size, in the second beaker the potato cube decreased in size and in the
third beaker there was no change in the size of the potato cube. The following diagram shows
the result of the same experiment :

(i) Give the technical terms of the solution used in beakers, 1, 2 and 3.
(ii) In beaker 3 the size of the potato cube remains the same. Explain the reason in brief.
(iii) Write the specific feature of the cell sap of root hairs which helps in absorption of water.
(iv) What is osmosis?
(v) How does a cell wall and a cell membrane differ in their permeability? [5]
(b) A potted plant was taken in order to prove a factor necessary for photosynthesis. The potted
plant was kept in the dark for 24 hours. One of the leaves was covered with black paper in the
centre. The potted plant was then placed in sunlight for a few hours.
(i) What aspect of photosynthesis was being tested?
(ii) Why was the plant placed in the dark before beginning the experiment?
(iii) During the starch test why was the leaf :
(1) boiled in water. (2) boiled in methylated spirit.
(iv) Write a balanced chemical equation to represent the process of photosynthesis.
(v) Draw a neat diagram of a chloroplast and label its parts. [5]
Answer.
(a) (i) (1) Hypotonic solution.
(2) Hypertonic solution.
(3) Isotonic solution.
(ii) In beaker 3, the concentration of potato cube and the medium is same. So there is no
osmosis taking place therefore the size of cube remains same.
(iii) The concentration of cell sap is higher in the root hair as compared to soil water due to
endosmoses is taken place facilitating absorption of water.
(iv) Osmosis is a process of flow of solvent molecules from lower concentration to higher
concentration through a semipermeable membrane.
(v) Cell wall is freely permeable and cell membrane is semi-permeable.
(b) (i) Sunlight is necessary for photosynthesis.
(ii) To make the leaves free from starch.
(iii) (1) To kill the cells. (2) To remove the chlorophyll.
Bio 242 2014
Sunlight
(iv) 6CO2 + 12H2O C6H12O6 + 6H2O + 6O2
Chlorophyll

(v)

Question 4.
(a) The diagram given below is a representation of a certain phenomenon pertaining to the nervous
system. Study the diagram and answer the following questions :

3
1
2

6
5

(i) Name the phenomenon that is being depicted.


(ii) Give the technical term for the point of contact between the two nerve cells.
(iii) Name the parts 1, 2, 3 and 4.
(iv) Write the functions of parts 5 and 6.
(v) How does the arrangement of neurons in the spinal cord differ from that of the brain?
[5]
(b) Give scientific reasons for the following statements :
(i) Use of C.F.C. is banned in many countries.
(ii) We cannot distinguish colours in moonlight.
(iii) Balsam plants wilt during midday even if the soil is well watered.
(iv) Carbon monoxide is highly dangerous when inhaled.
(v) A person after consuming alcohol walks clumsily. [5]
Answer.
(a) (i) Reflex action.
(ii) Synapse
Bio 243 2014

(iii) (1) Sensory neuron / Afferent fibre.


(2) Dorsal ganglion / Dorsal root.
(3) White matter
(4) Gray matter
(iv) Function of 5 (Synape) : Transmit the sensory impulse from sensory neuron to the
motor neuron.
Function of 6 (Motor neuron/Efferent neuron) : Transmit the command to the effectors
(muscle or glands).
(v) In Spinal cord : Cytons are in the inner grey matter and Axons in the outer white
matter.
In Brain : Cytons are in the outer grey matter and Axons in the inner white matter.
(b) (i) Chlorifluorocarbons (CFC) are one of the major cause for ozone depletion.
(ii) Moonlight is dimlight during which cone cells of our eye do not function well therefore
colour is not perceieved. It is only red cells that function in moonlight.
(iii) It is a herbaceous plant and loses too much of water by excessive transpiration. The rate
of transpiration is more than the rate of absorption. So they with weak stem droops and
wilts.
(iv) Carbon monoxide has great affinity with haemoglobin of our blood. It mixes with
haemoglobin almost 300 times more than that with oxygen. It cuts off the supply of
oxygen due to which it is fatal.
(v) Alcohol affects the cerebellum part of the brain which is responsible for muscular co-
ordination. So after consuming alcohol due to lack of muscular balance and co-ordination,
the person walks clumisly.
Question 5.
(a) Give below is a diagram representing a stage during mitotic cell division. Study it carefully and
answer the questions that follow :

(i) Is it a plant cell or an animal cell? Give a reason to support your answer.
(ii) Identify the stage shown.
(iii) Name the stage that follows the one shown here. How is that stage identified?
(iv) How will you differentiate between mitosis and meiosis on the basis of the chromosome
number in the daughter cells?
Bio 244 2014
(v) Draw a duplicated chromosome and label its parts. [5]
(b) (i) Name the disease for which the following of vaccines are given :
(1) Salk's Vaccine.
(2) B.C.G.
(ii) Give one example of each of the following :
(1) A water pollutant.
(2) An aquatic plant used in the lab to demonstate O2 liberation during photosynthesis.
(3) An antibiotic.
(4) A nitrogenous base in DNA.
(iii) Expand the following biological abbreviations :
(1) ATP (2) TSH
(3) DPT (4) DNA [5]
Answer.
(a) (i) Plant cell. Because Cell wall present and Centrioles absent.
(ii) Prophase.
(iii) Metaphase. Chromosomes are arranged in the equitorial plane of the cell.
(iv) In Mitosis : The chromosome number in daughter cell is same as that of the mother cell
i.e., diploid mother cell gives rise to 2 diploid daughter cells.
In Meiosis : The chromosome number is halved in the daughter cells i.e., the diploid
mother cell gives rise to four haploid daughter cells.
(v)
chromatid

centromere

(b) (i) (1) Poliomyelitis. (2) Tuberculosis.


(ii) (1) Chemicals like Mercury from Industries / Sewage / detergents
(2) Hydrilla. (3) Penicillin.
(4) Adenine.
(iii) (1) ATP : Adenosine triphosphate.
(2) TSH : Thyroid stimulating hormone.
(3) DPT : Diphtheria Pertussis Tetanus.
Bio 245 2014
(4) DNA : Deoxyribo Nucleic acid.
Question 6.
(a) The given diagram represents a nephron and its blood supply. Study the diagram and answer
the following questions :

(i) Label parts 1, 2, 3 and 4.


(ii) State the reason for the high hydrostatic pressure in the glomerulus.
(iii) Name the blood vessel which contains the least amount of urea in this diagram.
(iv) Name the two main stages of urine formation.
(v) Name the part of the nephron which lies in the renal medulla. [5]
(b) Briefly explain the following terms :
(i) monohybrid cross. (ii) Biomedical waste.
(iii) Innate immunity. (iv) Diapedesis.
(v) Hormones. [5]
Answer.
(a) (i) (1) Collecting duct. (2) Distal convoluted tubule.
(3) Loop of Henle. (4) Bowman's capsule.
(ii) Afferent arteriole is wider than efferent artiriole.
(iii) Blood vessel 6-Efferent arteriole that connects to renal vein.
(iv) (1) Ultrafiltration. (2) Selective Reabsorption.
(v) Loop of Henle.
(b) (i) Monohybrid Cross : A cross between two parents taking the alternative traits of one
single character. For example, a cross between tall and dwarf pea plants.
(ii) Wastes containing dressings, amputated body parts, used surgical instruments etc. from
hospitals, that spread diseases.
(iii) Innate immunity is the immunity that a person inherits from his parents i.e., the person
is born with it.
(iv) The process by which white blood cells squeeze out through the walls of capillaries to
reach the site of infection.
Bio 246 2014
(v) Hormones : Chemical substance produced by endorine gland and poured into blood to
act on target organs.
Question 7.
(a) (i) State any two harmful effects of noise pollution on human health.
(ii) Categorize the following activities as per the functions of the Red Cross Society and the
WHO :
(1) To suggest quarantine measures to prevent spread of disease.
(2) Humanitarian services to victim of war.
(3) To educate people in accident prevention.
(4) To promote projects for research on disease.
(iii) Write any two major reasons for the population explosion in India.
(iv) State Mendel's Law of segregation. [5]
(b) Give technical terms for the following :
(i) A method of contraception in which the sperm duct is cut and ligated.
(ii) Statistical study of human population.
(iii) The protective covering of the heart.
(iv) A sudden heritable change in the gene.
(v) Repeated units of DNA molecule.
(vi) The fluid portion of blood.
(vii) The nerve that transmits impulses from the ear to the brain.
(viii) Group of hormones which influence other endocrine glands to produce hormones.
(ix) Thin walled sac of skin that covers the testes.
(x) The permanent stoppage of the menstrual cycle in a woman aged 50 years. [5]
Answer.
(a) (i) Two harmful effects of noise pollution :
(1) Cause hearing disorder.
(2) Cause high blood pressure.
(ii) (1) WHO (2) Red Cross
(3) Red Cross (4) WHO
(iii) (1) Desire for a male child.
(2) Illiteracy and lack of awareness about birth control measures.
(iv) This law states that in a monohybrid cross, the contrasting characters or factors separate/
or segregate from each other at the time of gamete formation.
(b) (i) Vasectomy (ii) Demography
(iii) Pericardium (iv) Mutation
(v) Nucleotide (vi) Plasma
(vii) Auditory nerve (viii) Tropic hormones
(ix) Scrotum (x) Menopause.
ICSE QUESTION PAPER-2015 (SOLVED)
BIOLOGY
SCIENCE Paper – 3
(One hour and a half)
Answers to this Paper must be written on the paper provided separately.
You will not be allowed to write during the first 15 minutes.
This time is to be spent in reading the Question Paper,
The time given at the head of this Paper is the time allowed for writing the answers.
Attempt all questions from Section I and any four questions from Section II.
The intended marks for questions or parts of questions are given in brackets [ ].

SECTION I (40 Marks)


Attempt all questions from this Section
Question 1
(a) Name the following:
(i) The process of uptake of mineral ions against the concentration gradient using energy from cell.
(ii) The form in which glucose is stored in liver.
(iii) The vein that carries oxygenated blood.
(iv) The cross between two parents having one pair of contrasting characters.
(v) The structure formed by the villi of the embryo and the uterus of the mother. [5]
(b) The statements given below are False. Rewrite the correct form of the statement by changing the
word which is underlined:
(i) Alpha cells of pancreas secrete Insulin.
(ii) Formalin is an example of an Antiseptic.
(iii) CNG is mainly responsible for the formation of acid rain.
(iv) Sulphadiazine is an example of an Antiseptic.
(v) Cretinism is caused due to deficiency of Adrenaline. [5]
(c) Choose the correct answer from the four options given below:
(i) A single highly coiled tube where sperms are stored, gets concentrated and mature is known as:
A. Epididymis B. Vas efferentia
C. Vas deferens D. Seminiferous tubulewww.topperlearning.com 2
(ii) Chromosomes get aligned at the centre of the cell during:
A. Metaphase B. Anaphase
C. Prophase D. Telophase
(iii) BCG vaccine is effective against
A. Cholera B. Mumps
C. Tuberculosis D. Measles
(iv) Which one of the following is associated with the maintenance of the posture?
A. Cerebrum B. Cerebellum
C. Thalamus D. Pons
Bio 247 2015
Bio 248 2015
(v) An example of non-biodegradable waste is
A. Vegetable peels B. Sewage
C. Livestock waste D. DDT [5]
(d) Mention the exact location of the following structures:
(i) Thylakoids (ii) Organ of Corti
(iii) Lenticels (iv) Bicuspid valve
(v) Loop of Henle [5]
(e) The diagram given below represents a certain stage of mitosis:

Diagram

(i) Identify the stage of cell division.


(ii) Name the parts labelled A and B
(iii) What is the unique feature observed in this stage?
(iv) How many daughter cells are formed from this type of cell division? [5]
(f) Given below is an example of a certain structure and its special functional activity. On a similar
pattern fill in the blanks with suitable functions:
Example: Chloroplast and photosynthesis
(i) Xylem and _________________.
(ii) Ciliary body and _________________.
(iii) Seminiferous Tubule and _________________.
(iv) Thyroid gland and _________________.
(v) Eustachian Tube and _________________. [5]
(g) Rewrite and complete the following sentences by inserting the correct word in the space indicated:
(i) The phenomenon of loss of water through a cut stem or injured part of plant is called ___________.
(ii) ___________ is the scientific name of garden pea, which Mendel used for his experiments.
(iii) A fluid that occupies the larger cavity of the eye ball behind the lens is ___________.
(iv) Oxygen combines with haemoglobin present in RBC and forms ___________.
(v) ___________ causes corrosion of the marble or brick surface. [5]
Bio 249 2015
(h) Match the items in Column ‘A’ with those which are most appropriate in Column ‘B’.
Rewrite the matching pairs as shown in the example: [5]
Example: Fibrinogen – Clotting of blood.
Column A Column B
(1) Allele (a) Control of automobile exhaust
(2) Leydig cells (b) Tourniquet
(3) Utriculus (c) Alternate forms of genes
(4) Snake bite (d) Dynamic equilibrium
(5) Euro IV norms (e) Testosterone
(f) Sudden change in genes
(g) Static equilibrium
Answer :
(a) (i) Active transport (ii) Glycogen
(iii) Pulmonary vein (iv) Monohybrid cross
(v) Placenta
(b)
(i) Alpha cells of pancreas secrete glucagon.
(ii) Formalin is an example of a disinfectant.
(iii) Sulphur dioxide is mainly responsible for the formation of acid rain.
(iv) Sulphadiazine is an example of a sulpha drug.
(v) Cretinism is caused due to deficiency of thyroxine.
(c)
(i) A. Epididymis
(ii) A. Metaphase
(iii) C. Tuberculosis
(iv) B. Cerebellum
(v) D. DDT
(d)
(i) Thylakoids: Present in colourless ground substance/Stroma/Matrix of chloroplast
(ii) Organ of Corti: In the Cochlea / On the basilar membrane of median canal in Cochlea
(iii) Lenticels: Present on the older stems of plants in place of stomata.
(iv) Bicuspid valve: Located at the aperture between the left auricle and the left ventricle of the heart.
(v) Loop of Henle: Runs in the medulla of the kidneys and connects the proximal convoluted tubule
and the distal convoluted tubule.
(e)
(i) Anaphase
(ii) A - Spindle fibre
B - Centromere
(iii) Chromosomes are seen moving towards the poles.
(iv) This type of cell division (mitosis) results in the formation of two daughter cells.
(f)
Bio 250 2015
(i) Xylem and Absorption and conduction of water.
(ii) Ciliary body and Alteration of the shape of the eye lens.
(iii) Seminiferous tubule and Spermatogenesis.
(iv) Thyroid gland and Secretion of thyroxine and calcitonin.
(v) Eustachian tube and Equalising air pressure.
(g)
(i) The phenomenon of loss of water through a cut stem or injured part of plant is called bleeding.
(ii) Pisum sativum is the scientific name of garden pea, which Mendel used for his experiments.
(iii) A fluid that occupies the larger cavity of the eye ball behind the lens is vitreous humour.
(iv) Oxygen combines with haemoglobin present in RBC and forms oxyhaemoglobin.
(v) Acid rain causes corrosion of the marble or brick surface.
(h)
(1) Allele – Alternate forms of genes (2) Leydig cells – Testosterone
(3) Utriculus – Static equilibrium (4) Snake bite – Tourniquet
(5) Euro IV norms – Control of automobile exhaust

SECTION II (40 Marks)


Attempt any four questions from this Section
Question 2
(a) The diagram below shows two test-tubes A and B. Test-tube A contains a green water plant. Test-
tube B contains both a green water plant and a snail. Both testtubes are kept in sunlight. Answer the
questions that follow:

Bubbles of
oxygen

Green water plant

Snail
A B
(i) Name the physiological process that releases the bubbles of oxygen.
(ii) Explain the physiological process as mentioned above in Q.2 (a)(i).
(iii) What is the purpose of keeping a snail in test-tube ‘B’?
(iv) Why does test-tube ‘B’ have more bubbles of oxygen?
(v) Given an example of a water plant that can be used in the above process.
(vi) Write the overall chemical equation for the above process. [5]
(b) Give the biological/technical terms for the following:
(i) A mixture of smoke and fog.
(ii) Capacity of our body to resist diseases.
Bio 251 2015
(iii) Fixing of developing zygote on the uterine wall.
(iv) The permanent stoppage of menstruation at about the age of 45 years in a female.
(v) The hormone increasing reabsorption of water by kidney tubules.
(vi) A thin membrane covering the entire front part of the eye.
(vii) The lens of eye losing flexibility resulting in a kind of long-sightedness in middle aged people.
(viii) The number of persons living per square kilometre at any given time.
(ix) The sound produced when the atrio-ventricular valves close in the heart.
(x) The process by which white blood cells engulf bacteria. [5]
Answer :
(i) Photosynthesis releases the bubbles of oxygen.
(ii) Photosynthesis is a physiological process by which plant cells containing chlorophyll produce
food in the form of carbohydrates by using carbon dioxide, water and light energy. Oxygen is
released as a by-product.
(iii) Carbon dioxide released by the snail during respiration is used by the plant for photosynthesis. This
increases the rate of photosynthesis in the plant placed in test tube B. This also suggests that both
respiration and photosynthesis are complementary processes to maintain the concentration of
oxygen and carbon dioxide in the atmosphere.
(iv) In test tube B, a plant and a snail are kept. The plant in test tube B has more concentration of CO 2
available because the snail releases CO2 during respiration. This increases the rate of photosynthesis
in the plant placed in test tube B which leads to the release of more amount of oxygen.
(v) Hydrilla
(vi) Chemical equation for photosynthesis:
Light energy
6CO2 + 12H2O Chlorophyll
C6H12O6 + 6H2O + 6O2 
(b)
(i) A mixture of smoke and fog – Smog
(ii) Capacity of our body to resist disease – Immunity
(iii) Fixing of developing zygote on the uterine wall – Implantation
(iv) The permanent stoppage of menstruation at about the age of 45 years in a female – Menopause
(v) The hormone increasing reabsorption of water by kidney tubules – Anti-diuretic hormone
(Vasopressin)
(vi) A thin membrane covering the entire front part of the eye – Conjunctiva
(vii) The lens of eye losing flexibility resulting in kind of long-sightedness in middle aged people –
Presbyopia
(viii) The number of persons living per square kilometre at any given time – Population density
(ix) The sound produced when atrioventricular valves close in the heart – Lubb sound
(x) The process by which white blood cells engulf bacteria – Phagocytosis
Question 3
(a) An apparatus as shown below was set up to investigate a physiological process in plants. The
setup was kept in sunlight for two hours. Droplets of water were then seen inside the bell jar.
Answer the questions that follow:
Bio 252 2015

Water Droplets

Bell Jar

Covered Betted Plant

(i) Name the process being studied.


(ii) Explain the process named above in Q.3(a)(i).
(iii) Why was the pot covered with a plastic sheet?
(iv) Suggest a suitable control for this experiment.
(v) Mention two ways in which this process is beneficial to plants.
(vi) List three adaptations in plants to reduce the above mentioned process. [5]
(b) Briefly answer the following questions:
(i) State two reasons for the increase of population in India.
(ii) What is the significance of amniotic fluid?
(iii) What is the function of ear ossicles?
(iv) Mention any two activities of the WHO.
(v) State Mendel’s law of Dominance. [5]
Answer :
(a)
(i) Transpiration
(ii) Transpiration is a process during which water is lost in the form of water vapour through the aerial
parts of the plant. All plants are continously absorb water through their roots. More than 90% of
this water is lost to the atmosphere as water vapour during transpiration.
(iii) The pot is covered with a plastic sheet to prevent the evaporation of water from the soil.
(iv) A control for this experiment will be an empty polythene bag with its mouth tied.
(v) Transpiration is beneficial to plants in the following ways:
 It creates a suction force in the stem which enables the roots to absorb water and minerals.
 It helps in cooling the plant in hot weather.
(vi) Adaptations in plants to reduce transpiration are
 Leaves may be modified into spines as in cactus or into needles as in pines.
 The number of stomata is reduced and they may be sunken in pits.
 Leaves may be folded or rolled up.
(b)
(i) Reasons for the increase of population in India:
Bio 253 2015
 Most Indian families desire to have at least one son. Hence, a couple produces several children till
a son is born.
 Most of the rural population is still illiterate, ignorant and superstitious.
Therefore, they do not make any effort to avoid pregnancy.
(ii) Significance of amniotic fluid:
 It protects the embryo from physical damage by jerks and mechanical shocks.
 It also maintains even pressure all around the embryo.
(iii) Function of ear ossicles:
 Ear ossicles once set in vibrations transmit the vibration to the oval window which sets the cochlear
fluid into vibration.
(iv) Activities of WHO:
 Collecting and supplying information about the occurrence of diseases of an epidemic nature.
 Supplying information on the latest developments about the use of vaccines, cancer research,
nutritional discoveries, control of drug addiction and the health hazards of nuclear radiation.
 To promote and support projects for research on diseases.
 To organic compaigns for the control of epedemic and endemic diseases.
(v) Mendel’s law of dominance:
Out of a pair of contrasting characters present together, only one is able to express itself, while the
other remains suppressed.
Question 4
(a) The diagrams given below are cross sections of blood vessels:

(i) Identify the blood vessels A, B and C.


(ii) Name the parts labelled 1 to 3.
(iii) Name the type of blood that flows through A.
(iv) Mention one structural difference between A and B.
(v) In which of the above vessels does exchange of gases actually take place? [5]
(b) Differentiate between the following pairs on the basis of what is mentioned within brackets:
(i) Diffusion and Osmosis (Definition)
(ii) RBC and WBC (Shape)
(iii) Tubectomy and Vasectomy (Part cut and tied)
Bio 254 2015
(iv) Vasopressin and Insulin (Deficiency disorder)
(v) Rods and Cones of Retina (Type of pigment) [5]
Answer :
(a)
(i) A – Artery
B – Vein
C – Blood capillary
(ii) 1 – External layer of connective tissue / Tunica externa
2 – Lumen
3 – Muscular tissue
(iii) Oxygenated blood flow through A
(iv) Structure difference between A (artery) and B (vein):
A (Artery) : Valves are not present / Thick muscular layer / Narrow lumen.
B (Vein) : Valves are present / Thin muscular layer / Wide lumen.
(v) In blood capillaries, the exchange of gases takes place.
(b)
(i) Diffusion and Osmosis (Definition):
Diffusion Osmosis
Diffusion is the free movement of molecules Osmosis is the diffusion of water molecules
of a substance from the region of its higher across a semi-permeable membrane from
concentration to the region of its lower a more dilute solution (with a lower solute
concentration when the two substances concentration) to a less dilute solution
are in direct contact. (with a higher solute concentration).
(ii) RBC and WBC (Shape):
RBC : RBCs are disc-like, flat at the centre and round at the periphery.
WBC : WBCs are amoeboid or irregular in shape.
(iii) Tubectomy and Vasectomy (Part cut and tied):
Tubectomy : Fallopian tubes or oviducts are cut and ligated and then tied with nylon threads to
close the passage of the egg.
Vasectomy : Vas deferentia each from testis are cut and ligated then both the cut sides are tied
with nylon threads.
(iv) Vasopressin and Insulin (Deficiency disorder):
Vasopressin : Insufficient secretion of vasopressin causes diabetes insipidus.
Insulin : Insufficient secretion of insulin causes diabetes mellitus.
(v) Rods and Cones of Retina (Type of pigment):
Rods : Rods produce rhodopsin.
Cones : Cones produce iodopsin.
Question 5
(a) The diagram given below shows a section of human kidney. Study the diagram carefully and
answer the questions that follow:
Bio 255 2015

1
2

(i) Label the parts numbered 1 to 4.


(ii) Why does part ‘2’ have a striped appearance?
(iii) What is the fluid that passes down part ‘4’? Name the main nitrogenous waste present in it.
(iv) Mention the structural and functional units of kidneys.
(v) Name the two major steps in the formation of the fluid mentioned in Q.5 (a) (iii). [5]
(b) Draw neat and labelled diagrams of the following:
(i) Malpighian Capsule (ii) A Myelinated Neuron [5]
Answer :
(a)
(i) 1 – Cortex
2 – Medulla
3 – Renal pelvis
4 – Ureter
(ii) Due to the presence of Henle’s loops and collecting ducts of nephrons.
(iii) Urine passes down part ‘4’, i.e. ureter. The main nitrogenous substance present in it is urea.
(iv) Nephrons are the structural and functional units of the kidneys.
(v) The two major steps in the formation of urine are ultrafiltration and selective reabsorption.
(b)
(i) Malpighian capsule
Bio 256 2015

Afferent Arteriole

Efferent Arteriole
Bowman’s Capsule
Glomerulus

(ii) A myelinated neuron

Dendrites
Dendron
Nissil granules

Axon Cyton
Node of Ranvier
Myelin Sheath

Axon Ending

Question 6
(a) The diagram given below shows the male urinogenital system of a human being. Study the diagram
and answer the questions that follow:

1
2

3
4
5
6

7
8

(i) Label the parts numbered 1 to 8.


(ii) Name the corresponding structure of part (4) in female reproductive system.
(iii) What is the role of part 7? [5]
Bio 257 2015
(b) In a homozygous plant round seeds (R) are dominant over wrinkled seeds (r):
(i) Draw a Punnett square to show the gametes and offspring when both the plants have heterozy-
gous round seeds (Rr).
(ii) Mention the Phenotype and Genotype ratios of the offsprings in F2 generation.
(iii) Name the sex chromosomes in human males and females.
(iv) Briefly explain the term ‘Mutation’.
(v) What is the number of chromosomes in the gametes of human beings? [5]
Answer :
(a)
(i) 1 – Urinary bladder
2 – Ureter
3 – Prostate gland
4 – Vas deferens
5 – Urethra
6 – Testis
7 – Scrotum
8 – Epididymis
(ii) Fallopian tube or oviduct
(iii) Scrotum is a sac-like structure which encloses the testes. Because it is situated outside the body
cavity, it maintains the lower temperature favourable for the production of sperms.
(b)
(i) P1 = Rr P2 = Rr

Gametes = R, r
Offspring = RR, Rr, Rr, rr

(ii) Phenotypic ratio of the offspring in F2 generation: 3:1


Genotypic ratio of the offspring in F2 generation: 1:2:1
(iii) Sex chromosomes in human males are X and Y.
Sex chromosomes in human females are XX.
(iv) Mutation is the sudden change in one or more genes, or in the number or in the structure of
chromosomes.
(v) The number of chromosomes in the gametes of human beings is 23.perlearning.com 8
Question 7
(a) The diagram below represents the human heart
in one phase of its function. Study the diagram
carefully and answer the questions that follow:
Bio 258 2015

1 2

4
5
3

(i) Name the phase.


(ii) Which part of the heart is contracting in this phase?
Give a reason to support your answer.
(iii) Name the parts labelled 1 to 4.
(iv) What type of blood flows through ‘2’?
(v) State the function of the part numbered ‘5’.
(vi) Name the membrane that covers the heart. [5]
(b) Explain the following terms:
(i) Greenhouse effect (ii) Turgor pressure
(iii) Selective reabsorption (iv) Natality (v) Pulse
Answer :
(a) (i) Ventricular Diastole and auricular systole
(ii) Ventricles are contracting in this phase. In the diagram given, tricuspid valves and bicuspid valves
are closed, while the semi-lunar valves are open.
(iii) 1 – Pulmonary artery 2 – Aorta
3 – Bicuspid valve 4 – Aortic Semilunar valve (aortic semilunar valve)
5 – Pulmonary semilunar valve
(iv) Oxygenated blood flows through ‘2’, i.e. aorta.
(v) ‘5’ is pulmonary semilunar valve. It prevents the backflow of blood into the right ventricle at the
time of ventricular diastole.
(vi) Pericardium covers the heart.
(b) (i) Greenhouse effect:
Gases such as CO2, methane, nitric oxide and nitrous oxide in the atmosphere act as greenhouse
gases. Their increased concentration in the atmosphere prevents the escape of heat which warms
the air. This is called greenhouse effect.
(ii) Turgor pressure:
In a turgid plant cell, the pressure of the cell contents on the cell wall is called the turgor pressure.
(iii) Selective reabsorption:
The glomerular filtrate entering the renal tubule contains many useful substances.
Hence, as the filtrate passes down the tubule, water and other substances required by the body are
reabsorbed. This reabsorption occurs only to the extent that the normal concentration of the blood
is undisturbed. This entire process is called selective reabsorption.
(iv) Natality : The number of live births per 1000 people of population per year is called natality.
(v) Pulse : The pulse is the alternate expansion and elastic recoil of the wall of the artery during
ventricular systole.
ICSE QUESTION PAPER-2016 (SOLVED)
BIOLOGY
SCIENCE Paper – 3
(One hour and a half)
Answers to this Paper must be written on the paper provided separately.
You will not be allowed to write during the first 15 minutes.
This time is to be spent in reading the Question Paper,
The time given at the head of this Paper is the time allowed for writing the answers.
Attempt all questions from Section I and any four questions from Section II.
The intended marks for questions or parts of questions are given in brackets [ ].

SECTION I (40 Marks)


Attempt all questions from this Section
Question 1.
(a) Name the following:
(i) The exchange of chromatid parts between the maternal and the paternal chromatids of a pair of
homologous chromosomes during meiosis.
(ii) The number of individuals inhabiting per unit area.
(iii) The immunity acquired by providing readymade antibodies from outside for treating certain
infectious diseases.
(iv) The pollutants that cannot be broken down to simple and harmless products.
(v) The part of the brain that carries impulses from one hemisphere of the cerebellum to the other.
(b) Choose the correct answer from each of the four options given below: (5)
(1) A plant cell may burst when:
A. Turgor pressure equalises wall pressure.
B. Turgor pressure exceeds wall pressure.
C. Wall pressure exceeds turgor pressure.
D. None of the above
(ii) The individual Flattened stacks of membranous structures inside the chloroplasts are known a:
A. Grana B. Stroma
C. Thylakoids D. Cristae
(iii) The nephrons discharge their urine at the:
A. Urinary bladder B. Urethra
C. Renal pelvis D. Renal pyramid
(iv) Gigantism and Acromegaly are due to :
A. Hyposecretion of Thyroxine. B. Hyposecretion of Growth Hormone.
C. Hypersecretion of Thyroxine. D. Hypersecretion of Growth Hormone.
(v) The mineral ion needed for the formation of blood clot is:
A. Potassium B. Sodium
C. Calcium D. Iron
Bio 259 2016
Bio 260 2016
(c) In each set of terms given below, there is an odd one and cannot be grouped in the same category
to which the other three belong, identify the odd term in each set and name the category to which
the remaining three belong. [5]
Example: Ovary. Fallopian tube. Ureter. Uterus.
Odd term: Ureter
Category: Parts of female reproductive system.
(1) Sewage, Newspaper. Styrofoam, Hay.
(ii) Thymine, Cytosine. Adenine, Pepsin.
(iii) Malleus, Iris. Stapes, Incus.
(iv) Cortisone, Somatotropin, Adrenocorticotropic hormone, Vasopressin.
(v) Typhoid, Haemophilia, Albinism, Colour blindness.
(d) Complete the following paragraph by filling in the blanks (i) to (v) with appropriate words: [5]
The amount of urine output is under the regulation of a hormone called (i)___________ secreted
by the (ii) ___________ lobe of the pituitary gland. If this hormone secretion is reduced, there is
an increased production of urine. This disorder is called (iii); ___________ . Sometimes excess
glucose is passed with urine due to hyposecretion of another hormone called (iv) ___________
leading to the cause of a disease called (v)__________ .
(e) State the exact location of the following structures: [5]
(i) Centromere (ii) Chordae tendinae
(iii) Thyroid gland (iv) Ciliary body
(v) Proximal convoluted tubule.
(f) Given below is a diagram depicting a defect of the human eye, study the same and then answer
the questions that follow: [5]

(i)Name the defect shown in the diagram.


(ii)What are the two possible reasons that cause this defect?
(iii)Name the type of lens used to correct this defect.
(iv) With the help of a diagram show how the defect shown above is rectified using a suitable lens.
(g) Given in the box below are a set of 14 biological terms. Of these, 12 can be paired into 6 matching
pairs. Out of the six pairs, one has been done for you as an example. [5]
Example : endosmosis - Turgid cell.
Identify the remaining five matching pairs:
Cushing’s syndrome, Turgid cell, Iris, Free of rod and cone cells, Colour of eyes, Hypoglycemia,
Active transport, Acrosome, Addison’s disease, Blind spot, Hyperglycemia, Spermatozoa
Endosmosis, Clotting of blood.
(h) State the main function of the following: [5]
(i) Lymphocytes of blood (ii) Leydig cells
Bio 261 2016
(iii) Guard cells (iv) Eustachian tube (v) Corpus luteum
Answer:
(a) (i) Crossing over (ii) Population Density (iii) Passive Immunity
(iv) Non biodegradable (v) Pons varolii
(b) (i) B (ii) C (iii) C
(iv) D (v) C
(c) (i) Styrofoam, All others are biodegradable pollutants.
(ii) Pepsin, All others are nitrogenous bases present in nucleic acids
(iii) Iris, All others are earossicles
(iv) Cortisone, All others are secreted by the pituitary gland
(v) Typhoid, All others are congenital genetic disorders.
(d) (i) Antidiuretic/Vasopressin Hormone (ii) Posterior
(iii) Diabetes insipidus/Diuresis (iv) Insulin
(v) Diabetes mellitus
(e) (i) Between two sister chromatids of one chromosome
(ii) Attach free margins of bicuspid and tricuspid valves with the papillary muscles
(iii) In front of the neck just below the larynx.
(iv) Extension of the choroid of the eye.
(v) In the cortex region of the kidney between the Malpighian capsule and loop of Henle.
(f) (i) Myopia/Near-Sightedness
(ii) Excessive thickening of cornea and increase in the length of eye ball from front to back
(iii) Concave lens
(iv)

(g) (i) Cushing’s syndrome : Hyperglycemia


(ii) Free of rod and cone cells : Blind spot
(iii) Colour of eye : Iris
(iv) Hypoglycemia : Addison’s disease
(v) Acrosome: Spermatozoa
(h) (i) Production of antibodies to provide specific immunity.
(ii) Secrete male sex hormones called Testosterone
(iii) Help in opening and closing of stomata.
(iv) Act as ventilators to equalize air pressure on the two sides of tympanum.
(v) Secrete progesterone hormone to prepare the uterus for Implantation of Zygote.
Bio 262 2016

SECTION — II (40 Marks)


Attempt any four questions from this Section
Question 2.
(a) The figure given below is a diagrammatic representation of a part of the cross section of the root
in the root hair zone. Study the same and then answer the questions that follow: [5]

2
1

3 4
(i) Name the parts indicated by the guidelines 1 to 4.
(ii) Which is the process that enables the passage of water from the soil into the root hair?
(iii) Name the pressure that is responsible for the movement of water in the direction indicated by the
arrows. Define it.
(iv) Due to an excess of this pressure sometimes drops of water are found along the leaf margins of
some plants especially in the early mornings. What is the phenomenon called?
(v) Draw a well labelled diagram of the root hair cell as it would appear if an excess of fertiliser is
added to the soil close to it.
(b) Differentiate between the following pairs on the basis of what is mentioned within brackets : [5]
(i) Human skin cell and Human ovum (number of chromosomes)
(ii) Sperm duct and fallopian tube (function)
(iii) Red Cross and WHO (one activity)
(iv) Rod cells and cone cells (pigment)
(v) LUBB and DUP (names of the valves whose closure produce the sound)
Answer:
(a) (i) 1-Root hair 2-Soil particles 3-Xylem vessel 4-Cortex cells
(ii) Endosmosis, water potential gradient.
(iii) Osmotic pressure, Root pressure. It is the pressure that develops in the confined part of an
osmotic system due to osmatic entry of water into it.
(iv) Guttation
(v) Shrunken
Protoplasm

Flaccid root hair

(b) (i) Human skin cell (23 pairs) is diploid whereas human ovum (23) is haploid.
(ii) Sperm duct is meant for the passage of sperms from testis to urethra whereas fallopian tubes
has funnel shaped opening which receives the mature eggs from the ovary which are carried by
it to the uterus.
Bio 263 2016
(iii) WHO is concerned with the strengthening of health services whereas Red Cross is mainly concerned
with extending relief and help to the victims of any natural calamity.
(iv) Rod cells contain rhodopsin pigment whereas cone cells contain iodopsin pigment.
(v) LUBB is produced due to the closure of AV valves whereas DUP is produced due to the closure of
semi-lunar valves.
Question 3.
(a) Given below is the outline of the human body showing the important glands: [5]
1
2

3
4

(i)Name the glands marked I to 4.


(ii)Name the hormone secreted by part 2. Give one important function of this hormone.
(iii)Name the endocrine part of the part numbered 3.
(iv) Why is the part labelled 1 called the master gland? Which part of the forebrain controls the gland
labelled 1 ?
(v) Name the gland that secretes the ‘emergency hormone’.
(b) The diagram of an apparatus given below demonstrates a particular process in plants. Study the
same and answer the questions that follow: [5]

Leafy Shoot

Water Reservoir
Air Bubble

Graduated
Capillary Coloured
Tube water

(i) Name the apparatus.


(ii) Which phenomenon is demonstrated by this apparatus?
(iii) Explain the phenomenon mentioned in (ii) above.
(iv) State two limitations of using this apparatus.
Bio 264 2016
(v) What is the importance of the air bubble in the experiment?
(vi) Name the structures in a plant through which the above process takes place.
Answer:
(a) (i) 1-Pituitary gland, 2-Thyroid gland, 3-Pancreas, 4-Adrenal gland
(ii) Thyroxine hormone, it controls basal metabolic rate.
(iii) Islet of Langerhans.
(iv) Pituitary gland is known as the master gland of the body because it controls other endocrine
glands by the secretion of trophic hormones. Flour of diencephalon called hypothalamus controls
the pituitary gland by the secretion of releasing and inhibiting factors/hormones.
(v) Emergency hormone is secreted by the adrenal gland.
(b)
(i) Ganong’s potometer.
(ii) Rate of transpiration.
(iii) The loss of water in the form of vapours from the aerial exposed parts of a plant is called
transpiration.
(iv) The first limitation of this apparatus is that it is very difficult to introduce the air bubble at the
lower end of the capillary tube. Secondly, various changes in the environment such as, temperature
humidity, air velocity etc. may influence the position of air bubble.
(v) Rate of water uptake can be calculated by timing the air bubble movement over a fixed distance on
the scale.
(vi) Transpiration takes place through stomata, cuticle and lenticels.
Question 4.
(a)
(i) Draw a well labelled diagram of the membranous labyrinth found in the inner ear. [5]
(ii) Based on the diagram drawn above in (i) give a suitable term for each of the following descriptions:
1. The sensory cells that helps in hearing.
2. The part that is responsible for static balance of the body.
3. The membrane covered opening that connects the middle ear to the inner ear.
4. The fluid present in the middle chamber of cochlea.
5. The structure that maintains dynamic equilibrium of the body.
(b) Give the Biological / technical term for the following: [5]
(i) Complete stoppage of menstrual cycle in females.
(ii) Pigment providing colour to urine.
(iii) The vein which drains the blood from the intestine to the liver.
(iv) The canal through which the testes descend into the scrotum just before the birth of a male baby.
(v) The process causing an undesirable change in the environment.
(vi) The removal of nitrogenous wastes from the body.
(vii) The repeating components of each DNA strand lengthwise.
(viii) An alteration in the genetic material that can be inherited.
(ix) The process of uptake of mineral ions against the concentration gradient using energy from the
cell.
(x) Blood vessels carrying blood to the left atrium.
Bio 265 2016
Answer:
(a) (i) Semicircular duct
Saccule
Cochlea

ampulla
utricle Cochlear duct
A

(ii) 1- of Organs of Corti, 2-uriculus and sacculus, 3-round window,


4-Endolymph, 5-semicircular canal
(b) (i) Menopause (ii) Urochrome (iii) Hepatic portal vein
(iv) Inguinal canal (v) Pollution (vi) Excretion
(vii) Nucleotides (viii) Mutation
(ix) Active transport (x) Pulmonary veins
Question 5.
(a) The given diagram shows a stage during mitotic division in an animal cell: [5]

792

(i) Identify the stage. Give a reason to support your answer.


(ii) Draw a neat labelled diagram of the cell as it would appear in the next stage. Name the stage.
(iii) In what two ways is mitotic division in an animal cell different from the mitotic division in a plant
cell?
(iv) Name the type of cell division that occurs during:
A. Growth of a shoot B. Formation of pollen grains.
(b) Give scientific reasons for the following statements:
(i) Colour blindness is more common in men than in women.
(ii) Injury to medulla oblongata leads to death.
(iii) When an ovum gets fertilized, menstrual cycle stops temporarily in a woman.
(iv) Mature erythrocytes in humans lack nucleus and mitochondria.
(v) Blood flows in arteries in spurts and is under pressure.
Bio 266 2016
Answer:
(a) (i) Prophase-Chromosomes have duplicated, nuclear membrane is disappearing, nucleus has
disappeared.
(ii) Next stage is Metaphase as shown below :

(iii) Animal cell Plant cell


1. Asters are formed. 1. Asters are not formed.
2. Cytokinesis by formation of furrow in 2. Cytokinesis by cell plate Formation.
the cytoplasm.
3. Occurs in most tissues of the Whole body. 3. Occurs mainly at the at the growing
tips and sides.
(iv) A-Mitosis and B-Meiosis.
(b)
(i) Colourblindness is more common in men as compared to women because it is a sex-linked and
recessive trait. It need two alleles for its expression in women as she contains 2 X-chromosomes
whereas only one X-chromosome is present in males, hence a single gene expresses it.
(ii) Injury to medulla oblongata may lead to death as it controls the involuntry actions like breathing
movements and beating of heart.
(iii) When fertilization takes place, the corpus luteum is maintained which secretes large amounts of
progesterone hormone. This hormone maintains the endometrium of the uterus and prevents its
rupturing. Hence, no menstruation takes place.
(iv) Absence of nucleus increases surface area for absorption and transport of O2 to tissues. No
cellular respiration as mitochondria is absent and hence does not use O2 for itself.
(v) because of the rhythmic ventricular contractions and the narrow lumen of arteries.
Question 6.
(a) The diagram given below is that of a developing human foetus. Study the diagram and then
answer the questions that follow: [5]
Bio 267 2016

794

(i)
Label the parts numbered 1 to 3 in the diagram.
(ii)
Mention any two functions of the part labelled 2 in the diagram.
(iii)
Explain the significance of the part numbered 3 in the diagram.
(iv)Define the term ‘Gestation’. What is the normal gestational period of the developing human
embryo?
(v) Mention the sex chromosomes in a male and female embryo.
(b) The following diagram demonstrates a physiological process taking place in green plants. The
whole set up was placed in bright sunlight for several hours. Study the diagram and answer the
questions that follow: [5]

794

(i) What aspect of the physiological process is being examined?


(ii) Explain the physiological process mentioned in (i) above.
(iii) Label the parts numbered 1 and 2 in the diagram.
(iv) Write a well-balanced chemical equation for the physiological process explained in (ii) above.
(v) What would happen to the rate of bubbling of the gas if a pinch of sodium bicarbonate is added to
the water in the beaker? Explain your answer.
Bio 268 2016
Answer:
(a)
(i) 1-Umbilical cord, 2-Placenta, 3-Amniotic cavity full of amniotic fluid
(ii) – Diffusion of O2 antibodies and nutrients from maternal blood to foetal blood.
– Diffusion of carbon dioxide, urea, uric acid from foetal blood to maternal blood
– Acts as a barrier and prevents germs from entering foetal blood
– Secretes Oestrogen and progesterone
(iii) Amniotic fluid maintains even temeperature around the foetus. It also prevents the foetus from
jerks.
(iv) Pregnancy or gestation is the process or period of development of embryo inside the womb of the
mother between conception and parturition in viviparous mammals including humans.
For humans it is of 40 weeks.
(v) Sex chromosome in male embryo is XY whereas in female it is XX.
(b)
(i) Oxygen is released during photosynthesis.
(ii) Photosynthesis is a physiological process during which green plants synthesize complex organic
compounds (Glucose) from simple inorganic compounds- CO2 and H2O in the presence of sunlight
and chlorophyll.
(iii) 1-Oxygen, 2- aquatic plant
Sun Light
(iv) 6CO2 + 12H2O  C6H12O6+ 6H2O+ 6O2
Chlorophyll

(v) Rate of bubbling will increase by the addition of a pinch of sodium bicarbonate. Plants get more
CO2, hence rate of photosynthesis increases.
Question 7.
(a) A homozygous tall plant (T) bearing red coloured (R) flowers is crossed with a homozygous
dwarf (t) plant bearing white (r) flowers [5]
(i) Give the genotype and phenotype of the plants of F1 generation.
(ii) Mention the possible combinations of the gametes that can be obtained from the F1 hybrid plant.
(iii) State the Mendel’s law of Independent Assortment.
(v) Mention the phenotypes of the offsprings obtained in F2 generation.
(vi) What is the phenotypic ratio obtained in F2 generation?
(b) Briefly explain the following terms: [5]
(i) Reflex action (ii) Power of accommodation
(iii) Photophosphorylation (iv) Hormone
(v) Synapse
Answer:
(a) (i) Genotype of F1 plants : TtRr and the Phenotype will be Tall and red flowered.
(ii) The possible combination of gametes of F1 plants TR, Tr, tR and tr.
Bio 269 2016
(iii) Mendel’s law of independent assortment states that the factors or genes controlling different
characters assort independently without influencing each other during the formation of gametes.
(iv) Phenotypes in F2 generation:
Tall plants with red flowers, Tall plants with white flowers, Dwarf plants with red flowers and
Dwarf plants with white flowers’
(v) Phenotypic ratio in F2 generation : 9:3:3:1
(b)
(i) Reflex action is the spontaneous, automatic and stereotyped response to stimuli without consulting
the will power.
(ii) Ability of the eye to see the object equally clear from various distances is called Power of
accommodation.
(iii) Photophosphorylation is the synthesis of ATP from ADP and phosphate that occurs in a plant
using radiant energy absorbed during photosynthesis.
(iv) Hormone is a regulatory substance produced in an organism and transported in tissue fluids such
as blood or sap to stimulate specific cells or tissues into action.
(v) Close proximity between the terminal arborization of axon of one neuron and dendrons of another
neuron without any physical contact is called synapse.
ICSE QUESTION PAPER-2017 (SOLVED)
BIOLOGY
SCIENCE Paper – 3
(One hour and a half)
Answers to this Paper must be written on the paper provided separately.
You will not be allowed to write during the first 15 minutes.
This time is to be spent in reading the Question Paper,
The time given at the head of this Paper is the time allowed for writing the answers.
Attempt all questions from Section I and any four questions from Section II.
The intended marks for questions or parts of questions are given in brackets [ ].

SECTION I (40 Marks)


(Attempt all questions from this section)

Question 1.
(a) Name the following :
(i) The process by which root hairs absorb water from the soil.
(ii) The organ which produces urea.
(iii) The kind of lens required to correct Myopia.
(iv) The pituitary hormone which stimulates contraction of uterus during child birth.
(v) The international health organisation which educates people in accident prevention. (5)
Ans. (i) Endosmosis (ii) Liver
(iii) Concave/Diverging lens (iv) Oxytocin
(v) Red Cross
(b) Choose the correct answer from each of the four options given below : (5)
(i) The prime source of chlorofluorocarbons is
A. Vehicular emissions
B. Industrial effluents
C. Domestic sewage
D. Refrigeration equipment
(ii) Penicillin obtain from a fungus is
A. Antibiotic B. Antiseptic
C. Antibody D. Antiserum
(iii) Marine fish when placed in tap water bursts because of
A. Endosmosis B. Exosmosis
C. Diffusion D. Plasmolysis
(iv) Surgical method of sterilization in a woman involves cutting and tying of
A. Ureter B. Uterus
Bio 270 2017
Bio 271 2017
C. Urethra D. Oviduct
(v) Synthesis phase in the cell cycle is called so, because of the synthesis of more :
A. RNA B. RNA and proteins
C. DNA D. Glucose
Ans. (i) Refrigeration equipment.
(ii) Antibiotic
(iii) Endosmosis
(iv) Oviduct
(v) DNA
(c) The statements given below are incorrect. Rewrite the correct statement by changing the
underlined words of the statements.
(i) The Graafian follicle, after ovulation turns into a hormone producing tissue called Corpus
callosum.
(ii) Deafness is caused due to the rupturing of the Pinna.
(iii) Gyri and Sulci are the folds of Cerebellum.
(iv) Free movement of solutes in and out of the cell takes place across the cell membrane.
(v) The solvent used to dissolve the chlorophyll pigments while testing a leaf for starch is Soda
lime.
Ans. (i) The Graafian follicle, after ovulation turns into a hormone producing tissue called Corpus
luteum.
(ii) Deafness is caused due to the rupturing of the Tyumpanum/Ear drum.
(iii) Gyri and Sulci are the folds of Cerebrum.
(iv) Free movement of solutes in and out of the cell takes place across the cellwall.
(v) The solvent used to dissolve the chlorophyll pigments while testing a leaf for starch is methy-
lated spirit.
(d) Given below are sets of five terms each. Rewrite the terms in correct order in a logical se-
quence.
Example : Large intestine, Stomach, Mouth, Small intestine, Oesophagus.
Answer : Mouth  Oesophagus  Stomach  Small intestine  Large intestine.
(i) Fibrin, Platelets, Thromboplastin, Fibrinogen, Thrombin.
(ii) Cochlea, Malleus, Pinna, Stapes, Incus.
(iii) Receptor, Spinal cord, Effector, Motor neuron, Sensory neuron.
(iv) Uterus, Parturition, Fertilisation, Gestation, Implantation.
(v) Caterpillar, Snake, Owl, Frog, Green leaves.
Ans. (i) Platelets  Thromboplastin  Thrombin  Fibrinogen  Fibrin.
(ii) Pinna  Malleus  Incus  Stapes  Cochlea.
(iii) Receptor  Sensory neuron  Spinal cord  Motor neuron  Effector..
(iv) Fertilisation  Uterus  Implantation  Gestation  Parturition.
Bio 272 2017
(v) Green leaves  Catterpillar  Frog  Snake  Owl.
(e) Choose the ODD one out of the following terms given and name the CATEGORY to which the
others belong : (5)
(i) Aqueous humour, Vitreous humour, Iris, Central canal
(ii) Formalin, Iodine, DDT, Lime
(iii) ACTH, TSH, ADH, FSH
(iv) Phosphate, RNA, Sugar, Nitrogenous base
(v) Bile, Urea, Uric acid, Ammonia
Ans.
Odd term Category
(i) Central canal Parts of eye
(ii) Iodine Disinfectants
(iii) ADH Harmones of anterior lobe of pituitary.
(iv) RNA Components of nucleotide.
(v) Bile Nitrogenous waste excreted with urine.
(f) Given below are groups of terms. In each group the first pair indicates the relationship between
the two terms. Rewrite and complete the second pair on a similar basis. (5)
Example : Oxygen : Inspiration : : Carbon dioxide : Expiration.
(i) Eye Optic nerve : Ear : .............
(ii) Cytoplasm : Cytokinesis :: Nucleus : .............
(iii) TT : Homozygous : Tt : .............
(iv) Foetus : Amnion : : Heart : .............
(v) Adenine : Thymine : : Cytosine : .............
Ans. (i) Auditory curve. (ii) Karyokinesis.
(iii) Heterozygous (iv) Pericardium
(v) Guanine
(g) Match the items given in Column A with the most appropriate ones in Column B and rewrite the
correct matching pairs.
Column A Column B
1. Sacculus — dynamic body balance
2. Birth rate — Hyperglycemia
3. DNA and histones — Hypoglycemia
4. Euro norms — Natality
5. Diabetes mellitus — static body balance
— vehicular standards
— nucleosome
Bio 273 2017
Ans.
Column A Column B
1. Sacculus —Static body balance
2. Birth rate — Natality
3. DNA and histones — Nucleosome
4. Euro norms — Vehicular standards
5. Diabetes mellitus — Hyperglycemia
(h) The diagram given below represents the location and structure of an endocrine gland. Study the
same and answer the questions that follow :

(i) Name the endocrine gland shown in the diagram.


(ii) Name the secretion of the gland which regulates basal metabolism.
(iii) Name the mineral element required for the synthesis of the above mentioned hormone.
(iv) Name the disease caused due to under secretion of the above mentioned hormone in children.
(v) Name the disease caused due to hypersecretion of the above mentioned hormone.
Ans.
(i) The endocrine gland shown is the thyroid gland.
(ii) The secretion of the gland which regulates basal metabolism is thyroxine.
(iii) The mineral element required is iodine.
(iv) The disease caused due to under secretion of the hormone in children is cretinism.
(v) The disease caused due to hypersecretion of the above mentioned hormone is exopthalmic
goitre.

SECTION II (40 Marks)


(Attempt any four questions from this section)

Question 2
(a) Study the diagram given below which represents a stage during the mitotic cell division and
answer the questions that follow : (5)
Bio 274 2017

(i) Identify the stage giving suitable reasons.


(ii) Name the parts numbered 1 and 2.
(iii) What is the technical term for the division of nucleus ?
(iv) Mention the stage that comes before the stage shown in the diagram. Draw a neat labelled
diagram of the stage mentioned.
(v) Which is the cell division that results in half the number of chromosomes in daughter cells ?
Ans. (i) The stage is Telophase. The reasons are as follows : (a) Chromatids thin out in the form of
chromatin fibres, (b) Nucleoli and nuclear membrane reappear, (c) the cleavage furrow has
started deepening.
(ii) 1. Chromosome 2. Nuclear membrane
(iii) The technical term for division on nucleus is Karyokinesis.
(iv) The stage which comes before the stage shown in the diagram is Anaphase.

Chromatid

Centromere
Centriole
Aster

(v) The cell division that results in half the number of chromosome in daughter cells is meiosis.
(b) Differentiate between the following pairs on the basis of what is mentioned within brackets :
(5)
(i) Active Transport and Diffusion [significance in plants]
(ii) Demography and Population density [Definition]
(iii) Antibiotic and Antibody [Source]
(iv) Renal cortex and Renal medulla [Parts of the nephrons present]
(v) NADP and ATP [Expand the abbreviation]
Bio 275 2017
Ans.
Active Transport Diffusion
(i) Absorption of certain ions which serve Gaseous exchange across stomata in
as nutrients and maintain high osmotic processes like transpiration, respiration and
pressure for absorbing water. photosynthesis.
Demography Population density
(ii) It is the statistical study of human It is the number of individuals per square
population with reference to size, kilometre at any given time.
density, distribution and other vital
statistics.
Antibiotic Antibody
(iii) Antibiotic are chemical substances Antibody are special chemicals found in the
produced by certain micro-organisms, blood and produced by a type of specialised
e.g. Penicillium produces penicillin. lymphocytes on exposure to antigens.
Renal cortex Renal medulla
(iv) Malpighian capsule, PCT and DCT lies Loop of Henle lie in the renal medulla.
in renal cortex.
NADP ATP
(v) Nicotinamide adenine dinucleotide Adenosine triphosphate
phosphate

Question 3
(a) The diagram given below represents a plant cell after being placed in a strong sugar solution.
Study the diagram and answer the questions that follow : (5)
1

4
2
(i) What is the state of the cell shown in the diagram ?
Bio 276 2017
(ii) Name the structure that acts as a selectively permeable membrane.
(iii) Label the parts numbered 1 to 4 in the diagram.
(iv) How can the above cell be brought back to its original condition ? Mention the scientific term
for the recovery of the cell.
(v) State any two features of the above plant cell which is not present in animal cells.
Ans. (i) The state of the cell is plasmolysed/extreme flaccidity.
(ii) The structure that acts as a selectively permeable membrane is cell membrane or plasma mem-
brane.
(iii) 1. — Cell wall 2. —Strong sugar solution
3. — Cell membrane 4. — Nucleus
(iv) The cell can be brought back to its original condition by putting it in a hypotonic environment/
pure water.
(v) The two features present in a plant cell is, (a) presence of large vacuole (b) Chloroplast. These
are not present in animal cell.
(b) Give below is a representation of kind of pollution. Study the same and answer the questions
that follow : (5)

(i) Name the kind of pollution.


(ii) List any three common sources of this pollution.
(iii) Mention three harmful effects of this pollution on human health.
(iv) Explain the term ‘Pollutant’.
(v) Name two soil pollutants.
Ans. (i) The kind of pollution is noise pollution.
(ii) Three common sources of noise pollution are
(a) Industrial machines (b) Loudspeakers
(c) Automobiles on the street and trains.
(d) Jet aircrafts
Bio 277 2017
(iii) Three harmful effects of noise pollution are :
(a) Disturbs sleep and leads to nervous irritability.
(b) Lowers efficiency of work as it affects concentration of thought.
(c) Prolonged loud noise can lead to deafness as it may damage the ear drum or tympanum.
(iv) A Pollutant is any such constituent which causes pollution, e.g. dust, smoke.
(v) Two soil pollutants —
(a) Industrial waste (b) Chemical fertilizers
(c) Pesticides
Question 4.
(a) The diagram given below represent the relationship between a mouse and a physiological pro-
cess that occurs in green plants. Study the diagram and answer the questions that follow : (5)

SUNLIGHT

SUNLIGHT Mouse - Dead


A B

Green Plant

Mouse - Alive

C D

(i) Name the physiological process occurring in the green plant that has kept the mouse alive.
(ii) Explain the physiological process mentioned above.
(iii) Why did the mouse die in bell jar B ?
(iv) What is the significance of the process as stated in (i) for life on earth ?
(v) Represent the above mentioned physiological process in the form of a chemical equation.
Ans. (i) The physiological process occurring in the green plants is photosynthesis.
(ii) Photosynthesis is a process by which living plant cells containing chlorophyll produce an or-
ganic food substance glucose from carbon dioxide and water by using light energy and release
oxygen as a by-product.
Bio 278 2017

Light energy
6CO2 + 12H2O 
Chlorophyll C6H12O6 + 6H2O + 6O2 

(iii) The mouse as well as the burning candle utilised the oxygen present inside the air in the bell jar.
Both the processes going on inside the bell jar liberated carbon dioxide. No living organism can
survive in an environment of carbon dioxide. So the mouse died.
(iv) The significance of the process is as follows :
(a) Provides food for all organisms — All animals and even human beings depend on plants for
food either directly or indirectly. In any food chain the starting point is always a green plant
(producer).
(b) Provides oxygen — The life supporting gas oxygen is present in the atmosphere in a free, stable
form. This is utilised by living organisms.
(c) Carbon dioxide and oxygen proportion in the atmosphere is maintained.
(b) Mention the exact location of the following : (5)
(i) Prostate gland (ii) Myelin sheath
(iii) Islets of Langerhans (iv) Semi-circular canals
(v) Eustachian tube
Ans. (i) It is present around the urethra when it emerges from the bladder.
(ii) In most neurons the axon is surrounded by a white insulating sheath known as myelin sheath.
(iii) These are special groups of hormone secreting cells present in the entire pancreas.
(iv) It is present in the inner ear in a set of three which are arranged at right angles to each other.
(v) It is present in the middle ear which connects the cavity of the middle ear to the throat.
Question 5
(a) The diagram shown below is the longitudinal section of a testis of man. Study it carefully and
answer the questions that follow : (5)

1
2
3

(i) Label the parts numbered 1 to 3 in the diagram.


(ii) In which part of the testis are the sperms produced ?
(iii) State the functions of the parts labelled 1 and 3 in the diagram.
(iv) Name the cells that secrete Testosterone.
(v) Draw a neat, labelled diagram of a sperm.
Bio 279 2017
Ans.(i) 1. Scrotum
2. Sperm duct/Vas deferens.
3. Epididymis
(ii) Sperms are produced in the seminiferous tubules.
(iii) 1. The scrotum or scrotal sac is a thin walled maintains a temp of 2 - 3°C less than body temp.
for the maturation of the sperm.
2. It helps to store the sperms during which they mature and become motile.
(iv) The cells which secrete testosterone are Leydig cells.
(v)

Acrosome
Nucleus
Head
Centriole
Mid Mitochondria
piece

Tail

(b) Give biological reasons for the following statements : (5)


(i) Some woman have facial hair like beard and moustache.
(ii) Cutting of tree should be discouraged.
(iii) In some xerophytes leaves are modified into spines.
(iv) There is frequent urination in winter than in summer.
(v) The left ventricle of the heart has a thicker wall than the right ventricle.
Ans. (i) In some mature women there can be an overgrowth of the adrenal cortex. This leads to hyper-
secretion of cortisone which behave like sex hormone and the women develop certain male
characteristics such as beard and moustache.
(ii) Cutting down of trees should be discouraged because deforestation may lead to many problems
like droughts, flash flood, soil erosion, extinction of several wild animals and a threat of extinc-
tion of hundreds of species. So in order to keep the environment unharmed the cutting of trees
should be avoided.
(iii) Xerophytes live in an environment where there is less water or scarcity of water. So in some
xerophytes the leaves are modified into spines in order to stop or cut down transpiration. This
prevents too much of water loss and saves the plants.
(iv) During winter there is no perspiration. Hence there is less water reabsorption from the glom-
erular filtrate resulting in more volume of urine. Hence, there is frequent urination in winter than
in summer.
(v) The right ventricle pumps blood only to the lungs for oxygenation, whereas the left ventricle
pumps blood to the farthest points in the body, like up to the toes in the feet or up to the brain
against gravity. So the left ventricle of the heart has thicker walls than the right ventricle.
Bio 280 2017
Question 6.
(a) The diagram given below represents a section of the human heart. Answer the questions that
follow : (5)

(i) Which parts of heart are in the diastolic phase ? Give a reason to support your answer.
(ii) Label the parts numbered 1 and 2 in the diagram. What type of blood flows through them ?
(iii) What causes the heart sounds ‘LUBB’ and ‘DUP’ ?
(iv) Name the blood vessels that supply oxygenated blood to the heart muscles.
(v) Draw neat labelled diagrams of a cross section of an artery and a vein.
Ans. (i) The ventricles of the heart are in diastolic phase because the figure shows flow of blood from
auricles to ventricles. Here tricupsid and bicupsid values are open / semilunar values are closed.
(ii) 1. — Pulmonary Artery : Type of blood which flow through them are deoxygenated blood.
2. — Pulmonary artery : The type of blood which flows through them is oxygenated.
(iii) During ventricular systole the tricuspid and bicuspid valves close with a faint sound known as
‘LUBB’. During venticular diastole the semilunar valves at the roots of aorta and pulmonary
artery close with a sharp sound ‘DUP’.
(iv) The blood vessel that supply oxygenated blood to the heart muscles is coronary artery.
(v)
Artery Vein
Connective tissue

Muscle layer
figure
Endothelium

Lumen

(b) Give appropriate biological/technical terms for the following : (5)


(i) The type of immunity that exists in our body due to our genetic makeup.
(ii) The suppressed allele of a gene.
Bio 281 2017
(iii) The accessory gland in human males whose secretion activates the sperms.
(iv) An apparatus that measures the rate of water uptake in a cut shoot due to transpiration.
(v) The kind of twins formed from two fertilised eggs.
(vi) A pair of corresponding chromosomes of the same size and shape, one from each parent.
(vii) The mild chemical substance which when applied on the body kills germs.
(viii) The type of waste generated in hospitals and pathological laboratories.
(ix) The antiseptic substance in tears.
(x) Cellular components of blood containing haemoglobin.
Ans. (i) Innate (ii) Recessive
(iii) Seminal vesicle (iv) Potometer
(v) Fraternal twins (vi) Homologous chromosomes
(vii) Antiseptic (viii) Biomedical waste
(ix) Lysozyme (x) Red blood cells/Erythrocyte
Question 7
(a) In a homozygous pea plant, axial flowers (A) are dominant over terminal flowers (a). (5)
(i) What is the phenotype and genotype of the F1 generation if a plant bearing pure axial flowers is
crossed with a plant bearing pure terminal flowers ?
(ii) Draw a Punnett square board to show the gametes and offsprings when both the parent plants
are hetrozygous for axial flowers.
(iii) What is the phenotypic ratio and genotypic ratio of the above cross shown in (ii) ?
(iv) State Mendel’s Law of Dominance.
(v) Name two genetic disorders commonly seen in human males.
Ans. (i) Phenotype of F1 — Hyterozygous axial flowers
Genotype of F1 — Aa (Heterozygous)

(ii) Parents (Phenotype) Heterozygous Heterozygous


axial f lowers axial flowers
Genotype A a A a

Gametes A a A a

(iii) Punnett square

0
0 A a
+
A AA Aa A A – Homozygous axial
A a – Heterozygous axial
a Aa aa A A – Homozygous terminal

You might also like